Police Tests

Published on February 2017 | Categories: Documents | Downloads: 54 | Comments: 0 | Views: 1271
of 339
Download PDF   Embed   Report

Comments

Content

Table of Contents

Table of Contents

POLICE WRITTEN TEST Digital Manual
Police Practice Tests

PRACTICE TEST 1 25 Questions PRACTICE TEST 2 100 Questions PRACTICE TEST 3 83 Questions

PRACTICE TEST 4 100 Questions PRACTICE TEST 5 50 Questions PRACTICE TEST 6 Police Math Test

PRACTICE TEST 7 Police Grammar Test PRACTICE TEST 8 Reading Comp Test PRACTICE TEST 9 PATI Test

***
© CFI Research Company. All Rights Reserved.

Ebook Instructions To Find a Particular Test or Test Answer Key in this Ebook, simply use the Bookmarks option located on the upper left side of this page.

Police Practice Test 1

Page 1 of 13

Police Practice Test 1 / 25 Questions
Multiple Choice Identify the letter of the choice that best completes the statement or answers the question You have 50 minutes to complete the test. Reading Comprehension

1.

According to the rule: Officer Burton should... a. b. c. d.

2. The first face, on the left, is a sketch of an alleged criminal based on witnesses' descriptions at the crime scene. One of the four sketches to the right is the way the suspect looked after changing his/her appearance. Assume that NO surgery has been conducted on the suspect. Select the face to the right that is most likely to be that of the suspect on the left:

a. Mug shot A b. Mug shot B

c. Mug shot C d. Mug shot D

...

Police Practice Test 1

Page 2 of 13

3. Answer this question on the basis of the following sketches. The first face that appears on the left is a sketch of an alleged criminal, based on witnesses' descriptions at the crime scene. One of the four sketches to the right is the way the suspect looked after changing his/her appearance. Assume that NO surgery has been conducted on the suspect. Select the face that is most likely to be the suspect.

Ans A a. Mug shot A b. Mug shot B

c. Mug shot C d. Mug shot D

4. Answer this question on the basis of the following sketches. The first vehicle, which appears on the left, is a sketch of a vehicle used in a recent crime, based on witnesses' descriptions at the crime scene. One of the four sketches to the right is the way the vehicle looked after its appearance was changed. Assume no bodywork was performed when changing the vehicle's appearance. Select the vehicle that is most likely to be the vehicle used in the crime.

a. Truck A b. Truck B

c. Truck C d. Truck D

Use the MAP to answer the following questions:

Police Practice Test 1

Page 3 of 13

5. You are at the Police Station preparing to go out on patrol when you are dispatched to respond to a vandalism incident in progress at the auto school on High Street (between Dudley Boulevard and Jefferson Avenue). The most direct route to the auto school, from the Police Station, without breaking any traffic laws, is as follows: a. Travel north on Spruce, west on Marquette, north on Dudley, and east on High to the auto school. b. Travel north on Spruce, west on Garfield, north on Dudley, east on Marble, north on Jefferson, and west on High to the auto school. c. Travel north on Spruce, west on Garfield, north on Dudley, and east on High to the auto school. d. Travel north on Spruce, east on Marquette, north on Jefferson, and west on High to the

Police Practice Test 1

Page 4 of 13

auto school.
6. You are located at the intersection of Oak Street and Jefferson Avenue and are out on patrol in your district. You turn north and travel for 1 block, then turn west and travel for 4 blocks, then turn north and travel for 2 blocks, then turn east and travel for 5 blocks, and finally turn south and travel for 1 block before stopping. You have stopped at the intersection of:

a. Jefferson and Tucker. b. Jefferson and Stover.

c. Broad and Tucker. d. Ridge and Broad.

7. A police officer is talking to a local citizens' group on the role police play in problems of domestic violence. The officer has explained that police officers are often called because a domestic conflict is disturbing neighbors. In addition, the officer has pointed out that many calls occur prior to a crime being committed. This is important since one of the objectives of police officers is to stop crime before it starts. The most effective way for the officer to accurately sum up the role of the police in domestic conflicts is as follows: a. "Since frequently no crime has been committed, the role of the police in domestic conflicts is fairly limited." b. "In resolving domestic conflicts, the officer's function is to restore order and prevent possible crimes from occurring." c. "The officer's main goal in responding to domestic crimes is to protect the innocent." d. "Many people involved in domestic conflicts call the police in order to have an objective authority help settle disputes." 8. Police Officer Turner is writing a report regarding the criminal homicide he investigated this morning. The report will include the following sentences: (These sentences are NOT listed in the correct order.) 1. I noticed a display case for handguns was open, but there were no guns in it. 2. We got a call reporting that the front door of a pawnshop was open, but the owner was not there. 3. We interviewed tenants of the apartments in the upstairs sections of the same building. 4. On arrival, we discovered the body of the shop owner, apparently dead from knife wounds to the chest. 5. I called an ambulance to pick up the victim. Officer Turner could communicate the above sentences most effectively if he presented them in the following order:

a. 4, 5, 2, 3, 1. b. 2, 1, 4, 5, 3.

c. 2, 4, 5, 1, 3. d. 2, 5, 1, 3, 4.

Police Practice Test 1

Page 5 of 13

Reading Comprehension Passage Use the information in the following passage to answer the next two questions. At 9:47 p.m., two patrol units were dispatched to a suburban home on the evening of June 12. The dispatcher stated that the owner had reported a robbery, which took place earlier that evening when he and his wife were not at home. The owner also stated that the suspects were in the home at the time of their arrival, but that they fled the scene in a dark blue sedan. Upon arrival at the home, Officers Baker and Reigner began to question Mr. Corneal, the owner of the home. In the meantime, Officers Lucas and Gentry conducted a preliminary investigation of the home to determine the activities of the suspects and the extent of damage. The questioning revealed that Mr. Corneal collected exotic guns and had accumulated an extensive collection. In addition, Mrs. Corneal had several sets of antique jewelry. When questioned as to individuals who knew about the collections, Mr. Corneal stated that only friends of the family and members of the rifle club were aware of them. He was further questioned to determine whether any unusual events had occurred recently that might be related to this incident. Mr. Corneal stated that they had extensive remodeling done to the interior of the home during the past month and a half by several different companies. Officer Reigner asked if they had reason to suspect any of the workers. Mr. Corneal stated that several of the workers had shown an interest in seeing and discussing his collection of guns, but that it would be very difficult to determine which individuals, if any, may have been involved. Officer Reigner requested the names of the companies and the services they performed. After a few minutes, Officers Lucas and Gentry returned from their search of the home. Since the only other areas that were disturbed were ones that typically contain money (e.g., dresser top and dresser drawers), they determined that the robbers concentrated their efforts on the two gun collections. They concluded that the robbers must have been aware of the gun collections before entering the home. 9. According to the above passage, when the Corneals arrived home on the evening of June 12, they discovered that their home had been:

a. b. c. d.

ransacked, but nothing was stolen. entered, and the television, stereo, and computer were stolen. entered, and the television, stereo, and computer were stolen. robbed, and five antique guns and several pieces of exotic jewelry were the items that were taken.

10. According to the robbery and subsequent investigation described above, the ONLY statement that accurately reflects the information gathered is:

a. Officer Reigner requested the names of the companies doing the remodeling. b. Mrs. Corneal seemed to be the one to answer all of the police officer's questions.

Police Practice Test 1

Page 6 of 13

c. Officer Baker helped with the search of the home. d. The robbers were reported to have fled the scene in a dark blue van.
11. Police officers should follow the guidelines presented below when dealing with individuals suspected of driving while under the influence of alcohol: 1. A SEPARATE citation shall be issued for any traffic offense that originally brought the offender to the attention of the officers (e.g., reckless operation of motor vehicle). 2. If the driver refuses to submit to a blood alcohol test OR submits to such tests and scores above the limit, the arresting officer should order the driver to surrender his/her operator's license and issue a citation for driving under the influence. 3. If the violator scores below the legal limit on the blood alcohol test, the operator's license SHALL NOT be confiscated. The results of field tests (e.g., walking heel to toe on a straight line) have no influence on this guideline. 4. Even if the violator scores below the legal limit, the violator can still be charged with driving under the influence if the officer can justify the charge through the use of field tests. Officer Kelley notices a car swerving in and out of its lane. Officer Kelley stops the car and asks the driver to submit to a test for driving under the influence of alcohol. The driver submits to the test and scores below the legal limit. However, based on several field tests, Officer Kelley still charges the driver with driving under the influence. Officer Kelley writes a citation for reckless operation (i.e., swerving from lane to lane), a citation for driving under the influence, and a citation for failing the field tests. Officer Kelley then confiscates the operator's driver's license and takes the driver to the station for booking. According to the guidelines presented above, Officer Kelley's actions were:

a. b. c. d.

appropriate because they were consistent with the guidelines provided. problematic because the driver was forced to surrender his license. problematic because Officer Kelley did not have a witness to the field tests. problematic because Officer Kelley gave the driver a citation for driving under the influence even though the driver passed the blood alcohol test.

12. Officer Johnson received the following four statements from witnesses who were in The First National Bank during a robbery: Witness #1 - "There were two of them. Both male, about 6'2" tall with medium builds. One of them had dark hair and was wearing jeans, a leather jacket, and sunglasses. The other had short blond hair and was wearing jeans and a red lightweight jacket. Only the one with the sunglasses spoke, and he had a southern accent." Witness #2 - "Both of the men were about the same size, maybe 6', about 180 lbs. One of them was wearing a leather jacket and the other one was wearing a lightweight red jacket. I could not see their faces because one had on a ski mask and the other had on dark sunglasses. The one with the sunglasses had dark hair."

Police Practice Test 1

Page 7 of 13

Witness #3 - "The two men were both about 6'1" and 180 lbs. One had on a leather jacket, the other a red windbreaker. Both men were wearing jeans and tennis shoes. One had dark hair and was wearing sunglasses. The other one was wearing some kind of stocking cap that was pulled down over his face." Witness #4 - "Both men had on jeans, tennis shoes and jackets. One jacket was leather, the other bright red. One of the men had dark hair and I could not see the other one's face because of a ski mask. They both looked a little over 6 feet tall. Only the one with the dark hair spoke." Based on the description presented above, Officer Johnson should recognize that there is a problem with the account provided by Witness #:

a. 1 b. 2

c. 3 d. 4

13. Use ONLY the definitions provided below to answer this question.

Aggravated Burglary - The crime of aggravated burglary is committed when an individual trespasses by force or deception into an occupied structure with intent to commit a theft or felony. Also, during the trespass, the individual must harm another or carry a deadly weapon. Burglary - The crime of burglary is committed when an individual uses force or deception in order to trespass in an occupied structure with intent to commit a felony or theft. Breaking And Entering - The crime of breaking and entering is committed when an individual, intent on committing a theft or felony, trespasses in an unoccupied structure by force or deception.
Consider the following situation: A janitor enters a busy office building, identifies himself as a janitor, and begins cleaning. While cleaning one office, he notices an appointment book on the floor. When he mops, he picks up the book and places it in his pocket while he finishes the job. Once completed, he leaves the building. It turns out that this janitor was really one of the competitors of the company occupying the building and that he planned to take any documents he could get. Given the definitions described above, this scenario is best categorized as:

a. aggravated burglary. b. burglary.

c. breaking and entering. d. none of the above.

14. The following dress code guidelines apply to Police Officers. Consider only the information presented here when answering the question. 1. The Antron Jacket will be worn for outside duty during the months of November through February and whenever the temperature is expected to drop below 55 degrees. 2. The Eisenhower Jacket shall be worn for outside duty during the months of

Police Practice Test 1

Page 8 of 13

March, April, May, September, and October only if the temperature is below 65 degrees. Officers with the rank of Captain or higher may wear the Double Breasted Blouse instead of the Eisenhower Jacket. 3. A navy blue long sleeve uniform shirt shall be the standard to be worn with the jackets. All individuals at the rank of Captain or above will substitute a white shirt. 4. The short sleeve shirt may be worn whenever the temperature is to rise above 70 degrees during the months of May through September. 5. Officers assigned to indoor duty may also substitute the short sleeve shirt for the long sleeve shirt. 6. Regulation trousers shall be worn. The black trouser braid or stripe shall be worn by individuals at the rank of Lieutenant and above. Typical Police Officer Ranks (from highest to lowest): Police Chief Deputy Chief Captain Lieutenant Sergeant Police Officer Consider the following circumstance: The temperature for the day is expected to stay right around 60 degrees. The date is September 15th. According to the regulations outlined above, Captain Cross would be dressed appropriately for outside duty if he wore: a. an Eisenhower Jacket, a white long sleeve shirt, and regulation trousers with a black stripe. b. a Double Breasted Blouse, a navy blue long sleeve shirt, and regulation trousers. c. an Antron Jacket, a white long sleeve shirt, and regulation trousers with a black braid. d. a Double Breasted Blouse, a white short sleeve shirt, and regulation trousers with a black braid. 15. Police Officer Thompson has noticed that in the district of the city that he patrols, all of the assaults occur in the eastern and northern sections, all of the auto thefts occur in the southern and western sections, and all of the traffic accidents occur in the western section. The majority of the auto thefts take place between 3 a.m. and 7 a.m. Most of the traffic accidents occur either between 7 a.m. and 9 a.m. or between 5 p.m. and 8 p.m. Most of the assaults occur between 7 p.m. and 9 p.m. or between 11 p.m. and 4 a.m. In addition, the traffic accidents almost always occur on Mondays and Fridays, the assaults take place on any day from Wednesday through Saturday, and auto thefts typically take place on weekday mornings. Police Officer Thompson would be most likely to reduce the number of assaults by patrolling the:

Police Practice Test 1

Page 9 of 13

a. b. c. d.

eastern section between 5 a.m. and 1 p.m. eastern section between 5 a.m. and 1 p.m. eastern section between 11 a.m. and 7 p.m. northern section between 5 p.m. and 1 a.m.

16. During the first half of the month of May, Police Officer Riggins received a series of reports from people who were mugged outside the South Side Shopping Plaza. The description of each suspect appears below: #1 (May 7) - male, Black, early 20s, 5'9", 170 lbs., black hair, tattoo on his upper arm, blue jeans and T-shirt. #2 (May 13) - male, Black, 20-26, 5'10", 175 lbs., black hair, tattoo on left hand, tank top, jeans, and sneakers. #3 (May 15) - male, Black, late teens, 5'11", 190 lbs., brown hair, snake tattoos on both upper arms, and red tank top. #4 (May 20) - male, Black, 17-21, 6'1", 210 lbs., brown hair, brown pants, a three-quarter length sleeve shirt, and no jewelry. #5 (May 21) - male, Black, 16-20, 6'7", 230 lbs., black hair, an earring in the left ear, blue jeans, and a tank top. Officer Riggins referred to this information when examining the incident described below. On June 3rd, a Black male was arrested near the Plaza. In the description that led to the arrest, a witness described the suspect as a Black male who looked about 23 years old. He was about 5'10" tall and weighed about 180 lbs. He had black hair, a tattoo of a cobra on his bicep, a muscle shirt, and blue jean cut-offs. Based on this description and the information from the five incidents that occurred in May, this suspect should also be considered a suspect for incident number:

a. 1 b. 2

c. 3 d. 4

17. The technique for dusting for fingerprints involves the following procedures: (These techniques are presented in the correct order). 1. Choose a powder color in contrast with the surface to be dusted. 2. Dip a brush into the powder and work the powder into the fibers of the brush. 3. Lift the brush out of the powder, checking to make sure there isn't too much powder on the brush. 4. Holding the brush lightly, shake a light dusting of powder onto the suspected area. 5. If a light pattern shows up, brush more powder into the pattern lightly with the flow of the ridges.

Police Practice Test 1

Page 10 of 13

6. Lightly clean up the pattern by brushing excess powder out of the voids between the ridges to define the print. 7. Photograph the impression and proceed to lift the impression with transparent tape. An officer has followed correct procedure when dusting for fingerprints and finally notices a pattern emerging. The next step the officer should do is: a. follow along the pattern with additional powder on the brush. b. shake a light dusting of powder over the entire area. c. shake the brush lightly to make sure there is no excess powder on it. d. try to define the print by brushing excess powder out of the voids. 18. When a prisoner is brought in to be booked, a basic search is conducted. The steps involved in a search are listed below: (These steps are NOT presented in the correct order.) 1. Lower both of your hands to the base of the prisoner's neck and proceed with the search by covering the chest, stomach, and back. 2. Have the inmate empty all pockets in trousers, shirt, coat, and jacket. 3. Be sure no other inmates are in the room when the search takes place. 4. From the waistline, proceed down the legs, using both hands on one leg, then the other. 5. Have the inmate stand facing a wall with his back to you, legs spread, and arms extended straight out. 6. From behind, using both hands, start at the forehead and run your fingers or a comb through the inmate's hair. The above procedures should be performed in the following order:

a. 2, 1, 4, 3, 5, 6. b. 3, 5, 2, 6, 4, 1.

c. 3, 2, 5, 6, 1, 4. d. 3, 5, 4, 2, 6, 1.

19. The Divisions of Police and Fire have standard procedures for handling bomb threats and bomb-related incidents including the following: 1. Trained police personnel direct operations at the scene; fire personnel stand by and typically assist in rescue operations. 2. While radios may be left on at the scene of an unexploded device to receive instructions from the Communications Office, no radio transmissions are to be made from the scene, since radio signals can detonate an explosive device. 3. The decision to evacuate a building is to be made by the management of the building unless an explosive device has been found; in such an instance, the police personnel in charge of the operation make the evacuation decision. 4. No public statements are to be made to the media by police or fire personnel.

Police Practice Test 1

Page 11 of 13

5. If one device detonates, there is always the possibility of a second or third device, so police and fire personnel should stay clear of the area until it has been determined (usually by police bomb squad personnel) to be safe. Given the above procedures, the most potentially dangerous mistake has been made in which of the following situations? a. Immediately after a second bomb exploded in a bank, Firefighter Thomas entered the bank to conduct rescue operations. b. While standing by at the scene of a bomb threat, Police Lieutenant Caffey provided information regarding the incident to a reporter. c. While standing by at the scene of a bomb threat, Fire Lieutenant Griffin received instructions from the Communications Office via his radio. d. After an unexploded device was discovered in an apartment building, Assistant Fire Chief Johnson ordered the apartment building manager to evacuate the building. 20. Police Officer Dunn interviews four witnesses to a murder which took place in a crowded bar and grill. Each of the witnesses observed the perpetrator of the shooting as he was leaving the bar and grill. They described him as follows: Witness #1: "He was a White male, about 25 to 30 years of age, with brown shoulder length hair. He was about 6'0" and weighed about 185 pounds. He wore blue jeans and a brown leather jacket. He had a small scar on his forehead." Witness #2: "He was a male, White or Hispanic, late twenties, 5'10", 190 pounds, with long brown hair. He was stocky and wore dark pants and a brownish jacket." Witness #3: "He was a light skinned male, possibly White or Hispanic, in his mid twenties, about 6'0" and weighed about 180 pounds. He had a tattoo of a panther on his right forearm. He had moderately long hair and wore a brown shirt and dark pants." Witness #4: "He was a White male, around 25 years old, about 5'11" and weighed about 185 lbs. His hair was brown and over his ears, fairly long. He wore darkish clothes, I'm not sure of the color of his jacket or pants." Based on the above information, Officer Dunn should recognize that there is a problem with the description given by Witness #:

a. 1 b. 2

c. 3 d. 4

21. When a police officer arrived at the scene of a disturbance in an abandoned warehouse, she noticed a van leaving the site. Since her investigation revealed that the warehouse had been vandalized, the officer felt she should include her observation of the van in her report. The most effective way for the police officer to report this fact is by saying:

Police Practice Test 1

Page 12 of 13

a. b. c. d.

"When I arrived at the warehouse, I saw a van driving away from the site." "A van which should not have been there was at the warehouse." "The vandals escaped in a van." "I do not know if it's important but when I got there, I saw a vehicle leaving the scene."

22. Police Officer Wilkins is preparing a report after leaving the scene of an accident. The report will include the following five sentences: (These sentences are NOT listed in the correct order.) 1. The Dodge struck the right rear fender of Mrs. Smith's Ford, and continued on its way. 2. Mrs. Smith stated she was making a left turn from 40th St. onto Third Avenue. 3. As the car passed, Mrs. Smith noticed the dangling rear license plate #412AEJ. 4. Mrs. Smith complained to police of back pains and was taken by ambulance to Bellevue Hospital. 5. An old green Dodge traveling up Third Avenue went through the red light at 40th St. and Third Avenue. The most logical order for the above sentences to appear is as follows:

a. 5, 3, 1, 2, 4. b. 1, 3, 2, 5, 4.

c. 4, 5, 1, 2, 3. d. 2, 5, 1, 3, 4.

23. Police Officer Crawford received a series of reports from several people who were mugged in the early evening as they were exiting from the Spruce Street subway station. The description of each suspect is as follows: Report No. 1 (November 16): Male, White, early 30s, 5'10", 180 pounds, dark hair, mustache, one gold earring, blue jeans, black jacket, running shoes. Report No. 2 (November 20): Male, White, 25-30, 5'6", 120 pounds, dark hair, dark glasses, one gold earring, blue jeans, green sweat shirt, running shoes. Report No. 3 (November 21): Male, White, 40-45, 5'10", 130-140 pounds, dark hair, mustache, one gold earring, blue jeans, black jacket, running shoes. n November 23rd, another person was mugged by a male who was loitering near the subway station exit. The clerk selling tokens witnessed the mugging, called 911, and the male was apprehended two blocks away. The description of the suspect is as follows: Report No. 4 (November 23): Male, White, 25-30, 5'10", 175 pounds, dark hair, mustache, blue jeans, black jacket, green ski cap, boots. Based on the description of the suspects in the first three reports, the suspect in Report No. 4 should also be considered a suspect in:

a. Report No. 1, but not in Report Nos. 2 or 3. b. Report Nos. 1 and 2, but not in Report No. 3. c. Report Nos. 2 and 3, but not in Report No. 1.

Police Practice Test 1

Page 13 of 13

d. Report Nos. 1, 2, and 3.
24. Add: 327+512

a. 475 b. 510

c. 545 d. 839

25. An arrested man’s bail was set at $3500. He had to put up 10% of that amount in cash in order to be released. If the man had $215 in his wallet, how much did he still need?

a. $35 b. $125

c. $135 d. $145

Police Practice Test 2

Page 1 of 24

Police Practice Test 2
Multiple Choice

Identify the letter of the choice that best completes the statement or answers the question.
You have 2 hours and 20 minutes to complete the test.

MAP QUESTIONS

Use the below MAP to answer the following questions. NORTH ^

1. You are a police officer assigned District 10, located at Means Ave and Elm St. You

Police Practice Test 2

Page 2 of 24

currently are at the district finishing roll call. You are dispatched to a Traffic accident on N. Oneal Ave between W. 2nd St and W. 1St. The most direct route to take without violating any traffic laws is; a. Drive West on Means Ave to N. c. Drive West on Means Ave to N. Jessup Ave, North on N. Jessup to 1 Jessup Ave, South on N. Jessup to 1 St to W. Oneal And St, East on 1 St, East on 1St to W. Oneal And North On W. Oneal Ave to the South On W. Oneal Ave to the Accident. Accident. b. Drive West on Means Ave to N. d. Drive West on Means Ave to N. Jessup Ave, South on N. Jessup to 1 Jessup Ave, South on N. Jessup to 1 St to W. Oneal and St , West on 1 St , West on 1St to W. Oneal And South On W. Oneal Ave to the South On W. Oneal Ave to the Accident. Accident.
2. You’re at Elm St and Clark St. You travel South from that location for one block, you turn E. and travel 2 blocks, you turn S. and travel 2 blocks, turn W. and travel 2 blocks before stopping. When you stop what intersection are you at? a. Clark & Elm c. Means & Elm b. Oneal & 1 St d. 2nd & Means

Answer the following questions based on the following accident scene photo. You' have three minutes to examine the photo. The questions will test your ability to recall ll facts and events occurring in the photo.

Accident Scene 1:

You' have three minutes to examine the above photo. The questions will test your ability to ll recall facts and events occurring in the photo.

Police Practice Test 2

Page 3 of 24

Don’t scroll past this point until the three minutes are up.

3. How many firemen are in the photo? a. 3 b. 4 4. The car involved in the accident hit a...

c. 5 d. 6

a. truck b. fire hydrant
5. What were the firemen doing?

c. tree d. pole

a. All Five were standing by the vehicle c. Four were standing by the vehicle involved in the accident. involved in the accident and one holding the hose. b. Four were standing by the vehicle d. Three were standing by the vehicle involved in the accident and one was involved in the accident and two were kneeling on the ground several kneeling on the ground several feet away. feet away. 6. How many Traffic cones are in the photo? a. 2 c. 4 b. 3 d. 5

Answer the following questions based on the following accident scene photo. You' have three minutes to examine the photo. The questions will test your ability to recall ll facts and events occurring in the photo.

Accident Scene 2:

Police Practice Test 2

Page 4 of 24

You' have three minutes to examine the above photo. The questions will test your ability to ll recall facts and events occurring in the photo.

Don’t scroll past this point until the three minutes are up. 7. Where was the damage on the car? a. front bumper b. left side door 8. What color is the fire truck door? a. red and white b. blue

c. left front quarter panel d. right front quarter panel c. red d. blue and white

9. What are the letters on the fire truck? a. BL c. FD b. XA d. LA 10. What were the firemen doing? a. Both were looking at the car c. One was looking at the car and the

Police Practice Test 2

Page 5 of 24

b. Both were facing the police officer

other one was facing the police officer d. One was looking at the car and the other one was standing next to the fire truck

11. Officer Riley interviews four witnesses to an armed robbery of a gas station. Each witness observed the suspect leaving the scene of the crime. They described the suspect as follows: Witness 1 He was a white male, 20 to 30 years of age with brown curly hair. About 6' “, 190 lbs. He 1 wore blue jeans and a black leather jacket. Witness 2 He was white or Hispanic, in his late 20' 5' ,195 lbs. with curly hair. Who was wearing s, 11" dark pants and a dark colored jacket. Witness 3 He was a thin white male, in his late 20' about 6 “, 185 lbs. with a tattoo of a cross on his s right bicep. He had brown curly hair. Witness 4 He was a light skinned Hispanic male, with dark curly hair, 5' to 6' about 200 10" 1" pounds. Wearing a black or brown jacket and blue jeans. Officer Riley should recognize the problem with which witnesses description?

a. 1 b. 2

c. 3 d. 4

12. Situation: When arriving at a scene, where a person needs medical assistance, a police officer should respond in the following order: 1. Render reasonable aid to the injured or sick person. 2. If needed call for Medical Assistance including an ambulance or a doctor. 3. Notify the dispatcher if the person is wearing a medic-alert emblem, indicating the person suffers from a serious medical problem such as heart disease, diabetes etc... 4. Make sure the ambulance gets directed to the scene when it arrives. 5. If the ambulance does not arrive in a reasonable time, depending on the seriousness of the medical problem, place a second call for the ambulance. 6. Record all the pertinent information including, time, names of persons notified, medicalert information etc.. . . Based on the given information answer the following question: Officer Hunt is advised by a citizen that an elderly woman just collapsed, just around the corner. Officer Hunt responds to the scene and immediately renders first-aid. At this time he

Police Practice Test 2

Page 6 of 24

notices the woman wearing a Medic-Alert emblem indicating heart disease and Officer Hunt calls for an ambulance. What should Officer Hunter do next? a. Make arrangements to direct the ambulance to the scene. b. Wait for the ambulance. c. Record the information in his daily log. d. Notify the dispatcher of the medical alert.

13. Use the following information to answer the below questions : Traffic Accidents are classified in the following categories: 1. One vehicle accidents with no injuries. 2. Two or more vehicle accidents with no injuries. 3. One or more vehicle accidents with injuries that are not life threatening. 4. One or more vehicle accidents with injuries that are life-threatening. Officer Allen responds to an accident at N. 20th Street and W. Logan. Upon arrival, Officer Allen is informed by a witness that a Ford traveling south on 20th St. struck a boy on a bicycle at the intersection of 20th and Logan. Officer Allen observes a boy standing by a bike. He appears to have minor injuries. Based on the given information what category accident is this?

a. 1 b. 2

c. 3 d. 4

14. What is it called when a police investigation begins to focus upon a specific suspect?

a. a proactive law enforcement strategy c. the beginning of formal proceedings b. the accusatory stage d. the high point of the criminal process
15. Which part of the crime detection steps typically follows an arrest?

a. booking b. accusation

c. search d. diversion

16. Which of the following is NOT part of a drug courier profile? a. arriving late at night from a source city b. acting nervous or too calm c. listening to certain kinds of music

d. walking rapidly or aimlessly

17. Which of the following would NOT justify a frisk? a. the officer is alone and backup has c. the stop is for a crime where not yet arrived weapons are commonly used b. the officer is scared because his/her d. the stop occurs in a high-crime area partner got shot last week 18. What is the purpose of Miranda rights?

Police Practice Test 2

Page 7 of 24

a. to reform the police so they change c. to eliminate unnecessary pressure, their procedures bordering on coercion b. to outlaw torture in any form, d. to neutralize distinct psychological physical or psychological disadvantages 19. Which of the following crimes has a better chance for suspect identification?

a. auto theft b. burglary

c. robbery d. vandalism

20. Find the sum of 205, 87, 4352, and 361 a. 5004 d. 6005 b. 5001 e. 6015 c. 5005 21. Find the sum of 3412, 1234, 2143 and 4322 a. 12,110 d. 11,111 b. 11, 222 e. 10,011 c. 12,111 22. What' 798 X 450? s a. 366.976 b. 231,908 c. 259, 100 23. Divide 66,456 by 72 a. 923 b. 823 c. 913 24. 3/4 from 9/10 = a. 3/20 b. 1 c. 2/7 25. Find the sum of 3/5, 2/3, and 1/4 a. 3/4 b. 27/20 c. 4/5

d. 359, 100 e. 459, 200

d. 843 e. 123

d. 3/5 e. 7/40

d. 91/60 e. 1 6/7

26. It takes one man one day to dig one hole. How many days does it take 5 men to dig 5 holes? a. 1 d. 4 b. 2 e. 5 c. 3 27. A shirt is discounted from $49.95 to $44.95. The percentage discount is approximately;

a. 15% b. 14% c. 12%

d. 11% e. 10%

Police Practice Test 2

Page 8 of 24

28. What percentage of 220 is 33?

a. 0.15% b. 1.5% c. 5%

d. 15% e. 1500%

What Constitutes An Arrest?
The central idea of an arrest is the taking or detaining of a person by word or action into custody so as to subject his/her liberty to the actual control and will of the person making the arrest. There must exist the intent to take into custody and a corresponding understanding by the person arrested that he is in custody, although, no formal declaration of the arrest is required. An arrest also implies not only custody, but also the aim of bringing the person into the judicial process to answer for an offense. Answer the following question(s) based on the above passage: Questions Regarding Arrest: 29. An officer, in uniform, firmly grips the person and says, "you’re going down to the station". This is:

a. not an arrest b. an arrest

c. a stop and frisk d. none of the above

30. Same as above, except the officer is in plain clothes and identifies himself/herself as an officer. This is;

a. an arrest b. not an arrest

c. a stop and frisk d. all of the above

31. An officer makes no physical contact and says, "Would you mind coming down to the station to answer some questions?" The person asks the officer if he/she must, and officer says, "You don' have to, but it would help get this thing cleared up. " This is: t a. an arrest because the officer asks the c. a stop and frisk because it is not an subject to go to the station arrest b. not an arrest because the officer told d. both A and C the subject he did not have to go to the station READING COMPREHENSION PASSAGE:

Probable Cause:
The probable cause required for a warrantless arrest according to the Wisconsin Supreme Court, is no less than would support a warrant. The definition of probable cause in most

Police Practice Test 2

Page 9 of 24

jurisdictions is: That quantum of evidence that would lead a reasonable police officer to believe that the defendant committed a crime. It' more than a hunch or suspicion, but less s than the evidence required to convict at trial. The facts on which Probable Cause is based must be legally obtained. The Fresh-needle puncture marks on an addict' arm discovered s by forcing, without further justification, the addict to remove his/her jacket will not support an arrest. The test for probable cause is objective, not subjective. The arresting officer must believe the defendant probably committed a crime. It must be sufficient evidence to convince hypothetically, that a reasonable officer would find Probable Cause:
If an officer arrives at the scene of the crime in progress by chance or in answer to a call for assistance, there is no question that a warrantless arrest based on probable cause may be made when the suspect is apprehended, after a chase. The same is not true when an officer answers a call and upon arrival sees a person fleeing from the scene of what appears to be a completed crime. At this point the officer certainly has the right to stop the person for a reasonable time and question him/her regarding the crime but it is not sufficient basis to arrest without verifying some facts. The difference here is a crime in progress as opposed to a completed crime.

Answer the following questions based on the above passage:
32. Questions Regarding Probable Cause: An officer receives a tip from an unknown source that X , a known heroin user, will be purchasing a quantity of heroin from Z, an unknown person, at a certain time and place (which would be in public view). The officer conceals himself near the place and, at the time predicted, observes X receive a package from a person in return for paper money. Regarding Probable Cause: a. There is no probable cause c. There is probable cause to arrest both persons after the transaction b. There is probable cause to arrest only d. none of the above X

33. A police bulletin reports a robbery and describes the suspect and the suspect’s car. An officer, hearing the description, observes a car traveling away from the scene, a very short time after the reported incident. The observed car matches the description, and as the officer gives chase, the suspect attempts to flee at a high rate of speed. When caught… a. the suspect may not be arrested c. the suspect may not be arrested based on the fact no probable cause based on the fact the officer did not has been established for committing observe the robbery. the robbery. b. the suspect may be arrested based on d. all of the above probable cause for committing the robbery. READING COMPREHENSION

Passage: Use Of Deadly Force

Police Practice Test 2

Page 10 of 24

An officer may use deadly force when he/she believes it is necessary to prevent death or great bodily harm to himself/herself or others. An accepted definition of deadly force applied by police officers is: The use of any means or instrumentality intended to or likely to cause death. Therefore, the justification for deadly force is the immediate threat of death or great bodily harm. While the application of deadly force is the action which is likely to cause death, there are limited number of instances where an officer may use deadly force in attempting to make an arrest or prevent escapes. The arrest must be for a serious crime in which the suspect used or threatened to use deadly force against some member of the public, such as murder, rape, assault with a deadly weapon etc. Also, the officer must reasonably believe there is NO OTHER WAY to make the arrest in order to retain custody of person once arrested. Even if the suspect is fleeing from officer, there has to have been a use or threat of use of deadly force by the suspect for the officer to use deadly force. In any situation the officer should not use deadly force unless he/she reasonably believes it is necessary and ONLY AS A LAST RESORT. Can an officer use deadly force when no crime is committed? Yes, an officer may use deadly force at any time even when no crime has been committed. For example: whenever the officer believes it necessary to prevent imminent death or great bodily harm to himself/herself or another person(s) as seen as being in danger. Use Of Deadly Force In Minor Crime Situations: In all other cases, the officer may not use deadly force, even if this restriction makes it impossible to arrest the person. For example, a fleet footed suspect out running the officer and successfully escaping. It makes no difference whether the person flees or resist arrest if the resistance is not with deadly force. However, even in minor crime situations the officer has a duty to attempt to make the arrest and if the person resists using deadly force, the officer may use deadly force in self-defense. EVEN IN SELF DEFENSE OF DEADLY FORCE however, the officer must reasonably believe that force less than deadly force will not be effective. 34. Question- Regarding The Above Passage and The Use Of Deadly Force: A murder suspect, known to be armed and to have previously shot an officer is seen by Officer Mack, who calls on the suspect to stop and throw down his weapon. The suspect flees on foot from the officer and is escaping. The Officer: a. Cannot shoot the suspect because no c. Can shoot the suspect because the suspect is known to be armed and is shots were fired at him (Officer Mack) fleeing b. Cannot shoot the suspect because he d. Can shoot the suspect because the (Officer Mack ) did not observe the officer knows the suspect is wanted suspect commit the murder for murder, is known to be armed and is escaping.

Police Practice Test 2

Page 11 of 24

35. A robbery suspect has shot at the pursuing officer and then runs away. The officer: a. may shoot the suspect b. may not shoot the suspect because the suspect stopped shooting c. may not shoot the suspect because he knows where the suspect lives and can arrest him later d. may shoot the suspect only if the suspect turns and shoots at him again

36. A suspect, wanted for a traffic offense, resists arrest, lunging at the officer with a knife. Regarding this statement which of the following is true: a. The officer cannot use deadly force c. because the suspect is not wanted for a serious crime. b. The officer can use to deadly force d. because the suspect is resisting arrest. The officer can use to deadly force because the suspect is resisting arrest. The officer can use deadly force because even though the suspect is wanted for a minor crime, the suspect is resisting using deadly force against the officer.

37. Which word means the OPPOSITE of JUDICIOUS?

a. legal b. imprudent

c. lawful d. criminal

38. Which word means the OPPOSITE of PREJUDICED?

a. bias b. impartial

c. reason d. sorry

39. Which word means the OPPOSITE of CONCRETE?

a. difficult b. abstract

c. simple d. false

40. Which word means the SAME as EXPOSE ?

a. reveal b. cover

c. favor d. pretend

41. Which word means the SAME as INFLUENCE?

a. cause b. affect

c. probe d. deem

Police Practice Test 2

Page 12 of 24

Problem Analyses
It is suggested you use scratch paper to help answer these questions. Baseball Situation Questions: J and B minor league baseball team has four pitchers, Mays, Craig, Henry and Manthy. Each of the four is best known for throwing one type of pitch: fastball, curveball, slider and screwball. Each of the four has his own style of delivery in pitching; overhand, threequarters, side arm on or underhand. * Henry is best known for the slider * Neither Craig nor Manthy uses a three-quarters style of delivery. *The pitcher who uses the underhand delivery is best known for throwing a fastball. *Manthy is best known for throwing the screwball. *Mays uses the overhand delivery.

Answer the below questions based on the above information:
42. Which of the following matches a pitcher with his best known pitch and his delivery style? a. Mays (curveball-three-quarters) d. Manthy-(curve ball-underhand) b. Henry (slider-side arm) e. Mays-(screw ball-sidearm) c. Craig-(fast ball-underhand) 43. During a game, if the starting pitcher is ineffective he will be replaced by another pitcher. All the following are possible pitching changes EXCEPT: a. The curveball picture being replaced d. by the pitcher who uses the overhand delivery. b. The screw ball pitcher being replaced e. by Henry. c. Mays being replaced by the fastball pitcher. The slider pitcher being replaced by the picture who uses the side arm delivery. Craig being replaced by the curve ball pitcher.

44. In a four games series, the manager of the team decides to pitch the fastball pitcher first, the pitcher who uses the three-quarters delivery second, curve ball pitcher third and the pitcher who uses the side arm delivery fourth. In which order will pitchers appear? a. Manthy, Craig, Mays, Henry b. Craig, Henry, Mays, Manthy c. Mays, Craig, Henry, Manthy PROBLEM ANALYSES Situation:

d. Craig, Mays Manthy, Henry e. Mays, Henry, Manthy, Craig

Ivan, Jack, Ken, Larry, Mike, Nancy, and Sue

Police Practice Test 2

Page 13 of 24

Seven police officers, Ivan, Jack, Ken, Larry, Mike, Nancy, and Sue, apply for training positions at the police academy. There are seven training positions available, in two different fields, firearms and search and seizure. Each field may take either three or four officers. The police academy has to consider the following: * Ivan and Jack may not work in the same training fields. * Nancy and Larry must work in the same training field. * If Jack and Ken work in the same training field, there must be four in that field. * Mike must work in search and seizure. * Firearms may have only four officers IF Jack is one of them. 45. Which of the following is a possible placement of the officers? FIREARMS Jack, Ken, Ivan, Sue Jack, Nancy, Larry Ivan, Ken , Larry Jack, Ken, Sue Nancy Ivan, Ken, Larry, Nancy SEARCH & SEIZURE Larry, Mike, Nancy Ivan, Ken, Mike, Sue Jack, Mike, Nancy, Sue Ivan, Larry, Mike,

a. b. c. d. e.

Jack, Mike, Sue

46. If Jack works in firearms then which of the following statements are true? a. Ken works in firearms b. Sue works in firearms c. Nancy may not work in search and seizure d. Larry and Sue work in the same training field e. Sue and Ivan work in the same training field

47. If Nancy is one of the four officers placed in search and seizures, which of the following statements could be True?

a. Ivan works in search and seizure d. Ken works search and seizures b. Larry and Sue work in the same field e. Ken and Sue both work in firearms c. Jack and Sue both work in firearms
READING COMPREHENSION PASSAGE: While working the 8:00 a.m. to 4:00 p.m. shift on January 14, 1999, Police Officers Marks and Smith received a radio call at 1:45 p.m. to investigate a report of a man with a gun in front of 103 Lexington Avenue. Mary Parks had called 911 from home at 1:43 p.m. and explained that two days ago while on the way home from work, she had been threatened by a man with a gun in front of her home at 113 Lowell Street. She told the police operator that the same man now was standing in front Harry' Lounge at 103 Lexington Avenue. She s described him as being 30 to 40 years of age, 5 ' , 165 lbs, wearing a gray coat, a gray 6" baseball cap and eye glasses. The officers responded to the location and observed a male

Police Practice Test 2

Page 14 of 24

fitting the description given by Mrs. Parks. The officers approached the suspect and while searching his right front waistband, Officer Marks found a 38 caliber revolver, that was licensed and registered under the name of Joseph Jackson. Mrs. Parks was brought to the scene and identified the suspect as the person who had threatened her. Officer Smith then place the man, identified as Joseph Jackson under arrest.

48. On what day did the suspect threaten Mrs. Parks?

a. January 10th b. January 12th

c. January 14th d. January 16th

49. Officer Marks recovered the gun from the suspect’s:

a. left front waistband b. Right rear waistband

c. Left rear waistband d. Right front waistband

50. Mrs. Parks stated that the suspect had threatened her in front of ...

a. Harry's Lounge. b. her home.

c. a bar. d. the police station.

51. Joseph Jackson was arrested because he...

a. possessed an unregistered gun. b. pointed a gun at the bartender.

c. was wanted for robbery. d. was identified as the suspect who threatened the victim.

52. Officer Johnson arrested an elderly man for loitering. Officer Johnson doesn' handcuff the t elderly man before he places him into the paddy wagon. This is a... a. good policy, because in general, elderly people should not be handcuffed. b. good policy because under the circumstances Officer Johnson has full control of the situation. c. bad policy, because Officer Johnson is subject to a greater potential risk of being assaulted. d. bad policy, because Officer Johnson would appear too soft or lacking control in the way he handled the incident.

53. Officer Jackson comes across a man who is unconscious, in an alley. Further investigation reveals that the subject has an empty syringe in his hand. Officer Jackson attempts to wake the man but is unable to do so. What should Officer Jackson do next? a. Ignore the situation and let the man c. Take the man into custody and wake-up on his own accord. transport him to nearest the detoxification treatment center. b. Arrest the man for possession of d. Call for an ambulance immediately illegal drug paraphernalia. to transport the subject to medical facility.

Police Practice Test 2

Page 15 of 24

54. As an officer on routine patrol you observe what appears to be a drug deal going down. You observe two individuals in a shopping mall exchange drugs for money. Both suspects observe you and start to run away in opposite directions. What should you do? a. Give a description of the suspects on c. the police radio and chase the suspect who has the money. b. Fire a warning shot and order the d. suspects to halt. Write down a complete description of the suspects, and attempt to seek a warrant for their arrest Give a description on the police radio of the suspects involved and chase the suspect with drugs.

55. While you' on duty a young female approaches you and tells you that a street vendor has re ripped her off for $45. She demands that he be arrested immediately. What should you do? a. Approach the vendor and demand that he returns the $45 to female. c. Conduct further investigation by talking to the street vendor in question and then decide what action should be taken. b. Explain to the woman there's nothing d. Place the street vendor under arrest immediately. you can do because this is a civil matter and the police only handle criminal matters. 56. You and your partner arrest a suspect for domestic abuse. You feel your partner failed in several regards to adequately assist you in this arrest. You felt his lack of action placed you in unnecessary danger. How should you handle the situation? a. Report your partners lack of action to c. File a written request for a different a supervisor. partner. b. Confront the officer directly and d. None of the above discuss the problem. 57. An officer conveys two prisoners to the station house to be booked. After dropping them off, he finds a $20 dollar bill in the back seat of the patrol car. What should the officer do ? a. Ask both prisoners if they lost the c. Give the money to your immediate money. supervisor and request that it be donated to the Police Athletic League b. Keep the money. d. File a formal written report indicating how the money was found and place the money on inventory. 58. Officer Roberts observers two men fighting. One of the men stabs the other in the chest with a knife. The victim falls to the ground and he is bleeding profusely. Under the circumstances, what should Officer Roberts do?

Police Practice Test 2

Page 16 of 24

a. Pursue the suspect and attempt to c. Attempt to slow bleeding and call arrest him. for an ambulance . b. Radio for an ambulance and attempt d. None of the above to locate witnesses. 59. While on routine patrol, Officer Reynolds observers a downed power line in the roadway. What should Officer Reynolds do regarding this situation? a. Move the wire to the side of the road c. Don't touch the wire, place a traffic where it will not create a hazard to cone by it to divert traffic and traffic. contact the Electric Utility Service b. Don't touch the wire, secure the area d. None of the above and divert the traffic away from the power line and contact the Electric Utility Service 60. Which of the following crimes would be least likely affected by increased police presence?

a. burglary b. criminal trespass

c. murder d. robbery

61. While on patrol Officer Matthews discovers a fire on the first floor of a two-story apartment building. What should Officer Matthews first response to the fire be? a. Do whatever he can to extinguish the c. Immediately contact the fire fire. department and use the PA system in patrol car to alert the occupants of the fire. b. Enter the building and attempt to d. Call the Fire Department and check alert the occupants of the fire and for witnesses. help evacuate the building 62. 99 + 88+ 77+ 66+ 55 =

a. 384 b. 485

c. 385 d. 345

63. Officers must be accurate yet brief when broadcasting on the police radio. What is the best reason for the above statement? a. It makes officers sound more professional b. It saves the department money on radio air time expense c. Tying up the airwaves with long messages on the police radio may prevent emergency transmissions from getting through. d. Shorter messages are easier to understand.

64. Officer Smith is sent to a silent burglary alarm at a warehouse at 3 am. Officer Smith should take the following steps when responding to the alarm, EXCEPT: a. Make a cautious approach to the c. Approach the building with your red

Police Practice Test 2

Page 17 of 24

lights and sirens activated, so you can a arrive at the building as soon as possible . b. Attempt to contain the building upon d. Do not allow the owner of the your arrival until further help arrives building or any employees to enter the building until the premise has been searched. 65. Officer Jones sees a motorist commit a traffic violation at a construction site where street signs have recently been changed. What action should Jones take? a. Write down license plate and if he c. Stop the offender and issue a sees the motorist commit this warning. violation again issue him/her a citation. b. Ignore the incident because of the d. Stop the offender and issue a recent change. citation. 66. While attending a high school football game off-duty, Officer Smith observers some kind of incendiary device connected to the timer beneath the bleachers. What is Officer Smith' best s course of action at this time? a. Attempt to disarm the device. c. Notify the stadium officials and have them stop the game and evacuate the public, then call the bomb squad. b. Attempt to evacuate the area by d. Call the bomb squad and let them telling everybody there's a bomb in handle the entire matter. the stands. 67. Officers are told not to appear in places within their patrol areas at regular intervals. What' the best reason for this policy? s a. Staggering inspection times gives the c. Staggering patrols makes it more public the impression that there are difficult for the criminal element to more officers then there really are. predict when and where the police will be. b. Regular routines inspire d. None of the above. complacency. 68. An officer should know how to take a good defensive position when conducting field interviews to help ward off unexpected attacks. All the following are considered good defensive posture, EXCEPT:

building, using stealth so as not to alert the suspects.

a. feet moderately spread and staggered c. knees someone bent b. legs and back kept straight d. lowering of the buttocks
69. Officer Reynolds arrives at the scene of a disturbance in an abandoned warehouse. She notices a van leaving the site. Her investigation reveals the warehouse had been vandalized. Officer Reynolds will file a report regarding this incident. What is the most effective way Officer Reynolds should indicate in her report her observations of the van and leaving the scene? a. A van which should not have been c. The suspects escaped in a van. there, was at the warehouse.

Police Practice Test 2

Page 18 of 24

b. Upon my arrival at the warehouse, I d. All the above observed a van driving away from the site. 70. Officer Miller has completed an investigation of a traffic accident. Officer Miller is about to file a report regarding the accident. The following contains sentences that will be on Officer Miller' report. s What is the correct order the sentences should be in? 1.The Ford struck the right rear fender of Mr. Ricks Dodge and continued on its way. 2. Mr. Ricks stated that he was making a left turn from 35th Street onto 2nd Street. 3. Mr. Ricks stated he observed the license plate of Ford and it was XZY163. 4. Mr. Ricks complained of back pain as a result of the accident and was conveyed by ambulance to St. Luke' Hospital. s 5. A Ford with license plate number XZY163 was observed traveling north on 2 Street when it went through a red light on 35 St.

a. 1,3,2,5,4 b. 2,5,1,3,4

c. 4,5,1,2,3 d. 5,3,1,2,4

PASSAGE: Travel Agents
Travel agents assist prospective travelers in a number of ways. They provide information and suggestions regarding travel plans, make transportation and hotel reservations, and consult with clients regarding other details associated with traveling, such as passport regulations, foreign currency, and hotel rates. While an agent who works in a small office usually performs all of these duties, some, who work in large agencies with many employees specialize in a single function. Because travel agents make many detailed reservations, they must be capable of the utmost accuracy. Some experienced and highly skilled agents develop package tours. This involves planning an itinerary, finding a tour leader, making travel and hotel reservations, and publicizing the tour. Sometimes travel agents serve as tour guides. This gives them a chance to acquire personal travel experience and to become thoroughly acquainted with people who may be potential customers for future trips. 71. READING COMPREHENSION Travel agents help their clients to:

a. develop package tours b. make travel plans

c. meet other travelers d. serve as tour guides

72. According to the passage, travel agents must be especially capable of accuracy when they:

a. convert foreign currency b. serve as tour guides

c. arrange publicity d. make reservations

73. In general, the passage describes a travel agent' s:

Police Practice Test 2

Page 19 of 24

a. work environment

b. education and training

c. method of making detailed reservations d. duties and experiences

74. Situation: Police officers are to call for a supervisor in the following situations; Whenever force is necessary in making an arrest. Whenever a shooting occurs. All assignments where the suspect is armed. All traffic accidents involving life-threatening injuries. All accidents involving Department vehicles. ------------------------------------------------Based on the above information; A supervisor should be called to the following assignments, EXCEPT: a. armed robbery in progress b. burglary in progress c. traffic accident involving a car and a bicycle where the bicyclist has head injuries d. arrest of a burglary suspect who resisted and was pepper sprayed

Problem Analyses
Classroom Questions

Situation: An assistant principal must assign classes K,L,M,N,O,P and Q to the 7 classrooms lining the North corridor of his high-school. The rooms are numbered 1 through 7 from east to west. There is a broom closet between rooms 4 and 5. Only 1 and 7 have taping facilities. L and M may not be adjacent rooms. N and O must be adjacent rooms. L must be in room 3 Language classes must be in rooms with taping facilities.
Start making a list and a diagram. 1 2 ____ _____ (taping ) 3 ____ L 4 ____ BC ____ 5 ____ 6 ____ 7 _____ (taping)

LIST GIVEN INFORMATION --------------------------------------------------------L not adjacent M N is adjacent to O L is in room 3 Language classes are in rooms with taping facilities. Rms 1 & 7 have taping facilities A broom closet between 4 and 5

Police Practice Test 2

Page 20 of 24

----------------------------------------------------------------In this exercise, you' need to list the known facts for every individual question. ll Every question presents a different set of circumstances. Note: Keep the following in mind when answering the questions: You' notice the questions ll don' supply you with concrete information. You' not reminded about some important t re factors given in the stem/situation. Such as: Where' the language classes? s 1 2 ____ _____ (taping ) (Lang) 3 ____ L 4 ____ BC ____ 5 6 ____ ____ 7 _____ (taping) ( Lang)

The challenge is keeping track of special conditions. IE: broom closet, language classes, taping rooms. ---------------------------------------------75. If K and M are in 2 rooms adjacent to the broom closet and if O is in room 2 , which the following must be true?

a. N is in Room 7 b. P and M are in are adjacent rooms c. Q and P are in adjacent rooms

d. N and P are in are adjacent rooms e. P is in a room with taping facilities

76. Situation: If M and N are both language classes and if O and L are in adjacent rooms, it must be true that M is in:

a. room 1 b. room 7 c. a room adjacent to Q's room

d. a room adjacent to O's room e. a room adjacent to K's room

77. If K and O are both in the language classes, and if M is not in a room adjacent to the broom closet, all of the following must be true; EXCEPT:

a. O is in room one b. N is in room 2 c. M is in Room 6

d. P and L are in adjacent rooms e. Q is in a room adjacent to the broom closet

78. Situation: Officer Jackson has issued a summons to a driver and has obtained following information: Place of occurrence: Corner of Foster Rd and Woodrow Avenue Time of occurrence: 7:10 p.m. Driver: William Grant Offense: Driving through a red light Age of driver: 42 Address of driver: 23 Richmond Avenue --------------------------------------------------

Police Practice Test 2

Page 21 of 24

Officer Jackson is making an entry into his memo book regarding the incident. Which one of the following expresses the above information most clearly and accurately? a. William Grant lives at 23 Richmond c. William Grant age 42, was issued a Avenue at 7:10 PM went through a summons on the corner of Foster red light. He was issued a summons Road and Woodrow Avenue for at the corner of Foster Rd and going through a red light. He lives Woodrow Avenue. The driver is 42 that 23 Richmond Avenue at 7:10 years of age. p.m. b. William Grant, age 42 who lives at d. A 42 year-old man who lives at 23 23 Richmond Avenue, was issued a Richmond Avenue, was issued a summons for going through a red summons at 7:10 p.m. William Grant went through a red light at the light at 7:10 p.m. at the corner of Foster Road and Woodrow Avenue. corner of Foster Rd and Woodrow avenue. 79. Situation: Officer Kent has completed an investigation regarding a report of an auto theft and obtained the following information: Date of occurrence: October 26, 1988 Place of occurrence: 51St and 8th Ave Time of occurrence: 3:30pm Crime: Auto theft Suspect: Michael Wadsworth The action taken: Suspect arrested ------------------Which of the following expresses the above information most clearly and accurately? a. Arrested on October 26, 1980 was a c. stolen auto at 51 street and 8th Avenue at 3:30 p.m. driven by Michael Wadsworth. b. For driving a stolen auto at 3:30 p.m. d. Michael Wadsworth was arrested at 51st Street and Eighth Avenue on October 26, 1988. On October 26, 1988 at 3:30 p.m. Michael Wadsworth was arrested at 51st Street and 8th Avenue for driving a stolen auto. Michael Wadsworth was arrested on October 26, 1988 at 3:30 p.m. for driving at 50 St and 8th Ave. The auto was stolen.

80. Which word means the OPPOSITE of REVERENCE?

a. loyalty b. sane

c. disrespect d. balance

81. Which word means the OPPOSITE of AUDIBLE?

a. optical b. planned

c. easy d. silent

82. Which word means the OPPOSITE of DETAIN?

a. hold b. release

c. repeat d. fail

Police Practice Test 2

Page 22 of 24

83. Which word means the OPPOSITE of CHRONIC?

a. infrequent b. fatal

c. forever d. seldom

84. Which word means the OPPOSITE of CERTAIN?

a. ambiguous b. almost

c. reveal d. predict

85. Which word means the OPPOSITE of MAR?

a. break b. confuse

c. repair d. incur

86. Which word means the OPPOSITE of LUCID?

a. unclear b. apathetic

c. ordinary d. bolster

87. Which word means the SAME as UNINTELLIGIBLE?

a. insane b. garbled

c. retain d. savor

88. Which word means the SAME as HIDEOUS?

a. hidden b. uncovered

c. grotesque d. famous

89. Which word means the SAME as COMPEL?

a. rely b. coerce

c. repent d. release

90. While conducting a burglary investigation, Officer Jones interviews several witnesses. All the witnesses observe a vehicle leaving the scene of the burglary. Which of the description offered by the witnesses should Officer Jones consider most likely to be correct? a. A brown GMC truck, License plate # c. G-1843 b. A black Ford truck, License plate d. #G-1854 A Brown Ford truck, License plate# G -1845 A Brown Ford truck, License plate # C-1845

READING COMPREHENSION
PASSAGE: Officer Harris, after studying the crime trend analysis data noticed a significant increase in the number of certain crimes in his patrol area over the past three months. The data indicated that most of the rapes took place on East 98 Street between Lott Avenue and

Police Practice Test 2

Page 23 of 24

Herk Place: assaults took place on Lott Avenue between Chester Avenue and East 98 Street: and the majority of the robberies occurred on Lott Avenue between 98th Street and Hughes PLace. The assaults took place between 1:00 a.m. and 3:00 a.m.. All the robberies happen between 1:00 a.m. and 6:00 a.m. and most of the rapes occur between 8:00 a.m. and 11:00 a.m.. The rapes usually occur on Mondays and Wednesdays, the robberies on Fridays and Saturdays and the assaults on Saturdays and Sundays.
91. Based on the above information answer the following question. Officer Harris would most effectively reduce the number of robberies by patrolling... a. Lott Avenue between E. 98th Street c. and Hughes Pl. on Fridays and Saturdays between 1:00 a.m. and 8:00 a.m. b. Lott Avenue between East 98th d. Street and Chester Avenue on Saturdays and Sundays between 1:00 a.m. and 6:00 a.m. East 98th Street between Lott Avenue and Herk Place on Saturdays and Sundays between 1:00 a.m. and 3 a.m. East 98 Street between Herk Place and Chester Avenue on Mondays and Wednesdays between 8:00 a.m. and 11:00 a.m.

92. Officer Harris has been informed by supervisors that he will be assigned to a patrol area each week that would allow him to concentrate on reducing the number of rapes. What would be the most appropriate patrol for Officer Harris to work? a. Tuesdays through Saturdays, 8:00 c. Wednesday through Saturday, noon p.m. to 4:00 p.m. to 8:00 p.m. b. Monday through Friday, 7:30 a m to d. Monday through Friday 3:00 p.m. to 3:30 p.m. 11:00 p.m. 93. In order to conceal her guilt, Linda told a blatant lie.

a. harmless b. subtle
94. 837+ 415 minus 1035 =

c. careless d. obvious

a. 332 b. 245 c. 213
95. What' 526 X 317 ? s

d. 217 e. 226

a. 154,298 b. 166,742 c. 166.244

d. 266, 742 e. 66, 742

96. Police officers are required to remove potentially dangerous property from prisoners prior to placing them into jail cells. Which the following should be considered a potentially dangerous property?

a. an empty wallet

c. cough drops

Police Practice Test 2

Page 24 of 24

b. shoelaces

d. a wedding ring

97. Answer the following question based on the following information: Situation: When a OFFICER makes an arrest, the officer should do following in the order given: 1. Advise the person he/she is being arrested. 2. Handcuff the prisoner' hands behind his back. s 3. Search the prisoner and the nearby area for weapons and evidence. 4. Advise the prisoner of their constitutional rights before questioning him/her. -----------------------------------Police Officer Silver arrests Larry Tims for robbing a grocery store . He informs Mr. Tims that he is under arrest. He then handcuffs Mr. Tims, placing the handcuffs behind Mr. Tim' back. While handcuffing Mr. Tims Officer Silver notices a s small handgun in Mr. Tims back pants pocket. The next step Officers Silver should take is to: a. Inform the prisoner of his rights b. Question the prisoner c. Return, with the prisoner to lead to the serve store d. Search the prisoner and the immediate area for weapons and evidence

98. Civilization started to move ahead more rapidly when people freed themselves of the shackles that restricted their search for the truth. The sentence above best supports the statement that the progress of civilization a. came as a result of people’s dislike for obstacles. b. did not begin until restrictions on learning were removed. 99. Irresolute means: c. has been aided by people’s efforts to find the truth. d. is based on continually increasing efforts.

a. wavering b. insubordinate

c. impudent d. unobservant

100. Jim is arrested in a criminal case. He refuses to answer any questions about the alleged crime, claiming that he is “taking the Fifth Amendment.” “Taking the Fifth Amendment” means that... a. Jim is pleading innocent to the charges against him. b. Jim has not had the opportunity to discuss the case with a lawyer. c. Jim does not have to answer questions that might incriminate him. d. Jim is too young to be arrested for this crime.

Police Practice Test 3

Page 1 of 19

Police Practice Test 3
Multiple Choice

Identify the letter of the choice that best completes the statement or answers the question.
You have hours You have 2 hours and 10 minutes to complete the test.

1. Police officers have the discretion to give warnings instead of citations at traffic stops. Which of the following situations would a warning be more appropriate to give than a citation? a. A man runs through a stop sign because there's no other cars. c. A man has a heart attack and runs through a stop sign causing a minor accident. b. A woman is late for work and is d. A woman unfamiliar with the area caught driving 10 mph over the speed makes a left turn disregarding a no limit. left turn sign. 2. A car was seen leaving the scene of a burglary. Witnesses observed the license plate. Which of the following is most likely the correct license plate number given by the witnesses?

a. TLC 519 b. BIU 559

c. BIU 519 d. BIC 519

3. A witness observes an Armed Robber run out the door of a gas station. Which part of the witness' description would be most useful in identifying the suspect? a. The suspect was wearing a green jacket. b. The suspect had on a blue baseball cap. c. The suspect had a large nose and brown eyes. d. The suspect had a 2" inch scar on his left cheek. e. The suspect walked with a limp.

4. Officer Miller is advised by his sergeant that a shopping Mall in her area had a rash of auto thefts reported in the last two weeks. Officer Miller indicates she will intensify patrol in the area. Regarding the auto thefts what situation observed in the Mall by Miller should be investigated?

file://C:\Documents%20and%20Settings\Don%20Cirillo\Desktop\POLICE2\POLICE%20...

Police Practice Test 2

Page 24 of 24

b. shoelaces

d. a wedding ring

97. Answer the following question based on the following information: Situation: When a OFFICER makes an arrest, the officer should do following in the order given: 1. Advise the person he/she is being arrested. 2. Handcuff the prisoner' hands behind his back. s 3. Search the prisoner and the nearby area for weapons and evidence. 4. Advise the prisoner of their constitutional rights before questioning him/her. -----------------------------------Police Officer Silver arrests Larry Tims for robbing a grocery store . He informs Mr. Tims that he is under arrest. He then handcuffs Mr. Tims, placing the handcuffs behind Mr. Tim' back. While handcuffing Mr. Tims Officer Silver notices a s small handgun in Mr. Tims back pants pocket. The next step Officers Silver should take is to: a. Inform the prisoner of his rights b. Question the prisoner c. Return, with the prisoner to lead to the serve store d. Search the prisoner and the immediate area for weapons and evidence

98. Civilization started to move ahead more rapidly when people freed themselves of the shackles that restricted their search for the truth. The sentence above best supports the statement that the progress of civilization a. came as a result of people’s dislike for obstacles. b. did not begin until restrictions on learning were removed. 99. Irresolute means: c. has been aided by people’s efforts to find the truth. d. is based on continually increasing efforts.

a. wavering b. insubordinate

c. impudent d. unobservant

100. Jim is arrested in a criminal case. He refuses to answer any questions about the alleged crime, claiming that he is “taking the Fifth Amendment.” “Taking the Fifth Amendment” means that... a. Jim is pleading innocent to the charges against him. b. Jim has not had the opportunity to discuss the case with a lawyer. c. Jim does not have to answer questions that might incriminate him. d. Jim is too young to be arrested for this crime.

Police Practice Test 3

Page 3 of 19

9. Officer Hendly is sent to a purse snatch. Officer Hendly interviews four witnesses who observed the purse snatch suspect. Which of the witness's statement is probably the most accurate? a. He was medium height, husky, in his c. He was short, husky, early 20s mid 30s b. He was short, husky, in his mid 30s d. He was short, medium built, mid 30s 10. Police sometimes have to exceed the speed limit in order to perform their duties effectively. Which of the following incidents would be appropriate for a police officer to go faster than the speed limit? a. responding to a burglary that d. attempting to stop a speeder occurred the day before b. responding to a hospital to interview e. both c and d a victim of an assault c. pursuing a vehicle wanted in an armed robbery 11. Officer Raymonds, while conducting an investigation of a convenient store robbery, interviews 4 witnesses who observed the getaway car. Which of the descriptions is probably the right one?

a. dark green with gray roof b. dark blue with white roof

c. black with gray roof d. dark green with tan roof

12. Burglar Alarms Officer Doran gets dispatched to a silent burglar alarm at Lincoln Elementary School located at 1st and Madison. When the call came, Officer Doran was finishing a traffic stop two blocks from the school. She responds to the call and parks the squad in a strategic location away from the school. What should Officer Doran do next? a. Check the building for signs of entry. c. Notify the dispatcher of her arrival and location. b. Call for back up. d. Call for a supervisor. 13. Officer Winston is sent to a burglar alarm at a residence at 204 S. 10 St. It's 4pm., traffic is heavy at the time and he is about 2 miles away from the scene. Officer Winston should respond to the alarm in the following manner: a. Proceed to the scene WITH c. Proceed to the scene WITH emergency lights and siren activated, emergency lights and siren activated to get to the scene quick as possible than turn them off far enough away from the scene as not to alert the suspect. b. Proceed to the scene WITHOUT d. None of the above. emergency lights and siren activated so as not to alert the suspects. 14. At 2:00 am Officer Harris is sent to a burglar alarm at a shoe store at 204 N 10th St. This is the third time in 4 hours the alarm has gone off. All false alarms. The owner is on his way to

Police Practice Test 3

Page 4 of 19

the alarm. Officer Harris responds to the alarm parking a safe distance away and notifies the dispatcher of her arrival. What should Officer Harris do next ? a. Wait for the owner, it's probably another false alarm b. Call for a supervisor c. Check the building for signs of entry

d. Call for a back up

15. Officer Johns and Malloy were sent to a Domestic Disturbance at 1202 N. Henry. While walking up to the residence they hear breaking glass and screaming. What should the officers do next? a. Conduct a forced entry. c. Advise the dispatcher of their observations. b. Knock loudly and identify themselves d. Listen at the door to see if they can as police. tell what's happening 16. Officers Timm and Davis enter a residence of a Domestic Violence call. Inside they observe women bleeding from the arm profusely. The suspect is on the scene. They notify the dispatcher of the situation and request an ambulance. What should the officers do next?

a. Perform first aid. c. Call for a supervisor. b. Take control of the suspect(s) and d. Interview the victim. check for and secure weapons within reach of all parties in the residence.
17. In the United States the law has imposed strict consequences on any abuse of search and seizure procedures. Therefore, evidence discovered as a result of any search that does not comply with the procedures and standards laid down by the Supreme Court and by other courts, interpreting the various amendments to the U.S. Constitution collectively known as the Bill of Rights, is not admitted in the trial, even though; a. It clearly establishes the guilt of the c. The suppression of the evidence may accused person, and even though the prevent the conviction of a person suppression of the evidence may who is indigent. prevent the conviction of a person who is innocent. b. It clearly establishes the guilt of the d. It clearly establishes the innocence accused person, and even though the of the accused person, and even suppression of the evidence may though the suppression of the prevent evidence may the conviction of a person who is prevent the conviction of a person plainly guilty. who is innocent. 18. Common Sense Factors To Consider When Handling Evidence: Officer Keen, while investigating a burglary at Washington High School, observes what appears to be evidence inside one of the classrooms. He finds a crowbar and a flashlight near a broken window. Officer Keen places the crow bar into a paper bag. What should Officer Keen do next? a. Place the bag with the crowbar into c. Put the flashlight into the bag with his squad. the crowbar.

Police Practice Test 3

Page 5 of 19

b. Seal the bag with evidence tape.

d. Write his name and ID number on the bag

19. Officer Charles is investigating a hit and run accident at 124th and Andrews Lane. He finds a piece of a tail lamp belonging to the hit and run vehicle. He also found an auto mirror belonging to the same hit and run vehicle, several feet from the tail lamp. Officer Charles takes photos of the evidence and than proceeds to collect it. What is the first thing Officer Charles should do? a. Place both pieces of evidence in the c. Place both items in a cardboard box same evidence container than seal and transport them to the property them. division. b. Place both pieces of evidence in d. Place the items in a cardboard box separate evidence containers and seal and mark them with his ID number. them. 20. Officer Peters has collected evidence at a shooting scene. She placed the evidence in separate containers and sealed the containers. Officer Peters should place the following information on the evidence tag, EXCEPT; a. Her name. d. The date and time the evidence was collected. e. Name of the first officer at the scene.

b. The case number. c. The date and time the evidence was collected.

21. Officer Krans arrests a suspect for shoplifting. While he is on route to the jail, he hears another squad broadcast a vehicle pursuit several miles away. What should officer Krans do? a. Proceed to the chase with redlights c. Proceed directly to the jail with the and siren activated. prisoner. b. Proceed to the chase using normal d. Remain at the location of the arrest traffic speed, obeying all traffic laws. in the event the pursuit comes that way. 22. Officer Thomas arrests a drug dealer at 17th and Howard. He places handcuffs on the suspect, placing the handcuffs behind the suspect's back. What should Officer Thomas do next? a. Check the area of the squad where thec. Search the prisoner suspect will be seated for weapons. b. Place the suspect into the squad and d. Advise the prisoner of his Miranda seatbelt the suspect. rights. 23. When a prisoner is arrested and conveyed the officer should do the following, EXCEPT: a. Handcuff the prisoner securely. b. Search the prisoner. c. Convey the prisoner directly to a lock up facility. d. Let the prisoner contact his attorney

Police Practice Test 3

Page 6 of 19

from the location of the arrest
24. Police Officer Thompson is unsure about what factors should be considered when deciding if non-deadly physical force is reasonable. Police Officer Thompson asks Field Operations Lieutenant Hernandez to clarify factors that should be considered in a possible situation. Field Operations Lieutenant Hernandez should inform Police Officer that when deciding whether non-deadly physical force is reasonable, one (1) of the factors that should be considered is the individual's:

a. race b. age c. religion

d. sexual preference e. social status

25. Officer Harris responds to a sexual assault at 2334 S. 10st. Upon arrival, Officer Harris observes the victim with her clothing torn, sitting on the rear porch at that address. The victim says to the officer she was just rapped and the suspect ran south on 10th Street. What should Officer Harris do first? a. Obtain and broadcast a description. b. Return to the squad and check the area for the suspect c. Administer first aid and call for ambulance if needed. d. Check the scene of the crime for evidence.

26. Officer Sanders is sent to investigate a sexual assault at the Gas Station at 35th and Vine. Upon arrival at the scene she is met by the victim who is uninjured and appears calm. The victim informs the officer that she knows the suspect's first name and gives the officer a detailed disruption. She tells the officer she met the suspect at a night club on 27th street. He followed her out of the night club and raped her. She drove to the gas station and called the police. She thinks the suspect returned to the night club. The next thing the officer should do is: a. Go to the night club to check for the c. Check the area for evidence. suspect. b. Broadcast the description and request d. Transport the victim to a medical that a squad check the night club for facility. the suspect. 27. A supervisors should be called to the following assignments, EXCEPT: a. armed robbery in progress b. burglary in progress c. traffic accident involving a car and a bicycle where the bicyclist has head injuries d. arrest of a burglary suspect who resisted and was pepper sprayed.

28. Officer Kent responds to the welfare of an elderly man living alone at 1983 S 10th. Upon arrival he finds the elderly man inside the residence deceased in a bedroom. The neighbors advise Officer Kent that the deceased Mr. Rose lived alone. Officer Kent should take the following steps to protect the prisoners property, EXCEPT:

Police Practice Test 3

Page 7 of 19

a. In the presence of a supervisor, d. Call for back up squads. search the body and the residence for valuables and identification. b. Count all money in front of the e. Deliver the property to the police property room and place the property supervisor. Remove all property except clothing. recovered on a property inventor report. c. Enter in Memo Book a list of all property removed. 29. Officer Johnson investigates a report of a man found dead of natural causes in his apartment at 1903 W. Brantly. No foul play is suspected. Upon arrival at the scene, Officer Johnson finds the deceased man and calls for a supervisor. What should Officer Johnson do next? a. Enter in Memo Book a list of all property removed. b. Count all money in front of the supervisor. c. Remove all property except clothing. d. In the presence of a supervisor, search the body and the residence for valuables and identification. e. Request another squad to search the apartment

30. Officer Doyne is sent to investigate a missing person at 231 E. Milton. Upon arrival at the scene, she gets a detailed physical and clothing description from the caller. The caller informs Officer Doyne that the missing has no physical disabilities. The next thing officer Doyne should do is... a. Obtain a photograph if available. b. See if the missing person has a wanted status c. Determine if the person has a mental disability. d. File a missing person report

31. Officer Stoker arrests a man for auto theft. He handcuffs the prisoner's hands securely behind his back. He notices a slight bulge in the suspects left sock. What should Officer Stoker do next? a. Search the suspect front pocket. c. Search the area within reach of the suspect's handcuffed hands. b. Search the suspect's socks. d. Search the suspect's cap and hair. 32. Officer Slater and Gains arrest a suspect for assault. The suspect attempts to run. Both officers chase the suspect and tackle him. What should the officer do next?

a. Search the suspect for weapons. b. Advise the suspect of his rights.

c. Put handcuffs on the suspect. d. Put the suspect in the squad

33. Officer Ford is inside a residence investigating a domestic dispute. An angry husband pulls out a knife and starts moving toward the officer in a threatening manner. What's the FIRST thing the officer should do?

a. call for back up squads b. draw his firearm

c. seek cover d. pepper spray the man

Police Practice Test 3

Page 8 of 19

34. How Should An Off Duty Officer Respond To A Crime In Progress.

An off duty officer who is having lunch in a restaurant observes two men enter the restaurant pulling out guns, pointing them at the cashier and demanding money. What should the off duty officer do?
a. He should do nothing. He's off duty c. Attempt to get a detailed description and is the same as a private citizen. and relate the information to the police dispatcher as soon as the robbers leave. b. Attempt to stop the robbery and arrest d. Shoot the robbers as soon as the the robbers. opportunity presents itself. 35. While directing traffic at a busy intersection you observe a man walking across the intersection grabbing his chest in obvious pain. What should you do, FIRST? a. Perform first aid. c. Help the person get out of the roadway to prevent him from being hit by a car. b. Divert traffic and instruct the person d. Inform the person that you will help to lie down in the street. him after you’re finished directing traffic. 36. The LEAST appropriate situation for an officer to respond as an emergency vehicle, with redlights and siren activated is... a. Armed robbery in progress at a service station. b. A traffic accident involving serious injuries. c. A gang fight. d. A hospital reporting that a victim of a stabbing was just brought in.

37. A person making a complaint of a theft seems confused and the facts related seem a bit farfetched and somewhat unrealistic. The Officer taking the complaint should; a. Inform the citizen that his complain is c. unreal. b. Listen to the person's d. complaint,attempt to clarify the facts and inform the person you will investigate the matter. Arrest the person for obstructing an officer. Tell the person there is nothing you can do for him.

38. Officer Brown is sent to an armed robbery of a gas station at 35th and National. Upon arrival at the scene Officer Brown is informed by the complainant that she was just robbed by two white male suspects, armed with handguns who fled South on 35th Street, in a brown pickup truck, with a burned out tail lamp. What should Officer Brown do next? a. Obtain a full description of the armed c. Broadcast the short description given robbers and the vehicle, then to him by the complainant, broadcast it to all squads. immediately.

Police Practice Test 3

Page 9 of 19

b. Returned to his squad to drive south on 35th street to look for the suspects. 39. Officer Zens investigates an armed robbery where two white males, wearing leather jackets and blue jeans, armed with a knife and a hand gun, fled the scene in a blue Chevy. What information in the above description will best help responding squads locate the suspects?

a. type of weapon b. suspects race and sex

c. auto involved d. clothing description

40. Officer Mills and Butler are the first officers at a shooting scene. They interview a witness, who describes the suspect as a white or Hispanic male with a beard, a scar on his left cheek, wearing a red T-shirt ,blue jeans, and white tennis shoes. The suspect was armed with a silver semi auto handgun. What part of this description would be the most useful to other squads in identifying the suspect?

a. white or Hispanic male b. red T-shirt

c. scar on left cheek d. beard

OBSERVING AND RECALLING FACTS AND INFORMATION TEST TASK:

OBSERVING AND RECALLING FACTS AND INFORMATION TEST TASK:

Police Practice Test 3

Page 10 of 19

During a 3 minute time period you must try to memorize as much as you can about the people, objects and events shown in the above photograph.You will not be allowed to take notes during this time period. Look closely at the people in the scene: who they are (e.g., correction officer, inmate or instructor), what they are wearing, what they are doing, etc. Note the setting of the scene: what objects are present, how objects are positioned, what people are doing with objects, etc. After the 3 minutes the photograph should be taken away and you will have to answer questions about them.

Don’t scroll past this point until the 3 minutes are up.
41. What was the group’s focus of attention? a. a weightlifting bench c. a barbell on the ground b. an inmate holding his fist d. an inmate lifting weights 42. The barbell in the photo; a. was being held by an inmate b. was on the barbell rack 43. The man being shown the fist: a. was bald b. was wearing a baseball cap

c. was on the ground d. there was no barbell c. was wearing a knit hat d. had a hood on

44. How many of the men had sweatshirts with hoods? a. 1 c. 3 b. 2 d. 4 45. How many people were in the photo? a. 4 b. 5

c. 6 d. 7

46. Officer Johnson arrives at a traffic accident scene, at 7th and Mills, with no injuries involved. There are two vehicles involved with minor damage. He positions the squad behind the accident with emergency lights activated. What's the next thing Officer Johnson should do? a. Call for a tow truck. b. Have the drivers move the vehicles out of traffic. c. Put the reflective vest on and divert traffic around the accident. d. Interview the witnesses before they leave.

47. You’re on routine patrol and see two men fighting. One of the men punches the other man knocking him to the ground. When you approach the men regarding the altercation, they feel embarrassed by it and state that they're friends, just joking around. The victim who fell to the ground says he does not want to file charges. What do you do about this public disturbance that has occurred in front of you? a. Arrest the man who threw the punch c. Don't arrest either of the parties

Police Practice Test 3

Page 11 of 19

for assault. b. Arrest both men for disorderly conduct.

involved. d. Issue both men citations for disorderly conduct and let them go.

48. When dealing with children a Police Officer should: a. Treat them the same way you treat adults. b. Explain to them how the law can punish them if they do wrong. c. Take an authoritative approach. d. Attempt to gain their confidence.

49. Police Officers should always maintain the following attitude:

a. defensive b. constantly on the alert

c. hostile d. authoritative

50. As a new Police Officer you should LEAST expect your immediate supervisor to; a. Help you avoid errors. c. Make all necessary and difficult decisions for you. d. Inform you of your progress.

b. Give you specific instructions.

51. As the police officer on routine patrol, you find a man laying unconscious on the ground near a bus station. The man smells like alcohol. He does not immediately respond when you try to revive him. The next thing you should do is... a. Handled the situation yourself by c. Do nothing, as intoxicated people continuing to attempt reviving him. often pass out. b. Call for medical aid d. Place the man under arrest for public drunkenness. 52. Officer what would you do, IF? You find a two-year old child, standing outside a grocery store, crying. The people standing outside the store don't know who the child belongs to. What would you do next? a. Broadcast on the police radio that you c. have located a lost child and give a description. b. Take the child to the precinct and try d. to locate the parents. Take the child into the store and try to find his parents. Ask the dispatcher if there has been a report of a missing child in the area.

53. Officer what would you do, IF? You stop a motorist for a traffic violation. He's very rude and indicates he has friends in high places. He demands your name and badge number. What would you do? a. Inform him that the information he c. Tell him you'll give him your badge wants will be on the citation. number but not your name. b. Give him both your name and badge d. Completely ignore his request. number. 54. Officer Henry observes a red Station Wagon drive through a red light. He attempts to stop

Police Practice Test 3

Page 12 of 19

the vehicle regarding the traffic violation. The operator of the vehicle takes off at a high rate of speed through busy rush-hour traffic. Officer Henry recognizes the driver and was able to observe the vehicle's license plate number. Officer Henry's best course of action would be to: a. pursue the vehicle because traffic violators should not be allowed to escape b. not pursue because the risks out weigh the violation c. not pursue because identification has been made and arrest can be made later d. both B and C

55. Officer Johnson is involved in a high speed vehicle pursuit. Regarding continuing or discontinuing the pursuit, Officer Johnson should consider the following conditions, EXCEPT: a. The nature of the violation d. The nature of the area, whether residential, commercial, school zone, open highway etc. b. The type of headlights on the fleeing e. The population density. vehicle c. The volume of the traffic 56. An officer initiating a pursuit, shall in all cases notify the dispatcher as soon as possible that a pursuit is underway and provide the following information EXCEPT: a. Police unit identification b. Location, speed and direction of travel c. Pursued vehicle's description, including license number, if known d. The last time the officer was involved in a vehicle pursuit

57. Officer Smith observes a burglary suspect jump from a rear window of a warehouse. The suspect runs into an alley. Officer Smith starts chasing the suspect and radios for assistance. The suspect runs around the corner of an abandoned building. Officer Smith looses sight of the suspect. What should Officer Smith do next? a. run around the corner after the suspect b. slow down and peek around the corner from a safe distance c. fire a warning shot d. stop chasing the suspect

58. When involved in hand to hand combat, as a police officer you should consider the safety of the following people, in what order? 1. Criminals 2. Fellow officers 3. Civilians 4. You

a. 1,2,3,4 b. 4,2,3,1

c. 2,4,3,1 d. 3,2,4,1

Police Practice Test 3

Page 13 of 19

59. Officer Riley interviews four witnesses to an armed robbery of a gas station. Each witness observed the suspect leaving the scene of the crime. They described the suspect as follows: Witness 1 He was a white male, 20 to 30 years of age with brown curly hair. About 6' 1 ft., 190 lbs. He wore blue jeans and a black leather jacket. Witness 2 He was white or Hispanic, in his late 20's, 5'11", 195 lbs. with curly hair. Who was wearing dark pants and a dark colored jacket. Witness 3 He was a thin white male, in his late 20's about 6 ft., 185 lbs. with a tattoo of a cross on his right bicep. He had brown curly hair. Witness 4 He was a light skinned Hispanic male, with dark curly hair, 5'10" to 6'1" ft about 200 pounds. Wearing a black or brown jacket and blue jeans. Officer Riley should recognize the problem with which witnesses description?

a. 1 b. 2
60. Entrapment occurs when:

c. 3 d. 4

a. Officers arrange the opportunity for c. Officers encourage the commission the commission of a crime of the crime by a person who had no intent to commit a crime until persuaded to do so by the officers b. Officers participate in criminal d. None of the above activity to gain criminal's confidence 61. A suspect wanted for a traffic offense, resists arrest, lunging at the officer with a knife. Regarding this statement which of the following is true: a. The officers cannot use deadly force c. The officer can use to deadly force because the suspect is not wanted for because the suspect is resisting arrest a serious crime b. The officer cannot use deadly force d. The officer can use deadly force because a knife is not considered a because even though the suspect is deadly weapon wanted for a minor crime, the suspect is resisting using deadly force against the officer 62. A police officer arrives at the scene of an accident involving injuries. In what order should the officer do the following: A. Interview witnesses

Police Practice Test 3

Page 14 of 19

B. Determine if anyone needs medical attention C. Move the vehicles off the roadway D. Interview the drivers

a. B , D, C, A b. B, D, A, C

c. B, C, A, D d. D, B, C, A

63. Which of the following crimes has a better chance for suspect identification?

a. auto theft b. burglary

c. robbery d. vandalism

64. Officer Jackson comes across a man who is unconscious, in an alley. Further investigation reveals that the subject has an empty syringe in his hand. Officer Jackson attempts to wake the man but is unable to do so. What should Officer Jackson do next? a. Ignore the situation and let the man wake-up on his own accord. b. Arrest the man for possession of illegal drug paraphernalia. c. Take the man into custody and transport him to nearest the detoxification treatment center. d. Call for an ambulance immediately to transport the subject to medical facility.

65. As an officer on routine patrol, you observe what appears to be a drug deal going down. You observe two individuals in a shopping mall exchange drugs for money. Both suspects observe you and start to run away in opposite directions. What should you do? a. Give a description of the suspects on c. the police radio and chase the suspect who has the money. b. Fire a warning shot in order the d. suspects to halt. Write down a complete description of the suspects, and attempt to seek a warrant for their arrest Give a description on the police radio of the suspects involved and chase the suspect with drugs.

66. You and your partner arrest a suspect for domestic abuse. You feel your partner failed in several regards to adequately assist you in this arrest. You felt his lack of action placed you in unnecessary danger. How should you handle the situation? a. Report your partners lack of action to c. File a written request for a different a supervisor. partner. b. Confront the officer directly and d. None of the above discuss the problem. 67. Officer Roberts observers two men fighting. One of the men stabs the other in the chest with a knife. The victim falls to the ground and he is bleeding profusely. Under the circumstances, what should Officer Roberts do?

Police Practice Test 3

Page 15 of 19

a. Pursue the suspect and attempt to c. Attempt to slow bleeding and call arrest him. for an ambulance . b. Radio for an ambulance and attempt d. None of the above to locate witnesses. 68. Officers must be accurate yet brief when broadcasting on the police radio. What is the best reason for the above statement? a. It makes officers sound more professional b. It saves the department money on radio air time expense c. Tying up the airwaves with long messages on the police radio may prevent emergency transmissions from getting through. d. Shorter messages are easier to understand.

69. Officer Jones sees a motorist commit a traffic violation at a construction site where street signs have recently been changed. What action should Jones take? a. Write down license plate and if he sees the motorist commit this violation again issue him/her a citation. b. Ignore the incident because of the recent change. c. Stop the offender and issue a warning. d. Stop the offender and issue a citation.

70. Upon arrival at the scene of a demonstration, Officer Todd observes two officers fighting with demonstrators. Officer Todd pulls out his baton and goes to help the other officers. A demonstrator, not involved in the fight swings a broken bottle at Officer Todd. What should Officer Todd do next?

a. Grab the bottle out of the man's hand c. Call for more back up b. Administer several blows with the d. Shoot the man baton in the attempt to disarm him
71. Officer what would you do if? You and your partner respond to a burglary in progress at an office building. Upon arrival you observe a broken window and you see someone moving around inside the building. You call for additional squads to help search the building. Until backup arrives what should you and your partner do? a. Position yourself at one corner of the c. building, while your partner positions herself at the diagonally opposite corner of the building. b. You position yourself at the front d. entrance, while your partner positions herself at the rear exit. You and your partner enter the building and conduct a search for the suspect(s). One officer maintains surveillance on the perimeter of the building, while the other officer enters the building to search for the suspects.

Police Practice Test 3

Page 16 of 19

72. Officers conducting missing persons investigation should include the following information in their report, EXCEPT: a. if the person has a medical emergency b. detailed physical and clothing description c. physical or mental disability d. religion e. wanted status

73. An officer conducting a search of a prisoner she just arrested, should consider the following, EXCEPT:

a. Checking legs and ankles b. Searching the prisoner from the prisoner's side.

c. Searching the prisoner from the rear. d. Checking neck area and both arms

74. When should a person be advised of their Constitutional Rights? a. Before they're arrested. c. After they're taken into custody and before they're questioned about a crime. b. Right after they're taken into custody. d. After they're taken into custody and before they are taken to court. 75. Officer Ryan arrives at the scene of a traffic accident where there are no injuries involved. Both vehicles have significant damage and cannot be driven. Regarding the vehicles involved, what should Officer Ryan do FIRST? a. Tow the vehicles. b. Have the drivers move the vehicles out of traffic. c. Leave the vehicles in the roadway until the investigation is completed d. Position the squad behind the autos involved in the accident, with emergency lights activated.

76. A police officer, chasing a suspect on foot should do the following, EXCEPT:

a. Radio for assistance immediately. c. Pace the suspect. b. Tackle the suspect when he gets close d. Maintain visual contact. enough.
RULE INTERPRETATION: This question is based on rule interpretation. RULE: Patrol vehicles should be checked at the start of each shift. Do not assume that the vehicle is in satisfactory condition. Check all of the lighting equipment, all emergency equipment, siren, engine, oil, transmission fluid, battery, radiator and gasoline levels, tire pressure and condition (including spare), lug wrench, jack, windshield wipers and windshield washer fluid level. Check the body of the vehicle for damaged or missing parts and report any

Police Practice Test 3

Page 17 of 19

problems, damage or discrepancies to your supervisor. At the end of your shift, leave the vehicle in optimum condition for emergency use by the next officer. SITUATION: Officer Burton is about to begin her patrol shift when she discovers that her police vehicle has a large dent in the left rear bumper. She knows that the vehicle did not have this dent yesterday, when she last drove it. 77. QUESTION: According to the Rule above, Officer Burton should most properly... a. request that she be assigned a different vehicle b. begin her shift and be alert to any operating problems c. find out what other officers have used the vehicle since her last shift d. inform her supervisor about the dented bumper

PREPARING WRITTEN MATERIAL IN A POLICE SETTING: This question tests your ability to prepare the types of reports that police officers write. You are presented with a page of notes followed by several questions. Each question consists of four restatements of the information given in the notes. From each set of four, you must choose the version that presents the information most clearly and accurately.

TEST TASK: You must determine which one of the choices presents all the information from a particular portion of the notes and whether the phrasing and the punctuation of the sentence(s) results in a clear and accurate presentation of the information.
QUESTION: The following is a portion of the notes you took about an accident you were dispatched to. NOTES: Responded to a call from 26 Arbor Ave. Residence of Tessa and John Wynter. Pulled in driveway. Saw woman on Wynters’ porch. Identified herself as Mrs. Orvis, a neighbor. 78. Which one of the choices that follow expresses the facts presented in the notes? a. I responded to a call from 26 Arbor c. When I responded to a call from 26 Avenue, the residence of Tessa and Arbor Avenue, the residence of John Wynter. When I pulled into the Tessa and John Wynter, I saw driveway, I saw a woman on their pulling into their driveway a woman porch. She identified herself as Mrs. on their porch who identified herself Orvis, a neighbor. as Mrs. Orvis, a neighbor. b. Responding to a call from 26 Arbor d. Responding to a call from 26 Arbor Avenue, I saw a woman on the porch Avenue, the residence of Tessa and John Wynter, and pulling into the of Tessa and John Wynter’s driveway, I saw a neighbor on their residence. She identified herself as porch, who identified herself as Mrs. Mrs. Orvis, a neighbor. Orvis. READING, UNDERSTANDING AND INTERPRETING WRITTEN INFORMATION:

Police Practice Test 3

Page 18 of 19

This question tests your ability to read, understand, and interpret the kinds of written information that police officers are required to read during their formal training TEST TASK: You are provided with brief reading selections and asked questions relating to the selections. All the information required to answer the questions is provided in the selections; you are not required to have any specific knowledge of the content areas covered in the selections. QUESTION: "The increasing demands upon our highways from a growing population, and the development of forms of transportation not anticipated when the highways were first built have brought about congestion, confusion, and conflict, until the yearly toll of traffic accidents is now at an appalling level. If the death and disaster that traffic accidents bring throughout the year were concentrated into one calamity, we would shudder at the tremendous catastrophe. The loss is no less catastrophic because it is spread out over time and space." 79. Which one of the following statements concerning the yearly toll of traffic accidents is supported by the above selection? a. It is increasing the demands for safer c. It does not shock us as much as it means of transportation. should because the accidents do not all occur together. b. It has resulted in increased d. It has resulted mainly from the new congestion, confusion, and conflict forms of transportation. on our highways. MEMORY FOR FACTS AND INFORMATION: This question tests how well you can remember facts and information presented in written form after you have been given a period to read and study the information. TEST TASK: You are first given a Memory Test Booklet containing a story. It will be considerably longer than the one presented here. You will have a limited period of time in which to study the details contained in the story. You will NOT be allowed to take notes while studying the story. At the end of the study period, the monitor will collect the test booklets containing the story and then hand out the test booklets containing the test questions. The first group of questions in this test booklet will ask you to recall the facts and information presented in the Memory Story. MEMORY STORY: Officer Gary Hanson of the Burke Police Department was questioning Mathew Meyers, the owner of Meyers Sporting Goods, located at 321 Payne Avenue about a burglary that occurred the previous evening. Meyers said that when he arrived at the store at 8:50 A.M., he noticed that the rear door had been broken into. Meyers said that, after he had checked his inventory, he was missing 20 rifles, 16 pellet guns, 12 shotguns and 8 pistols. 80. QUESTION: How many shotguns did Meyers tell the Officer were missing from his store? a. 8 c. 16 b. 12 d. 20

Police Practice Test 3

Page 19 of 19

81. You are an off-duty police officer, invited to a party at a nearby home (private residence). While at the party, you observe a couple of people using illegal drugs. The best course of action would be: a. Do nothing.You are off-duty. Stay and enjoy yourself. c. Call the dispatcher and have whomever is on-duty to come and arrest the people concerned. b. Approach the people concerned and d. Leave the party immediately. arrest them. Drugs are illegal. 82. The 4th Amendment to the Constitution provides a high degree of protection for individuals from searches and seizures made by the police. Searches and seizures carried out by officers without a warrant must be clearly justified by officers as falling into one of the well-defined exceptions to the Fourth Amendment requirement. To avoid suppression of evidence and possible personal liability an officer should: a. always obtain a warrant before instituting a search and a seizure b. only attempt to obtain a warrant whenever no other options are available before instituting a search and a seizure c. always attempt to obtain a warrant whenever reasonably possible before instituting a search and a seizure d. obtain a warrant 7 days before instituting a search and a seizure

83. Use the map below to answer the sample problems.

You are at the store on Lake and heading to the cleaners on Forest. What is the shortest way to get there? a. North on Lake to Forest, east on c. South on Lake to Indian, east on Forest. Indian to Meadow, north on Meadow to Forest, west on b. South on Lake to Indian, east on d. South on Lake to Indian, east on Indian to Black, north on Black to Indian to Pacific, north on Pacific to Forest, east on Forest. Forest, west on Forest.

Police Practice Test 4

Page 1 of 16

Police Practice Test 4
Multiple Choice

Identify the letter of the choice that best completes the statement or answers the question.
You have 2 hour and 20 minutes to complete the test.

1. Which of the following is NOT entrapment?

a. An officer observes a car traveling d. A Patrolman in an unmarked car on behind him. The officer speeds up a a deserted highway challenges a little and the car follows suit. The persons to race and arrests the officer continues to increase his person prior to actually drag racing. speed, clocks the following car and arrests the driver for speeding. b. An undercover officer begs a drug user to sell the officer some drugs claiming that he is suffering from withdrawal symptoms. c. An undercover officer acts as a drunken derelict in a high crime area and arrests two men attempting to rob him. 2. Which one, of the following search and seizure situations is exempt from requiring a warrant? a. searching hidden places in a dwelling d. a b & c for contraband b. automobile search e. a & b c. searching the person when no charges are likely be filed 3. When a person politely refuses to answer an officer's questions during a stop, what should e. All of the above are examples of entrapment.

... 6/23/2004

Police Practice Test 4

Page 2 of 16

the officer do? a. allow the person to go on his way. d. issue a citation for loitering. b. demand that the person comply with e. arrest the person for obstructing an the request. officer . c. search the person for identification. 4. Officer Jones hears a shot and observes a subject running into a home with a gun. Which of the following should Officer Jones do? a. Obtain a warrant to enter the home c. Enter without a warrant because there is an inherent threat of injury or death to others b. Attempt to get consent to enter d. Ignore the situation 5. When are Miranda warnings required? a. when the investigation has moved c. When a person enters a police from the investigatory to custodial station and confesses to a crime stage b. where an officers stops a speeding auto 6. The critical element that constitutes entrapment is whether: a. the defendant was predisposed to d. participation on the part of the commit the crime in question officer in order to gain evidence occurred b. the opportunity to commit a crime e. the degree of cooperation by the was arranged by the police officer destroys the essential element of the crime c. deception was used to gain evidence. 7. Entrapment occurs when: a. Officers arrange the opportunity for c. Officers encourage the commission of the crime by a person who had no the commission of a crime intent to commit a crime until persuaded to do so by the officers b. Officers participate in criminal d. None all the above activity to gain criminal's confidence 8. Under what circumstances may an officer, on patrol, stop a person? a. The officer doesn't like the way the d. The officer has mere suspicion person looks b. The officer has reasonable suspicion e. None of the above c. The person says hello to the officer and walks away 9. Which of the following would constitute exigent circumstances? a. immediate threat of escape c. immediate threat of death of great bodily harm b. immediate threat of destruction of d. all the above evidence 10. An officer may arrest a subject on the belief that a _______ warrant has been issued in

Police Practice Test 4

Page 3 of 16

another state. a. Misdemeanor b. Felony

c. Misdemeanor or felony d. None of the above

11. That quantum of evidence which would lead a reasonable police officer to believe the defendant has committed a crime, is called: a. an arrest c. probable cause b. a stop and frisk d. a warrant READING COMPREHENSION This section of the test measures how well you understand what you read. It consists of a number of short paragraphs followed by questions. The information needed to answer the questions is contained in the paragraphs. In some instances, you may have to draw a conclusion based on the information given. Sample Passage There are several types of investigation or inquiries, which police typically undertake as part of their job. Investigations of incidents, which are violations of laws and ordinances, include crime and traffic accident investigations. Personnel investigations are conducted to evaluate the suitability of individuals for positions of public trust. 12. According to the passage, conducting investigations is a routine part of a police officer's job. a. True b. False 13. According to the passage, police investigations of incidents which are violations of laws and ordinances, include: a. traffic accidents c. personnel b. crime d. A and B only 14. Choose the answer which most closely describes the word POWERFUL

a. happy b. energetic
15. Add: 327+512 a. 475 b. 510

c. strong d. blue c. 545 d. 839

16. A police officer has to search a warehouse that is 60 feet long and 40 feet wide. What is the perimeter of the warehouse? a. 100 feet c. 200 feet b. 150 feet d. 240 feet 17. 3 X 9 + 15 = a. 27 b. 42

c. 72 d. 92

18. Which of the following is not a mammal:

Police Practice Test 4

Page 4 of 16

a. bat b. horse

c. whale d. shark

19. The permanent set of teeth in a human adult including wisdom teeth numbers:

a. 28 b. 30

c. 32 d. 34

20. Which one of these birds does not have feathers:

a. penguin b. ostrich c. eagle

d. a robin e. a they all have feathers

21. When formulating goals into specific activities, one should: a. Give priority to activities done easiest. b. Overrule any previously scheduled appointments. 22. Teachers: a. are concerned with the welfare of every student b. can help students by encouraging critical thinking 23. Movies today: a. invariably promote death and c. seem to be much more graphic in violence and anarchy. nature. b. are the cause of the increasing crime d. all of the above. rate in America. 24. How many sides does the Pentagon have in Washington DC: c. Make sure activities have a date, time, and place to ensure their fulfillment. d. Wait two weeks or until no longer busy to start activities. c. are unfair d. are all critical thinkers

a. eight b. nine

c. five d. seven

25. The skiing industry is more profitable in the following state:

a. Utah b. Florida

c. Puerto Rico d. Louisiana

26. You can have hallucinations if you eat: a. raisin bran c. LDS b. LSD d. beans 27. The University of Michigan is in the ________ Conference:

Police Practice Test 4

Page 5 of 16

a. Western Athletics b. Big Twelve's
28. A species is a:

c. Big Ten d. General

a. bunch of similar looking organisms c. group of organisms that live in the same place b. genetically distinctive natural d. group of individuals that cannot population that shares a common breed with each other gene pool 29. Altruism, as defined by social scientists, is: a. selfless concern for others and unselfish behaviors that foster the welfare of others b. social norm of reciprocity 30. Patrimony is: a. a father's relationship with his children b. a neutral nation c. property inherited from one's father or ancestors d. the cutting of a precious stone c. connected series of rules that govern a complex pattern

d. running away from the situation

31. Several hours before an athletic event, athletes should eat a lot of:

a. candy bars b. high protein foods

c. fatty foods d. carbohydrates

32. In the following series, what comes next? 1, 2, 4, 8... a. 6 c. 32 b. 16 d. 15 33. In the following series, what comes next? 1, 3, 5, 7... a. 9 c. 11 b. 10 d. 13 34. In the following series, what comes next? 1, A, 9, I, 17... a. Q c. E b. Z d. U 35. The bar graph below lets us compare the sales of each fruit over a 2 day period, not just the sales of one fruit compared to another. We can see that the sales of star fruit and apples stayed nearly the same. The sales of oranges increased from day 1 to day 2 by ___ kilograms.

Police Practice Test 4

Page 6 of 16

a. 4 b. 14

c. 10 d. 21

36. The bar chart below shows the weight in kilograms of some fruit sold one day by a local market. We can see that ___ kg of apples were sold, 40 kg of oranges were sold, and 8 kg of star fruit were sold.

a. 32 b. 52

c. 67 d. 59

37. In the following series, which letter comes next? A, C, E, G, ..

a. I b. K
38. Which does not belong?

c. M d. H

a. grape b. apple

c. corn d. orange

Police Practice Test 4

Page 7 of 16

39. Fill in the blank in this series: 000, 001, 011,___.

a. 111 b. 101
40. Which is faster?

c. 010 d. 110

a. light b. sound

c. fly d. eagle

41. If every day is a Sunday, and there is no work on Sundays:

a. Everyone goes to church b. No one works
42. What color is the sky at night?

c. Everyone works everyday d. Tomorrow is Saturday

a. blue b. gray

c. yellow d. black

43. If a car takes 10 minutes to travel 60 km, how long would it take to travel 1800 km. at this same rate? a. 300 hours c. 50 minutes b. 4 hours d. 300 minutes 44. If all males are green, and all females are red then:

a. All boys are pink b. All girls are orange

c. All boys are green d. All girls are green

45. If it takes 20 years for a tree to grown 1 meter. How long would it take for the tree to become 5 meters in height?

a. 4 years b. 100 meters

c. 100 years d. 20 years

46. Arrange the following words to form a sentence and indicate whether this sentence is true or false. ( men, mortal, all, are.) a. true b. false 47. If it takes 8.3 minutes for the light of the sun to reach the earth, and the speed of light is 300,000km/s per minute. How far is it to the sun?

a. 149400000 km b. 24900000 km

c. 5 cubic cm d. 928328.56 MI

48. Which of the following is closest in meaning to: dipsomaniac a. sot c. hydrophobic b. roger d. repose

49. If one were to be pusillanimous one would be:

Police Practice Test 4

Page 8 of 16

a. dead b. in love
50. Eros is:

c. a coward d. unconscious

a. darkness b. love

c. the word for grape in Pakistan d. a natural cleanser

51. If you rearrange the letters "aber", what would you have the name of?

a. Automobile b. Town c. State

d. Animal e. Vegetable

52. Which activity does not belong in this group?

a. jump b. run c. jog

d. read e. ski

53. Which number doesn't belong in the following numerical sequence? 2 4 6 8 9 10 12 14

a. two b. eight c. nine

d. twelve e. fourteen

54. If you rearrange the letters "faegrfi", what would you have the name of?

a. Town b. State c. Animal

d. Vegetable e. Mineral

55. Which word makes the best analogy? A IS TO Z as START IS TO:

a. go b. finish

c. begin d. wait

56. Tom is Mary's aunt's nephew. Which of the following statements is most likely NOT to be true?

a. Tom is Mary's Father b. Tom is Mary's cousin c. Tom is Mary's half brother

d. Tom is Mary's brother e. Tom is related by blood to Mary

57. Which word makes the best analogy? SWIM IS TO WALK as BOAT IS TO:

a. Airplane b. Ship

c. Jet d. Car

Police Practice Test 4

Page 9 of 16

58. Which word makes the best analogy? PITCH IS TO THROW as STEP IS TO:

a. Touch b. Leg c. Fly

d. Match e. Walk

59. Which number does not belong in the following sequence? 4 6 5 7 6 9

a. four b. five c. six

d. seven e. nine

60. If you purchased an item for 64 cents and you received five coins in change from your one dollar bill, which kind of coin would you receive the most of?

a. Penny b. Nickel

c. Dime d. Quarter

61. Which item does not belong in this group?

a. Coat b. Watch c. Shirt

d. Socks e. Dress

62. In another language, "clup sonk wita" means "dead bad enemies" and "zorn gin clup" means "enemies hate people" and "fal wita nomi" means "see dead animals." What does "sonk" mean?

a. Dead b. Bad c. Hate

d. Nothing e. Enemy

63. Which word is least like the other four?

a. Cold b. Snow

c. Mild d. Hot

64. Which of the five is least like the other four?

a. Car b. Bus c. Wagon

d. Van e. Bicycle

65. If you rearrange the letters "giaccho", what would you have the name of?

a. Animal b. Vegetable

d. Country e. Sea

Police Practice Test 4

Page 10 of 16

c. City
WRITING SKILLS This test measures three aspects of good writing: clarity, vocabulary, and spelling. Instructions: In the following pairs of sentences, identify the sentence which is most clearly written. If sentence "A" is clearer than sentence "B", mark "A" on your answer sheet. If sentence "B" is more clear than sentence "A", mark "B" on your answer sheet. Make no marks in the test booklet. 66. Identify the sentence, which is most clearly written. a. The officer was hoping to get a new b. The officer was hoping to get a new partner, one that had a great deal of partner. One that had a great deal of experience. experience. 67. Identify the sentence which is most clearly written. a. Bullet fragments were gathered by officers in envelopes. b. Bullet fragments were gathered in envelopes by officers.

68. Identify the sentence, which is most clearly written. a. The suspect disliked the officer, as did the judge. b. The suspect disliked the officer as much as the judge.

69. Identify the sentence which is most clearly written. a. The officer had probable cause to b. When the officer arrived at the arrest the suspect when he arrived at scene, he had probable cause to the scene. arrest the suspect. 70. Identify the sentence, which is most clearly written. a. Jail is not a pleasant place to be, but b. Jail is not a pleasant place to be, but they do get their basic necessities prisoners do get their basic met. necessities met. VOCABULARY Instructions: In each of the following sentences, choose the word or phrase that has the same meaning as the underlined word. 71. The witness corroborated the suspect's story.

a. verified b. contradicted

c. added to d. questioned

Police Practice Test 4

Page 11 of 16

72. It was not a very pragmatic plan. a. plausible b. serious

c. practical d. sensible

73. The police sequestered the suspect.

a. caught b. isolated

c. arrested d. released

74. In order to conceal her guilt, Linda told a blatant lie.

a. harmless b. subtle

c. careless d. obvious

75. He was ignorant of the proper procedures.

a. unaware b. conscious

c. uncertain d. cognizant

READING COMPREHENSION
Instructions

Officers must read and understand a wide range of materials. This test is designed to measure your ability to read and understand various types of written material. Read each paragraph or passage and choose the statement which best answers the question. All questions pertain only to the material in the passage which precedes them. Therefore, you are to choose your answer solely on the basis of the material contained in the passage. Mark the letter which identifies your choice in the space on your answer sheet. Do not spend too much time on any one item.
The case of Gideon v. Wainwright, decided by the Supreme Court in 1962, granted courtappointed counsel to people charged with felonies who could not afford to pay for their own attorneys. In a more recent case, the Supreme Court declared that the right extends to persons who are charged with any crime for which prison is a potential penalty. According to these court decisions, a defendant may waive his or her right to be represented by counsel at trial, but the state must at least provide the defendant with the opportunity to have free counsel. 76. Based on the above passage, which of the following statements is most correct regarding defendants who cannot afford an attorney: a. anyone who must appear in court has c. anyone charged with a felony must the right to a free attorney have an attorney present for his or her trial b. anyone whose charges may result in d. anyone charged with an offence that prison must have an attorney could lead to prison must have the representing him or her at trial option of a free attorney 77. During the 1970s, rural America registered a population gain of almost 14 percent, while metropolitan growth was just under 10 percent. The surprising spurt in rural growth had

Police Practice Test 4

Page 12 of 16

nothing to do, however, with prolific reproduction. On the contrary, rural areas were mired in aging townsfolk. This fact, combined with trend toward smaller families, actually resulted in a 10 percent decline in the existing rural population between 1965 and 1970. Newcomers more than made up for that loss. The latest census tells us that one sixth of the U.S. population now lives in rural areas and 40 percent of all new housing in the 1970s was built on rural land. The rural areas are now growing faster than the metropolitan areas for the first time since the early 1800s. Based on the preceding passage, which of the following statements is most accurate? a. Rural areas were mired in aging c. The overall gain in the American townsfolk because the young and population was almost 14 percent. farmers were immigrating to the cities. b. There has been more than a 10 d. The trend toward small families percent increase of newcomers to contributed to the decrease in the rural areas. urban population growth rate. 78. Entrapment is defined as officers or agents of the government provoking a person to commit a crime, that he did not originally contemplate, in order to prosecute him. The goal of law enforcement is not to encourage nor to create an offense, but to prevent people from committing crimes and/or arrest people when they do commit offenses. Based on the preceding passage, which of the following statements describes an instance of entrapment? a. Property that has been marked by c. police so that they can trace it at a later time. b. A victim, learning that a person d. intends to rob him, does nothing to stop the crime, but instead allows the robber to carry through with the crime so that police will catch him in the act. A door to a warehouse containing valuable merchandise is purposely left open by police. An undercover officer approaches a known drug dealer and tries, unsuccessfully, to purchase drugs.

79. Community policing has gained increased acceptance during the past 10 years. The community policing model places service to the public and prevention of crime, as the primary role of police in society. This also emphasizes problem solving, with active citizen involvement in defining those matters that are important to the community, rather than crime fighting and arrest statistics. Officers at the patrol level are required to spend less time in their cars communicating with other officers and more time on the street communicating with citizens. Proponents of this style of policing insist that addressing the causes of crime, makes police officers more effective and at the same time enhances the quality of life in the neighborhood. Community policing concepts, if successfully implemented, offer the prospect of effective crime prevention and substantially improved community relations. Although community-based policing is not a panacea for the problems of crime in society, it does offer valuable opportunity to both reduce crime and enhance the police image. According to the preceding passage, which of the following statements is most accurate?

Police Practice Test 4

Page 13 of 16

a. Community policing is expected to c. increase the number of arrests made by patrol officers. b. Implementing community policing is d. expected to decrease the amount of crime.

Community policing is not designed to lessen crime.
The community policing model defers decision-making to citizens and community members.

GENERAL ORDER 62 (Radio Codes) The purpose of this general order is to describe the coding system that identifies the various departmental and non-departmental units. Below you will find three components of police radio codes (A, B, and C). When a dispatcher sends officers out on a call, these 3 code components are used. Each call number shall be comprised of three components: 1. the first digit identifies shift 2. the alpha digit describes the operational assignment 3. the last two digits identifies unit or area A. Call numbers shall be assigned as follows: Shift designation 1 day 2 swing 3 graveyard B. Operational assignments A. patrol division, platoon 1 B. patrol division, platoon 2 C. patrol division, special activities D. patrol division, gang unit E. traffic division, motorcycle unit 1 F. traffic division, motorcycle unit 2 G. traffic division, administration H. investigation division, fraud I. investigation division, drugs J. administration division C. Unit or Area 1-13 Metropolitan area 13-20 Airport 21-40 Industrial parks 41-45 Residential areas 80. Using the above coding system, the call number 2-B-12 would identify a. swing shift, platoon 2, metropolitan c. swing shift, administration, area residential b. graveyard, platoon 1, airport d. day, traffic unit 1, airport SPELLING 81. His ______________ of the situation was incorrect.

Police Practice Test 4

Page 14 of 16

a. analisys b. annalysis

c. analysis d. anallysys

82. It was not _____________ who committed the crime.

a. apparent b. apperant
83. She was a ____________ worker.

c. aparent d. aperant

a. conscienteous b. consceintious

c. consceinteous d. conscientious

84. She did not actually _____________ the stolen goods.

a. receive b. recieve

c. recieve d. recive

85. There was only one witness to the boy's ______________.

a. abducton b. abducsion

c. abduction d. abduckion

86. Propaganda is a(n) — of truth, a mixture of half-truths and half-lies calculatedto deceive.

a. revision b. perversion c. dissension
87. NOSE: HEAD::

d. perception e. invasion

a. hand: arm b. foot: toe c. eye: lid
88. WHEAT: GRAIN ::

d. wrist: finger e. teeth: gums

a. cow: beef b. orange: lime c. carrot: vegetable
89. COTTAGE: CASTLE::

d. coconut: palm e. hamburger: steak

a. house: apartment b. puppy: dog c. dory: liner
90. OLD: ANTIQUE::

d. man: family e. poet: gentleman

a. new: modem b. cheap: expensive

d. wanted: needed e. rich: valuable

Police Practice Test 4

Page 15 of 16

c. useless: useful
91. DIGRESS: RAMBLE::

a. muffle: stifle b. rust: weld c. introduce: conclude

d. rest: stir e. find: explain

92. If a car can drive 25 miles on two gallons of gasoline, how many gallons will be needed for a trip of 150 miles?

a. 12 b. 3 c. 6

d. 7 e. 10

93. A school has enough bread to feed 30 children for 4 days. If 10 more children are added, how many days will the bread last?

a. 5 1/3 b. 1 1/3 c. 2 2/3

d. 12 e. 3

94. He dashed into the house, ran for the phone, and answered—, tripping over the cord.

a. hesitantly b. nobly c. soothinsly

d. distantly e. breathlessly

95. The criminal record of the witness caused the juryto ------ his testimony.

a. affirm b. belie c. retract
96. Write .2% as a decimal.

d. acauit e. discredit

a. .2 b. .02
97. Write 3.4% as a fraction.

c. .002 d. .2

a. 34/1000 b. 34/10
98. Write ¾ % as a decimal.

c. 34/100 d. 340/100

a. 75 b. .075 c. .0075

d. .00075 e. 7.5

Police Practice Test 4

Page 16 of 16

99. Find 60% of 70.

a. 420 b. 4.2 c. 1162

d. 4200 e. 42

100. By a stroke of luck, the troops -------, avoiding a crushing -------.

a. converged-blow b. transmuted-oblivious

c. prevailed-defeat d. diverged-siege

Police Practice Test 5

Page 1 of 14

Police Practice Test 5
Multiple Choice

Identify the letter of the choice that best completes the statement or answers the question.
You have 1 hour and 20 minutes to complete the test.

1. In the following pairs of sentences, identify the sentence, which is most clearly written. If sentence "A" is clearer than sentence "B", mark "A" on your answer sheet. If sentence "B" is more clear than sentence "A", mark "B" on your answer sheet. a. Bullet fragments were gathered by b. Bullet fragments were gathered in officers in envelopes. envelopes by officers 2. Identify the sentence, which is most clearly written. a. The officer was hoping to get a new partner, one that had a great deal of experience. b. The officer was hoping to get a new partner.One that had a great deal of experience.

3. Identify the sentence, which is most clearly written. a. The suspect disliked the officer as did b. The suspect disliked the officer as the judge. much as the judge.

4. Identify the sentence, which is most clearly written. a. The officer had probable cause to arrest the suspect when he arrived at the scene. b. When the officer arrived at the scene, he had probable cause to arrest the suspect.

5. Identify the sentence, which is most clearly written. a. Jail is not a pleasant place to be, but b. Jail is not a pleasant place to be, but prisoners do get their basic necessities they do get their basic necessities met. met.

...

Police Practice Test 5

Page 2 of 14

OBSERVATION This section will test your ability to attend to details in visual material. For each questions, you will examine a set of five sketches of faces. For each set, the first drawing will be identified as a sketch of an alleged criminal. The other drawings will be labeled “a”, “b”, “c”, and “d”. For each set of drawings, one of the drawings labeled “a”, “b”, ”c”, or “d” will have near identical facial features as that identified as the alleged criminal. Your task is to identify this drawing. In making your decision, you are to assume that no plastic surgery has occurred.

To complete the task accurately, focus on features that are unlikely to be altered without plastic surgery. For example, examine the shape of the eyes, the size and appearance of the nose, the shape of the face, etc. Do not be distracted by features that can easily be altered. For example, hair style and color, facial hair, and clothing. Use a process of elimination when completing the task. Eliminate any pictures where you notice a difference in a feature that should not change. Once three pictures have been eliminated, the remaining picture should be the correct answer.
6. Identify the suspect:

Police Practice Test 5

Page 3 of 14

Which of the above drawings is most likely the suspect? a. Mug shot A c. Mug shot C b. Mug shot B d. Mug Shot D 7. For each set of drawings, one of the drawings labeled “a”, “b”, ”c”, or “d” will have near identical facial features as that identified as the alleged criminal. Your task is to identify this drawing.

Police Practice Test 5

Page 4 of 14

Use a process of elimination when completing the task. Eliminate any pictures where you notice a difference in a feature that should not change.
Which of the above drawings is most likely the suspect? a. Mug shot A c. Mug shot C b. Mug shot B d. Mug shot D STREET SCENE Answer the following questions based on the following crime scene sketch. Sketches often present unclear images so your task is not only to identify the events in the picture but also figure out what the sketches represent.

Police Practice Test 5

Page 5 of 14

You'll have five minutes to examine the sketch. The questions will test your ability to recall facts, figures and events occurring on the sketch.

You have five minutes to examine the sketch.

8. Which one of the following vehicles is parked on Rock Ave ? a. A police car c. A tow truck b. A taxi d. A fire truck 9. What are the words printed on the side of the bus?

a. Northside bus b. Southside bus

c. Eastside bus d. Westside bus

Police Practice Test 5

Page 6 of 14

10. The man with the gun is wearing a:

a. white shirt with black stripes b. T-shirt with the words spot news

c. sweatshirt with the word Ace d. white shirt with black dots

11. At what intersection is the man changing the tire?

a. Rock and Elm b. Hess and London

c. Howard and Crawford d. Smith and First

12. The person in the second floor window is directly above;

a. Uncle Dan's Lunch b. Mandley Discount Clothing Outlet

c. Cafeteria Europa d. M & P Music World

13. What is the man with the gun stealing from the woman?

a. a purse b. a necklace

c. earings d. a car

14. The ability to identify and resolve problems, and also to apply the principles of logic to given situations, is needed to perform many state jobs.These types of questions are designed to measure the applicant's ability in these areas.

New York time is three hours ahead of San Francisco time. What time would it be in New York if a San Francisco clock, one hour behind the time, shows 4 o'clock? a. 1 o'clock c. 7 o'clock b. 6 o'clock d. 8 o'clock
15. Most of the Gaspard family lives in South Louisiana. Many of the people in South Louisiana celebrate Mardi Gras. Frank is a member of the Gaspard family. Based on these facts, we can conclude that: a. Frank lives in South Louisiana. c. All of the Gaspards celebrate Mardi Gras. b. Frank does not celebrate Mardi Gras. d. None of the above can be concluded. 16. Bill must use Highway 19 to get to work. Bill has a meeting today at 9:00 AM. If Bill misses the meeting, he will probably will lose a major account. Highway 19 is closed all day due to repairs. Based on the above, we can conclude that: a. Bill will not be able to get to work. c. Bill will lose a major account. b. Bill will probably not be able to d. None of the above can be concluded. reschedule the meeting. 17. Mrs. Sellers reports that she was in the elevator late yesterday evening after leaving her office on the 15th floor of a large office building. A man got on at the 12th floor, pulled her off the elevator, and assaulted her, stealing her purse. She believes that she has seen the man in elevators and hallways of the building before. She thinks that he works in the building. Study the following parts of Mrs. Sellers' description of the man. Which one would most useful in finding him, assuming that he is a regular occupant of the building? a. He had very bad breath. c. He had a scar on his left cheek. b. He was wearing a striped tie. d. He was carrying a blue backpack.

Police Practice Test 5

Page 7 of 14

Reading Comprehension

Many state jobs require skill in analyzing, understanding, and interpreting written material of varying levels of difficulty. The reading comprehension questions on our tests are designed to measure applicants' abilities to understand and interpret written material. These questions require an employee to read and understand a paragraph, and then, to choose an answer based on their understanding of the main concept put forth in the written passage. The correct answer will usually restate this main concept, using different wording. In some cases, the correct answer will be a conclusion that is drawn from the content of the paragraph. After reading the passage, choose as your answer the statement that is best supported by the contents of the passage. A viable affirmative action program must contain specific procedures designed to achieve equal employment opportunities for specified groups. Appropriate procedures, without necessary determination to carry them out, are useless. Determination, without well defined procedures, will achieve only partial success.
18. The paragraph best supports the statement that: a. Well defined procedures will assure thed. An agency may guarantee success of success of an affirmative action its affirmative action program by program. developing and implementing well defined procedures. b. A high degree of determination is e. Two important ingredients of a necessary and sufficient for a highly successful affirmative action program successful affirmative action program. are well defined procedures and a sincere resolve to implement those procedures. c. It is impossible for an agency to develop a viable affirmative action program. 19. Education in the United States is a state responsibility, a local function and a federal concern. Unlike other social service programs, this arrangement also places state governments between the federal government and local governing bodies. The paragraph best supports the statement that: a. The federal government plays an d. No federal funds are used to support advisory role only in matters local educational programs. concerning education. b. Enforcement of federal education e. Federal aid is often used to induce policies is left to state discretion. local school systems to implement federal policies. c. Federal educational policies are generally implemented by local governments under the direction of the state. 20. Technological and psychological conditions are changing so rapidly that most agencies and organizations must continually adapt to new situations in order to remain viable. The paragraph best supports the statement that: a. Changes in general conditions d. The effectiveness of an organization is determine the effectiveness of an equally dependent upon technological organization. advances and psychological changes.

Police Practice Test 5

Page 8 of 14

b. The effectiveness of an organization e. The effectiveness of an organization is depends more on technological dependent upon its technological and advances than on psychological psychological advances. changes. c. Organizations must be able to adapt to technological and psychological changes in order to maintain effectiveness. 21. Choose the sentence that represents the best English Usage. a. Of the two runners, John is the worst. c. John is the worst of the two runners. b. Of the two runners, John is the better. d. John is the best of the two runners. 22. Choose the sentence that represents the best English Usage. a. We seldom ever receive this type of c. Each of these regulations apply to request anymore. your case. b. Neither of the employees are doing d. I have enclosed a copy of the file you what is expected of him. requested. 23. Both women have made previous complaints, none of which were followed through because of insufficient evidence or strong fear of retaliation by the suspect. The suspect is attempting to intimidate the women and is known for irrationale behaviour. Which word in the sentences above is misspelled?

a. insufficient b. retaliation

c. intimidate d. irrationale

24. On the night of the accident, coeficient of friction testing was conducted using police transport on a cycloidal skid mark. Which word in the sentence above is misspelled? a. accident c. friction b. coeficient d. cycloidal 25. Which word completes the next sentence? Once she became a public figure, she had to give __________ her anonymity.

a. about b. back

c. in d. up

26. Which words complete the next sentence? My partner and _____ went to the home and _____ knocking on the door.

a. I, began b. me, began

c. me, begun d. I, begun

27. Which of the following is the best definition of the word amendment?

a.

cancellation by making invalid or outdated

c. improvement by revision or correction

Police Practice Test 5

Page 9 of 14

b. identification by comparison and elimination

d. protection by establishing rules and laws

Reading Comprehension:
Passage : Throughout its long history, China has seen the rise and fall of a large number of secret organizations. Originally, most of them had no criminal intent. They were simply brotherhoods based on shared political purposes and mutual assistance. However, some of them fell under the control of dishonest people. They then drifted into crime and ended up posing a threat to the social order. This was the case for two of the largest secret organizations of Chinese modern history, the Tsing and Hong organizations.

Many criminal organizations such as the Tsing and Hong and many others controlled brothels, opium dens, casinos and drug trafficking operations. They forced business owners to give them a percentage of their profit. They also committed more serious crimes such as abduction, the trade of women and children, and even assassinations. To achieve impunity from the law, they corrupted government and colonial officials. It was not uncommon for leaders of these organizations to occupy legitimate positions in companies or even in government agencies. They used these jobs as a front for their illegal activities. Each organization had its own system of laws and punishments. The internal law of the Hong organization had five possible punishments: capital punishment, corporal punishment, caning, degradation and banishment. A strict hierarchy existed and obedience to superiors was mandatory. New members, called apprentices, were placed under a master and were at his service. In the golden age of the Tsing and Hong organizations, a powerful leader could recruit thousands of apprentices.
28. According to the above passage, which one of the following statements is true? a. Chinese secret organizations have always threatened the social order. b. Financial assistance is encouraged in Chinese criminal organizations. c. Few criminal organizations are known in China; only two are known today. d. Some secret Chinese organizations have not turned to crime.

29. According to the previous passage, which one of the following statements is true? Criminal organizations bribed c. The trade of women and children was government employees and senior the main activity of Chinese criminal officials to avoid facing consequences organizations. of their misdeeds. b. All leaders of these organizations had d. Prostitution is one of the rare illegal legitimate positions in businesses or in businesses in which Chinese criminal the government. organizations did not participate. 30. Which one of the following statements is supported by the above passage? a. In the Hong organization, only c. In the Hong organization, the penalty apprentices had to obey their superiors. for disobedience was corporal a.

Police Practice Test 5

Page 10 of 14

punishment. b. In the Hong organization, a persons d. In the Hong organization, the leader rank could not be lowered as a form of recruited many new members. punishment.
31. This section will test your ability to use appropriate resources and strategies to achieve objectives. You demonstrate good judgement by reaching sound decisions and taking the appropriate courses of action. The term judgement is used synonymously with the concept of common sense.

You are a police officer in a small town. You are on patrol at 2:00 a.m. when you see a car leaving town and moving very fast. You turn on the lights and sirens of your police car and attempt to pull the car over. The car does not pull over and a brief high speed chase results. After a short distance, the driver loses control of the car and it skids into the ditch. Four men jump out of the car and run into the forest. Of the following, which is the best initial course of action to take? Call for backup and immediately chase c. Determine the registered owner of the the suspects into the forest. car. The following day, go to the registered car owner's home and question him about the incident. b. Call for backup and broadcast a d. Draw your gun and order the men to description of the suspects. When stop. If they do not comply, fire a backup arrives, begin the chase of the warning shot into the air suspects into the forest.
a. 32. You are a police officer in a large municipality. Your coworker and friend, Officer Grey, is in constant financial trouble as the result of loans that she obtained before joining the PD. Grey is married and has recently taken on a second job to pay her bills. You have noticed that since she has had this second job, the quality of her work has gone down considerably. Of the following, which is the best initial step to take? a. Suggest to Officer Grey that she speak c. Speak to Officer Grey's spouse to to a financial consultant. identify possible solutions to her financial difficulties. b. Lend Officer Grey some money. d. Speak to Officer Grey and tell her about your concerns. 33. At a meeting, your supervisor tells all the members on shift about a new police policy. The policy states that police officers are to wear their hats at all times while on duty in the downtown area. The rationale for the policy is that the wearing of hats projects a professional policing image and allows the public to easily identify the person as a police officer. Later that evening, you and your partner receive a call that a young female was just sexually assaulted by two males who are both well-known to yourself. The two males are to be arrested. You believe that the two male suspects are at a fountain in the downtown area that is only approachable on foot. There is a music festival being held in the area that is attracting large crowds of people. You and your partner feel that, under these circumstances, it would be inappropriate to wear your hats; the suspects would likely see you approaching and may flee. Of the following, which is the best course of action to take? a. Call your supervisor, explain the c. Wear your hat while in the area,

Police Practice Test 5

Page 11 of 14

situation, and request permission not to wear your hats. Follow the decision given. b. Proceed to the area without wearing d. your hats. Explain to your supervisor afterwards the reason for your action.
34. Logic problems:

consistent with the police policy. Do not attempt to arrest the suspects at this time as their identity is known. Wait for another time to arrest them.

You are preparing a report concerning a car accident. The following five pieces of information are to be included in the report: 1. Officer MacInnis explained that the car was heading east bound on 84th Avenue in the first lane and had a struck a young girl who was running across 84th Avenue. 2. The statement was not obtained at this time, as the driver of the car was too emotionally upset.
3. Officer Smith attended the scene of a car accident and was briefed by Officer MacInnis of what had occurred. 4. The driver was then turned over to Officer Smith for a statement.

5. Officer MacInnis then stated that he had detained the driver of the car and that he had read him his rights.
In which of the following order should the information in the report be presented?

a. 1, 5, 2, 4, 3 b. 3, 5, 1, 4, 2

c. 1, 2, 4, 3, 5 d. 3, 1, 5, 4, 2

35. You are investigating a serious accident involving five cars. The car at the front of the accident is labeled 1 and the car at the back of the accident is labeled 5. The remaining cars are labeled according to their location in the chain of cars. The information related to the accident is sketchy and somewhat disorganized. You know the following: I. The drivers involved in the accident are Paul, Kim, Jennifer, Kevin, and Sarah. II. Kevin was in one of the cars ahead of Kim III. Sarah and Jennifer were in car 1 and 5, although not necessarily in that order. Which one of the following statements must be false?

a. Kim is in a car ahead of Sarah b. Kim is in car 2

c. Paul is in car 2 d. Kim is in a car behind Paul

36. If Kevin is in car 3, which one of the following statements must be true?

a. Paul is in a car behind Kim b. Kim is in a car ahead of Sarah
37. How much is 150% of 80?

c. Sarah is in car 1 d. Paul is in a car ahead of Kevin

Police Practice Test 5

Page 12 of 14

a. 1.8 b. 5.3

c. 70.0 d. 120.0

38. Of the 300 people working at a medical facility, 14% are physicians' assistants. How many workers at the medical facility are not physicians' assistants?

a. 42 b. 86

c. 258 d. 286

39. A pump discharges water at a rate of 184 gallons per minute. Which one of the following is the number of gallons of water this pump will discharge in an hour?

a. 1,104 gallons b. 1,840 gallons

c. 2,208 gallons d. 11,040 gallons

40. An office received 2,400 license applications during the month of April, 1/5 of which were for tavern licenses. If 1/8 of the applications for tavern licenses were rejected, how many applications for tavern licenses were ACCEPTED?

a. 60 b. 180

c. 420 d. 780

41. Cincinnati had a population of three hundred sixty-four thousand, one hundred fourteen people in 1990. What would be the population of Cincinnati if the city grew by ten thousand?

a. 365,114 b. 374,114

c. 464,114

42. Round the numbers 42 and 31 to the nearest ten. Then subtract the rounded numbers to estimate the difference. Which statement is true? a. Your estimate is larger than the exact c. Your estimate is the same as the exact difference. difference.

b. Your estimate is smaller than the exact difference. 43. About 11 million people live in Ohio. About how many of them live in cities? Use the graph to answer the question.

Police Practice Test 5

Page 13 of 14

a. 8 million b. 3 million

c. 1 million

44. Navin wants to make a small walkway between the house and the garage, using red and white bricks of the same size. He wants to use 32 bricks in all, 8 red and 24 white. What fraction of the bricks will be red? a. b. c. d.

45. Look at the graph line below. The greatest difference in daily high temperatures occurred between which two consecutive days?

Police Practice Test 5

Page 14 of 14

a. Between Tuesday and Wednesday b. Between Monday and Tuesday

c. Between Friday and Saturday d. Between Sunday and Monday

46. Mr. Wright is planning to teach five self-defense classes, one class each day. On Monday, Wednesday, and Friday, he can have 20 students in each beginner’s class. On Tuesday and Thursday, he can have 10 students in each advanced class. What is the total number of students Mr. Wright can enroll in all five classes? Choose the expression that will help you solve the problem. a. b. c. d.

47. Which of the following sentences best describes a basic principle of a democracy?

A democracy is a governmental system c. in which the people have protection from aggressors. b. A democracy is a governmental system d. in which the people have equal economic opportunities.
a.

A democracy is a governmental system in which the people have the right to petition their government. A democracy is a governmental system in which the people have the power to govern.

48. While serving on a jury, a citizen is expected to a. go along with the majority to ensure a c. exercise an independent judgment case is successful. before deciding how to vote. b. consult independently with the judge d. protect the individual defendant whenever a question of law emerges. against the majority opinion. 49. Many presidents have asked Congress to pass a law giving them the line item veto. Congress has always resisted primarily because this would

give the executive branch added power c. increase the federal deficit, thus to check the legislative branch. reducing Congress’s ability to spend more money. b. furnish lobbyists with a mechanism for d. force Congress to reduce the number increasing their influence on Congress. of staff working for its members.
a. 50. Jim is arrested in a criminal case. He refuses to answer any questions about the alleged crime, claiming that he is “taking the Fifth Amendment.” “Taking the Fifth Amendment” means that a. Jim is pleading innocent to the charges c. Jim does not have to answer questions against him. that might incriminate him. d. Jim is too young to be arrested for this crime.

b. Jim has not had the opportunity to discuss the case with a lawyer.

Police Math Test

Page 1 of 13

Police Math Test
Multiple Choice

Identify the letter of the choice that best completes the statement or answers the question. You have 1 hour and 20 minutes to complete the test.

1. Graph Question A line graph is a way to summarize how two pieces of information are related and how they vary depending on one another. The numbers along a side of the line graph are called the scale.

Example 1:

The graph above shows how John's weight varied from the beginning of 1991 to the beginning of 1995. The weight scale runs vertically, while the time scale is on the horizontal axis. Following the gridlines up from the beginning of the years, we see that John's weight was 68 kg in 1991, 70 kg in 1992, 74 kg in 1994, and 73 kg in 1995. Examining the graph also tells us that John's weight increased during 1991 and 1995, stayed the same during 1991, and fell during 1994. What was John’s weight in 1993? a. 74 kg c. 77 kg b. 71 kg d. 79 kg

Police Math Test

Page 2 of 13

2. Graph question: Example 2:

This line graph shows the average value of a pickup truck versus the mileage on the truck. When the truck is new, it costs $14000. The more the truck is driven, the more its value falls according to the curve above. Its value falls $2000 the first 20000 miles it is driven. When the mileage is 80000, what is the truck's value? a. $2000 c. $8000 b. $4000 d. $1200 3. The pie chart below shows the ingredients used to make a sausage and mushroom pizza. The fraction of each ingredient by weight is shown in the pie chart below. We see that half of the pizza's weight comes from the crust. The mushrooms make up the smallest amount of the pizza by weight, since the slice corresponding to the mushrooms is smallest. Note that the sum of the decimal sizes of each slice is equal to 1 (the "whole" pizza").

The cheese and sausage makes up what percentage of the pizza?

a. 24% b. 45%

c. 53% d. 33%

Police Math Test

Page 3 of 13

4. The pie chart below shows the fractions of dogs in a dog competition in seven different groups of dog breeds. We can see from the chart that 4 times as many dogs competed in the sporting group as in the herding group. We can also see that the two most popular groups of dogs accounted for almost half of the dogs in the competition. Suppose 1000 dogs entered the competition in all. How many dogs would be in the toy group?

a. 72 b. 120

c. 144 d. 210

5. A double bar graph is similar to a regular bar graph, but gives 2 pieces of information for each item on the vertical axis, rather than just 1. The bar chart below shows the weight in kilograms of some fruit sold on two different days by a local market. This lets us compare the sales of each fruit over a 2 day period, not just the sales of one fruit compared to another. We can see that the sales of star fruit and apples stayed most nearly the same. The same amount of apples and oranges was sold on the second day.

Police Math Test

Page 4 of 13

How much did the sale of oranges increase from day 1 to day 2? a. 10 kilograms c. 20 kilograms b. 30 kilograms d. 15 kilograms 6. Refer to the below graph to answer the following questions:

A bar graph is fairly easy to read. However, before we can learn anything useful, we have to know what the graph is about (in this case "Sales") and the parameters or limits of the graph (in this case "Sales by Quarters of the year" and "Thousands of Dollars"). Simply reading the numbers at the top of each bar isn't enough. We have to give them some "flesh and blood". Putting all of our data together, we can make a meaningful statement such as: "Quarter 3 was our best quarter with sales of $4,075,000.00." We can mine other facts from this graph: What was the second best quarter? a. 1 c. 3 b. 2 d. 4 7. What was the lowest quarterly sales?

a. $2,800 b. $44,000

c. $2,800,000 d. $3,800,000

Police Math Test

Page 5 of 13

8. In what quarter did we have the worst sales? a. 1 c. 3 b. 2 d. 4

Refer to below chart:

9. What two expenditures, added together, equal what is spent on rent? a. clothes & transportation c. food & transportation b. other & transportation d. clothes & food 10. Refer to the following information: The RCMP has developed strategic partnerships with various financial institutions, both nationally and internationally, in an effort to reduce the counterfeiting of currency and credit cards. The following table reflects the amount of success they have had over a five year period.

Police Math Test

Page 6 of 13

Counterfeiting Seizure Statistics

What percentage of the total funds seized in 1994 was Canadian?

a. 3% b. 4%

c. 25% d. 33%

11. What percentage of all the seized American funds were seized in 1992?

a. 29% b. 78%

c. 80% d. 99%

12. Look at this series: 44, 44, 50, 50, 56, . . . What number should come next?

a. 44 b. 48

c. 56 d. 62

13. Which of the following multiplications is right?

a. 4 ´ 27 = 108 b. 4 ´ 34 = 146 c. 7 ´ 22 = 144
14. How much is 150% of 80?

d. 8 ´ 17 = 136 e. 5 ´ 27 = 145

a. 1.8 b. 5.3

c. 70.0 d. 120.0

15. Of the 300 people working at a medical facility, 14% are physicians' assistants. How many workers at the medical facility are not physicians' assistants?

a. 42 b. 86

c. 258 d. 286

16. A pump discharges water at a rate of 184 gallons per minute. Which one of the following is the number of gallons of water this pump will discharge in one hour?

a. 1,104 gallons b. 1,840 gallons

c. 2,208 gallons d. 11,040 gallons

Police Math Test

Page 7 of 13

17. Write .2% as a decimal

a. .2 b. .02
18. Write 3.4% as a fraction.

c. .002 d. .2

a. 34/1000 b. 34/10
19. Write 3/4% as a decimal.

c. 34/100 d. 340/100

a. .75 b. .075 c. .0075
20. Find 60% of 70. a. 420 b. 4.2 c. 1162

d. .00075 e. 75

d. 4200 e. 42

21. Reduce 20/60 to it’s lowest common denominator: a. 2/3 c. 2/6 b. 1/3 d. 1/4 22. What does 1/5 +1/3 = a. 3/4 b. 7/8 23. What does 1/5 x 2/3 = a. 2/3 b. 5/8 24. What does 3 divided by 1/4 = a. 3 b. 12

c. 5/7 d. 8/15 c. 2/15 d. 2/11 c. 21 d. 24

25. If the average of five numbers is -10, and the sum of three of the numbers is 16, then what is the average of the other two numbers?

a. -33 b. -1 c. 5

d. 20 e. 25

26. In traveling from city A to city B, John drove for 1 hour at 50 mph and for 3 hours at 60 mph. What was his average speed for the whole trip?

a. 50 b. 53 1/2 c. 55

d. 56 e. 57 1/2

Police Math Test

Page 8 of 13

27. 2/7 x 4/5 = a. 3/8 b. 7/29 28. 5/12 x 9/10 = a. 13/41 b. 6/7 29. 2 5/8 x 6 = a. 9 2/3 b. 15 3/4 30. 5 1/3 x 6 3/8 = a. 30 1/3 b. 33 31. Number-series problem: 9, 10, 12, 15, 19, ___ a. 18 b. 21 32. 43, 39, 35, 31, _____ a. 37 b. 17 33. 189, 93, 45, 21, 9, ___ a. 3 b. 18 34. 1, 4, 8, 11, 22, 25, 50, ___, ___ a. 51,103 b. 53,106 35. How many faces does this box have?

c. 9/31 d. 8/35 c. 3/8 d. 11/45 c. 12 1/2 d. 24 c. 29 1/5 d. 34

c. 13 d. 24 c. 27 d. 45 c. 5 d. 23 c. 55, 117 d. 63, 127

a. 2 b. 4 c. 6
36. Where should the number 347 be placed in this row so that the numbers will be in order from the smallest to the largest?

Police Math Test

Page 9 of 13

a.

b.

c.

37. Cincinnati had a population of three hundred sixty-four thousand, one hundred fourteen people in 1990. What would be the population of Cincinnati if the city grew by ten thousand? a. 365,114 b. 374,114 c. 464,114 38. The Ohio State Fair is held every year in Columbus. At one of the state fairs, there were 48 Girl Scouts marching in the parade. There were 6 girls in each row. Which equation would you use to find how many rows of Girl Scouts were marching in the parade? a. 48 + 6 = n b. n × 6 = 48 c. 48 − n = 6 39. Round the numbers 42 and 31 to the nearest ten. Then subtract the rounded numbers to estimate the difference. Which statement is true? a. Your estimate is larger than the exact difference. b. Your estimate is smaller than the exact difference. c. Your estimate is the same as the exact difference. 40. Jenny, Brian, and Omar were in three sports events. The chart shows how many minutes it took the children to complete each event. The winner is the person with the lowest total time in all the events. Which child won?

Swimming Bicycling Running a. Jenny b. Brian c. Omar

Jenny 4.3 minutes 6.8 minutes 5.1 minutes

Brian 5.1 minutes 7.2 minutes 4.9 minutes

Omar 3.9 minutes 7.5 minutes 5.0 minutes

41. Approximately 11 million people live in Ohio. About how many people live in cities? Use the

Police Math Test

Page 10 of 13

graph to answer the question.

a. 8 million b. 3 million c. 1 million
42. Carmen made a wood sculpture from small pieces of wood. The sculpture is 2 feet 10 inches tall. Carmen places her sculpture on a base that is 6 inches tall. How tall are the sculpture and the base together? a. 2 feet 4 inches c. 3 feet 6 inches b. 3 feet 4 inches d. 8 feet 10 inches 43. What is 30% of $48.00? a. $14.40 b. $16.00

c. $18.00 d. $33.60

44. Which of the following pairs of lines appears to be perpendicular? a. c.

b.

d.

Police Math Test

Page 11 of 13

45. There are 4 boys to every 6 girls in Mr. Simpson’s class. If there are 30 students in the class, how many of those students are girls? a. 10 c. 18 b. 12 d. 20 46. In the picture below of part of a ruler, what does each small division represent?

a. b.

c.

inch inch

d.

inch inch

47. Neil’s club held a fund-raising drive. During the first week, the club received the donations shown below from five members.

$8.00 $10.00 $10.00 $22.00 $25.00
What was the average amount donated by the members during the first week? a. $15.00 c. $25.00 b. $17.00 d. $75.00

48. During their last vacation, a family took a train trip. They traveled 594 miles in 9 hours. What was the train’s average speed? a. 56 mph c. 66 mph b. 60 mph d. 585 mph 49. Ernest paid for a book with a $20 bill and received one dollar, one quarter, two dimes, and four pennies in change. How much did he pay for the book? a. $18.49 c. $19.49 b. $18.51 d. $19.51 50. Coach Newman has been keeping a record of the temperature in his classroom in the chart below.

Police Math Test

Page 12 of 13

How many hours did the temperature rise only one degree? a. none c. 2 b. 1 d. 3 51. If one bench can seat 8 people, what is the fewest number of benches needed to seat 42 people? a. 4 c. 6 b. 5 d. 7 52. A container holds a. 3 quarts. gallons when full. This amount can also be written as c.

b. 12 pints.

quarts. d. 15 pints.

53. Mike and Treecie are planning a party. If they plan to make 30 hamburgers for 25 people, what is the least number of hamburgers they will need if 40 people show up? a. 30 c. 55 b. 48 d. 65 54. Which is the correct order to show capacity from smallest to largest? a. pint, cup, quart, gallon c. pint, cup, gallon, quart b. cup, pint, gallon, quart d. cup, pint, quart, gallon 55. Which of the following is a correct statement? a. 0.02 > 20% c. 20% < 0.02 b. 0.02 = 20% d. 20% > 0.02

Police Math Test

Page 13 of 13

Police Grammar

Page 1 of 7

Police Grammar/ Vocabulary/Spelling Test
Multiple Choice

Identify the letter of the choice that best completes the statement or answers the question. You have 2 hours and 20 minutes to complete the test.

1. Choose the word or phrase that has the same meaning as the underlined word. The witness corroborated the suspect's story. a. verified c. added to b. contradicted d. questioned

2. Choose the word or phrase that has the same meaning as the underlined word. It was not a very pragmatic plan.

a. plausible b. serious

c. practical d. sensible

3. Choose the word or phrase that has the same meaning as the underlined word. The police sequestered the suspect.

a. caught b. isolated

c. arrested d. released

4. Choose the word or phrase that has the same meaning as the underlined word. In order to conceal her guilt, Linda told a blatant lie. a. harmless c. careless b. subtle d. obvious

5. Choose the word or phrase that has the same meaning as the underlined word.

Police Grammar

Page 2 of 7

He was ignorant of the proper procedures.

a. unaware b. conscious

c. uncertain d. cognizant

6. Choose the sentence that represents the best English Usage. a. We seldom ever receive this type of c. Each of these regulations apply to request anymore. your case. b. Neither of the employees are doing d. I have enclosed a copy of the file what is expected of him. you requested. 7. Choose the sentence that represents the best English Usage. a. Of the two runners, John is the worst. c. John is the worst of the two runners. b. Of the two runners, John is the better. d. John is the best of the two runners. 8. Which of the following words can be defined as: The process of deriving general principles from particular instances?

a. concoction b. deduction

c. induction d. reduction

9. Which one, if any, of the following sentences contains a misspelled word? a. The discussion of budgetary issues took up most of the time. b. This is not a frequently occurring situation. c. They are unable to give us a definate answer. d. None contains a misspelled word.

10. Which one of the following is grammatically INCORRECT? a. Only one of the software packages suits our needs. b. Are their information correct? 11. Choose the correctly spelled word:: c. The number of applications has increased this year. d. Enclosed are copies for every committee member.

a. confidance b. confidence
12. Choose the correctly spelled word::

c. confedense d. none of these

a. disappoint b. dissapoint
13. Choose the correctly spelled word::

c. diseppoint d. none of these

a. defendent b. deffendent

c. diffendant d. none of these

Police Grammar

Page 3 of 7

14. Choose the correctly spelled word::

a. amung b. among
15. Choose the correctly spelled word::

c. ammung d. none of these

a. occassionnally b. occassionally
16. Choose the correctly spelled word::

c. occasionally d. none of these

a. calendar b. calender
17. Choose the correctly spelled word::

c. calander d. none of these

a. merchendice b. merchandice
18. Choose the correctly spelled word::

c. merchendise d. none of these

a. competant b. compitent
19. Choose the correctly spelled word::

c. compettent d. none of these

a. apparant b. apperant
20. Choose the correctly spelled word::

c. apparrant d. none of these

a. insistant b. incistant
21. Choose the correctly spelled word::

c. insistent d. none of these

a. reference b. refferance
22. Choose the correctly spelled word::

c. referance d. none of these

a. separete b. separate
23. Choose the correctly spelled word::

c. seperate d. none of these

a. truely b. truley 24. Choose the correctly spelled word::

c. truly d. none of these

Police Grammar

Page 4 of 7

a. begining b. beginning
25. Choose the correctly spelled word::

c. beggining d. none of these

a. preffered b. preferred
26. Choose the correctly spelled word::

c. prefferd d. none of these

a. memorandem b. memorandom
27. Choose the correctly spelled word::

c. memorandum d. none of these

a. posibility b. possability
28. Choose the correctly spelled word::

c. possebility d. none of these

a. probibility b. probebility
29. Choose the correctly spelled word::

c. probability d. none of these

a. prominent b. prominant
30. Choose the correctly spelled word::

c. promenant d. none of these

a. judgemant b. judgment
31. Choose the correct word or phrase. I didn't expect John to be out." "He _______________ shopping.

c. judgemint d. none of these

a. must be gone b. should go
32. Why isn't she at home?" "She _______________ the bus.

c. ought to have gone d. must have gone

a. could miss b. could have missed

c. might miss d. can have missed

33. To travel in the European Community you _______________ a passport.

a. haven't got

c. have need

Police Grammar

Page 5 of 7

b. don't need

d. mustn't have

34. When we arrived, the play _______________ .

a. has been started b. had been started

c. had started d. would have started

35. If you _______________ wet in the rain, you wouldn't have caught a cold.

a. hadn't got b. wouldn't have got
36. I asked her if _______________.

c. haven't got d. wouldn't get

a. liked she the candy b. did she like the candy

c. likes she the candy d. she liked the candy

37. I'll wash the dishes, if you _______________ the living room.

a. will clean b. clean

c. have cleaned d. would clean

38. What does your suitcase _______________?" "It's red and has a black handle.

a. like? b. look like?

c. seem to be? d. similar to?

39. Who _______________ the room? They only have done two walls and they left the paint open.

a. paints b. will paint

c. painted d. has been painting

40. Please _______________ my mother when we get back home.

a. tell to me to call b. remind to me to call
41. Were you able to get the discount? " "No, but I wish I _______________."

c. remember me to call d. remind me to call

a. might b. will

c. could have d. may have

42. I'm always too busy. If I _______________ the time, I would attend the conference.

a. had had b. have

c. had been d. had

43. What happened to your computer?" "It's _______________ for at least a week now.

Police Grammar

Page 6 of 7

a. broke b. broken

c. been broken d. broked

44. I think it is time for you to _______________ your own living.

a. win b. maintain

c. gain d. earn

45. Which one of the following words is misspelled?

a. manageable b. circumstants

c. legality d. None of the above is misspelled

46. Of the following, which one should be THIRD in an alphabetic file?

a. Docker, John b. Decker, Jane
47. Paragraph Organization

c. Dinckman, June d. Dockman, James

The question below is based upon the following group of scrambled sentences. You are to read the sentences and then indicate which order of presentation would represent their best arrangement. 1. Eventually, they piece all of this information together, and make a choice. 2. Before actually deciding upon a corrections job, people usually think about several possibilities.

3. They imagine themselves in different situations, and in so doing, they probably think about their interests, goals, and abilities.
4. Choosing among occupations in the correctional services field is an important decision to make. QUESTION: Which one of the following is the best arrangement of these sentences?

a. 2-3-1-4 b. 2-3-4-1

c. 4-2-1-3 d. 4-2-3-1

48. Correction Officer Gilson failed to take proper precautions. His failure to take proper precautions caused a personal injury accident. QUESTION: Which one of the following best presents the information above? a. Correction Officer Gilson failed to c. Correction Officer Gilson's failure to take proper precautions that caused a take proper precautions caused a personal injury accident. personal injury accident. b. Proper precautions, which Correction d. Correction Officer Gilson, who

Police Grammar

Page 7 of 7

Officer Gilson failed to take, caused a personal injury accident.
49. Information Presentation:

failed to take proper precautions, was in a personal injury accident.

Martin Wilson failed to take proper precautions. His failure to take proper precautions caused a personal injury accident. QUESTION: Which one of the following best presents the information above? a. Martin Wilson failed to take proper precautions that caused a personal injury accident. b. Proper precautions, which Martin Wilson failed to take, caused a personal injury accident. c. Martin Wilson’s failure to take proper precautions caused a personal injury accident. d. Martin Wilson, who failed to take proper precautions, was in a personal injury accident.

50. Select the space where the underlined name or number should be placed to be in correct order.

a. This location b. This location c. This location

d. This location e. This location

Reading Comprehension Test

Page 1 of 8

Reading Comprehension Test
Multiple Choice

Identify the letter of the choice that best completes the statement or answers the question.
You have 30 minutes to complete the test.

Reading Comprehension
Based on the information provided for you in the passage below, you are to answer the question that follows. The answer is taken directly from the passage. Thus, if you have trouble determining the correct answer, look back into the paragraph for clues.

Passage:
Although the detectives do not have a suspect, evidence from the crime scene suggests several possibilities. The direct route of the burglar indicates that the perpetrator most likely knew the inside of the home fairly well. He passed up the camera and VCR in the living room and the Gucci tennis bracelet and silver jewelry lying on the dresser in the bedroom. No steps were wasted. The fact that the burglar was able to deactivate the elaborate alarm system suggests that the burglar may be a career criminal. The fact that the burglar took only the real emeralds and left the “paste” seems to indicate that he or she is a gemstone expert. The Franklin Insurance Company (FIC) insured the emeralds for $100,000. They were taken from the safe that is hidden behind a picture in the bathroom. 1. Which of the following factors indicate(s) that the burglar knew the inside of the home fairly well? a. The fact that the burglar was able to d. The fact that the detectives do not deactivate the elaborate alarm system. yet have a suspect. b. The fact that the burglar took a direct e. Both a and c. route to the emeralds. c. The fact that the burglar took only the real emeralds.

Passage:

Reading Comprehension Test

Page 2 of 8

Too many consumers believe in the quick fix, the easy solution, the magic ingredient, the miracle cure. And, of course, there are corporations ready to provide them with the product which will solve their health care problems. An independent review board is needed to interpret the nutritional data available. If such a review procedure is not adopted, then our society is left to the whims of the private sector whose vested interests and public pronouncements on very specific products already seriously compromises public understanding of these health issues. The nutrition information Americans are getting may be fragmentary, contradictory and confusing, when it’s not outright incorrect, according to nutritional biochemist T. Cohn Campbell. Campbell, who has been doing nutrition research for the past 25 years, is one of the co-authors of the National Academy of Sciences’ report on “Diet, Nutrition and Cancer,” which recommended increased consumption of fruits, vegetables and whole-grain cereals. He says that confusion about nutritional guidelines and chronic disease risk is directly attributable to the use of scientific data which are much too narrowly focused for policy and marketplace purposes. An extensive nutritional study conducted in China recently provides some examples of the inconsistencies that Campbell is concerned about. Some U.S. studies associate low cholesterol levels in the blood with increased risk of colon cancer. In China, however, incidences of colon cancer were low among those who had low cholesterol levels, a finding much more consistent with the recognized beneficial effects of low cholesterol levels. Such an observation provides further evidence that the U.S. data showing an increase in colon cancer at lower cholesterol levels is an experimental fluke. Another example, dieting is the way to lose weight in the States. We count calories, but obviously have limited success with this method since so many of us are constantly dieting and continually overweight. Chinese people consume about 20 percent more calories than Americans, yet there is very little obesity amongst the Chinese, and they are healthier in general than Americans. This finding tells us that excess caloric intake does not necessarily cause excess body weight nor is it likely to be the determinant of chronic disease risk. One difference between the diets is the proportion of fat in calories consumed. Chinese fat consumption ranges from 6 to 25 percent, with an average of about 15 percent of daily calories from fat. This is well below the 30 percent usually recommended in American diets. So, the current wisdom which says that people cannot subsist on diets with fat intakes much lower than 30 percent of calories is seriously called into question by the Chinese study. 2. Which of the statements given below is best supported by the passage above? a. The Chinese suffer from less obesity c. The existence of an independent because they have a more active lifereview board keeps corporations style. from peddling products whose effectiveness is questionable. b. The average person in China d. Confusion about nutritional consumes half the fat of the average guidelines in the U.S. is due to the American. improper use of scientific research.

Passage
"Increasingly, behavior termed ' rage' being viewed as a public health issue, because of the road is number of deaths and injuries related to it. Such behavior is often a reaction to feeling one has been treated unfairly by another driver, and is much less likely to occur if a driver is treated fairly. ' Fair play' the road includes the observance not only of traffic regulations but also of the rules of on

Reading Comprehension Test

Page 3 of 8

courtesy. Courteous driving is based on common sense consideration for other drivers and a strong desire to make the roads safe for everyone. Good highway manners should become just as much a matter of habit as other kinds of manners." 3. Which one of the following statements is best supported by the above selection? a. Being courteous when driving is c. more important than observing traffic regulations. b. Courteous driving contributes to road d. safety. Those who are generally polite are also courteous drivers. Unlike driving courtesy, the observance of traffic regulations is a matter of habit.

Passage:
Transporting Inmates: Correction officers who are transporting inmates are responsible for the safety and security of such inmates from the time they leave the jail until the inmates are delivered to their estination or until the inmates are returned to the jail. The officers should not make any stops during the trip unless it is absolutely necessary. If stopping is absolutely necessary, the officers must take all possible precautions to maintain their own safety and the safety and security of the inmates. If an unavoidable delay is experienced (for example heavy traffic, car trouble, or bad weather) the officers must take the first opportunity to notify the jail of the delay. If a situation occurs which is not covered by specific instructions, the officers must contact their jail' watch commander if it is s possible. If it is not possible to contact the watch commander, the officers must exercise their best judgement to fulfill their responsibility to maintain the safety and security of the inmates." 4. Which one of the following statements concerning the transportation of inmates is best supported by the passage above? a. If it appears that the trip might be c. The officers transporting the inmates delayed, the officers transporting the are responsible for maintaining the inmates should notify the jail safety and security of the inmates at immediately. all times during the trip. b. If it is necessary to make a stop, the d. If the vehicle transporting the inmates should be secured with legofficers and the inmates becomes irons and handcuffs. involved in a traffic accident, the first thing the officers should do is notify their watch commander.

Passage
"Oxydizing materials shall include a substance that yields oxygen readily to stimulate the combustion of organic matter. Oxidizing materials shall be stored in cool, dry, ventilated locations and separated from stored organic materials. Oxidizing materials shall be stored separately from flammable liquids, flammable solids, combustible materials, hazardous chemicals, corrosive liquids and such other noncompatible materials. Bulk oxidizing materials shall not be stored on or against wooden surfaces."

Reading Comprehension Test

Page 4 of 8

5. Which one of the following statements is correct according to the above passage? a. Oxidizing materials should be stored c. Oxidizing materials should be stored away from organic materials. with flammable solids. b. Organic matter should be stored on d. Corrosive liquids should be stored wooden piles. with hazardous

Reading Comprehension Passage:
Mass markets are dead. It’s time to let go of the old mass marketing strategies – or so it would seem. Markets are subdividing like crazy, creating a kaleidoscope of goods and services. Every day a new product, or a new variation on an old product, appears. There’s Shredded Wheat, and MiniWheats, and Frosted Mini-Wheats. There’s diet Coke and Classic Coke, Cherry Coke and decaffeinated Coke. It’s a global phenomenon. In the urban centers (at the very least) of each nation, countless brands of soaps and soups, jeans and jerseys, soft drinks and softballs can be found, each promising to be better or different. But lest we be too hasty, perhaps we should look at this more closely. Have the markets really been transformed into a myriad of unrelated niches, creating the need for a multitude of ad campaigns? Is it all change and no consistency? Are there no theories or traditions to make sense out of this? In reality, large markets have always subdivided into smaller specialty ones. The main difference is that today they subdivide at a phenomenal rate. Market division is happening faster and faster, and the new segments show up everywhere, almost simultaneously. But who are the customers? In all product categories and places, people increasingly occupy many, and often disparate, segments and opt for many different brands. It’s no longer brand loyalty that holds sway, and you can’t use the old stereotypical guidelines. It doesn’t hold that customers who buy herbal tea prefer cotton t-shirts and brown rice. It’s coffee and a bagel in the morning, sushi for dinner, and herbal tea before bedtime. In this world of rapid transit and instantaneous communication, people everywhere are becoming more alike in their tastes and preferences. And one thing they certainly share is the desire for ever-present, everchanging variety. Companies who continue to have a line of items specifically for clearly defined preference groups have certainly failed to accurately assess the marketplace. 6. Which of the following statements is best supported by the passage? a. The existence of so many products prevents successful marketing strategies. b. It’s not that there are no more mass markets; it’s that the nature of the mass has changed. c. Ad campaigns need to be more varied and creative to keep the attention of the new consumers. d. People’s tastes are very ethnic.

Passage:
According to Hume, all of life and its experiences is merely a passing fancy with nothing tying it

Reading Comprehension Test

Page 5 of 8

together. There is no order, no organization, merely a dizzying array of fantastic and bewildering images. Kant revolutionized Western philosophy by proving that true experience consists of the judgments we impose upon the data of the senses. The senses yield information about the world, but its understanding which gives the world its true and proper form. Understanding applies its own laws on the sense experiences and transforms them into a coherent and consistent body of knowledge. Kant called these laws categories, and said that they are intrinsic to the mind. The mind intuits them; they are basic to the mind. For Kant, space and time are forms of intuition and man can only experience his world within a spacio-temporal frame. Nothing can be known outside of this framework. Some of the following statements are true according to the passage, some are false. 7. Some of the following statements are true according to the passage, some are false. I. According to Kant, categories are basic to the mind. II. Kant revolutionized Eastern philosophy. III. True experience comes directly from the data of the senses, according to Kant. IV. According to Kant, man can only experience his world within a framework of time and space.

a. Statements I and III are true. b. Statements I and IV are true.

c. Statements II and IV are true. d. Statements I, II, and IV are true.

Passage:
With the coming of the VCR and the camcorder, personal and affordable television production became a reality. Yet the same technological revolution that allowed the amateur to produce near studio-quality productions has given the television industry an amazing array of special effects. Through advances in computer technology, we are treated to such wonders as digital representation of athletic events, photos of athletes spinning into view, screens split any which way, and logos exploding into fireworks. And, of course, versions of this same technology are starting to be available in the home. Called desktop video, because all the components can fit on a standard desk or table, this system uses the computer to create and the VCR to print. They offer titling, computer graphics, animation, digital video effects, and the ability to mix live video with computer pictures. What' needed is a computer to generate and manipulate images, a camera to originate pictures, a s VCR or camcorder to use as a source for editing and overdubbing, another VCR to edit and rerecord and finally the right software to make it all happen. Systems also take advantage of a variety of components like: character generators to create electronic text in a variety of styles and colors; video titling programs which mix words with pictures, animate images, and move from screen to screen; painting systems that let you draw, paint, or manipulate pictures; animation Systems that create moving images in two or three dimensions using realistic, shaded objects; and digital video effects hardware which let images and words roll, tumble, and dance around the screen. 8. According to the above passage, a. the technological revolution has c. digital video effects hardware lets allowed the amateur to produce images and words roll around the studio-quality productions. screen. b. video titling systems allow you to d. a camera may be used for editing create electronic text and mix words and overdubbing.

Reading Comprehension Test

Page 6 of 8

with pictures.

Passage:
Nearly twenty years ago, the United Farm Workers boycott of grapes was successful in eliminating deadly poisons like DDT, DDE, and Dieldrin from fields worked by union workers. Yet today more than fifty other chemical products used on table grapes have been identified. The most lethal poisons are listed below. Each has injured or killed farm workers, and each might be present on the grapes you buy. There is methyl bromide, responsible for more occupationally-related deaths than any other pesticide and for permanent brain damage or blindness. Parathion and Phosdrin also can kill, sometimes very quickly. Since they are sprayed from the air, they also negatively affect people in the surrounding areas. (Ninety percent of all aerially sprayed pesticides miss their targets.) Dinoseb has a cumulative effect, causing illness and blindness. Finally, there is Captan which can cause cancer, birth defects, and changes in cell structure. This compound is the one most frequently found on storebought grapes.
Federal and state agencies have recommended the ban of each of these substances yet they continue to be used today (1988). Pesticide poisoning has doubled in the past ten years: more than 300,000 farm workers are made ill every year through pesticide exposure, and female farm workers are seven times more likely to suffer miscarriages than the average American woman. Of course, farm workers are not the only ones affected by the massive amounts of pesticides dumped on the nations food supply. A survey done in San Francisco showed that 44 percent of fruits and vegetables contained measurable amounts of pesticide residues and 42 percent of those contained more than one chemical. Pesticides are thought to be responsible for groundwater contamination in 23 states groundwater provides 50 percent of our drinking water supply. 9. All of the following are true according to the above passage, except; a. methyl bromide is responsible for c. more than forty chemical products more occupationally-related fatalities used on table grapes have been than any other pesticide. identified. b. United Farm Workers have called for d. Captan can cause cancer, birth another nation-wide boycott of table defects, and changes in cells. grapes.

Passage:
In spite of the fact that very few organizations keep accurate records of employee exit interviews, we can make some generalizations about why women leave and how long they stay. A small proportion of women leave their jobs after working less than a year. Usually, they leave because they dislike the work, their co-workers or supervisors, or because there has been a change in their personal lives. Another small percentage leave after ten to fifteen years, and they tend to leave for personal reasons or because it would significantly advance their careers. Women who leave after working between three and five years are likely to do so because they feel blocked in their careers. Many trainee jobs are merely interesting, moderate-paying jobs without much hope for

Reading Comprehension Test

Page 7 of 8

advancement. Young women find the jobs attractive when they are first starting out, but become disappointed over time because they are looking for more and are qualified for more. Often women leave jobs that they like and are good at because they watch others advance more rapidly simply because they are men. 10. Which of the following statements is best supported by the passage? a. Most young women find their first c. jobs attractive if they lead to more advanced positions. b. Women become frustrated when they d. see men advancing rapidly. Organizations should keep accurate exit interviews. Many trainee positions do not lead very far on the career ladder.

Passage:
Most of us assume that we and other human beings are basically rational. Being rational means that once we figure out what we want to happen, we develop a theory or plan to bring it about, act in such a way as to foster the plan, seek to correct any action which interferes with our desired result, and feel good or badly depending on our degree of success. This concept of rationality assumes that people plan their actions and are therefore personally responsible for them. It also assumes that people would not act against their intentions. In addition, it assumes that people do their utmost to control their own ability to put their actions and desires in motion, rather than being pawns in someone else’s game or victims of fate.
Given these assumptions, it’s understandable that, when confronted with their own irrationality, most people become rather upset. They become frightened when they realize that their actions are countering their intentions and that they have been unaware of this fact. If they can’t trust themselves, whom can they trust? In this rational society, where concrete facts and the scientific method are worshipped, the only logical conclusion anyone can come to is that there is something wrong with herself. (Such thinking might run something like this: “If I want one thing, but do something that works against that outcome, there must be something wrong with my thinking, or my ability to plan and assess my actions.”) Therefore, people become confused, bewildered, ashamed, frustrated, guilty, and angry when they realize that they were unable to plan and put into effect what they intended to, that they were predisposed to sabotage their plans, and that others were aware of what they were doing all along. It would be far better for most of us if we realized that we very often do things for reasons that make no sense to our intellect. If we did, we would spend far less time defending our actions, and we would be able to spend more time figuring out how to get where we want to go and what’s really stopping us. 11. Which of the following is best supported by the passage? a. Most of the time people act in ways c. If a person wants one thing but does that are consistent with their another, there is something wrong intentions. with him or her. b. It is sometimes easier for someone d. People do their utmost to control else to see that we are working their own ability to put their desires against ourselves than it is for us to in motion. see it.

Reading Comprehension Test

Page 8 of 8

PATI TEST 1

PATI (Police Analytical Thinking Inventory)Test
Multiple Choice Identify the letter of the choice that best completes the statement or answers the question. To check your score fill in your name on the top and click the “check your results” button on the bottom.

You have 1 hour and 10 minutes to complete the test.

Situation: Nine colleges (O, P, Q, R, S, T, U, V, W) provide staff for three law firms: B, C,and D. Each college provides exactly one law student who is employed by only one of the three law firms. Law firm B cannot employ a law student from O, P, or Q. Law firm C cannot employ a law student from R, S, or T. Law firm D cannot employ a law student from U, V, or W. Law firm B must have exactly one more law student than law firm C. 1. If U and O are the only law students employed by law firm C, how many of the nine law students must be employed by Law firm D? a. 3 b. 4 c. 5 d. 6 e. 7

PATI TEST 1

2. If no law students are employed by Law firm D, which college must be represented in Law firm B? a. O b. P c. R d. U e. V 3. What is the greatest number of law students that could be employed by law firm D? a. 9 b. 8 c. 7 d. 6 e. 5 4. Which colleges must provide staff for Law firm B, if W is the only one of the nine schools providing a law student for Law firm C? a. R and V b. S and T c. S and U d. T and V e. U and V 5. If Law firm C has law students from only U, V, and Q, the law students employed by Law firm D must be: a. O and P b. O and R c. P and S d. P and T e. Q and S

PATI TEST 1

Situation: Five executives of a European corporation are in Madrid for a conference. Mr A can speak Spanish and Italian Mrs B can speak Spanish and English Miss C can speak English and Italian Ms D can speak French and Spanish Rev. E from Italy, can also speak French 6. Which of the following can act as interpreter when Miss C and Ms D wish to confer? a. Only Mr A b. Only Mrs B c. Only Rev E d. Mr A or Mrs B e. Any of the other three executives. 7. If a sixth executive is brought in, to be understood by the maximum number of the original five, he should be fluent in a. English and French b. Italian and English c. French and Italian d. Italian and Spanish e. English and Spanish Situation: There are 7 flags arranged in a row. The flags are all single-color, solid color flags. The flags are red, white, green, and blue. There are exactly 2 red flags. At least one color has exactly 3 flags. At least one color has exactly 1 flag. A blue flag must not be next to another blue flag. A green flag must always be next to another green flag. A white flag must be in the middle of the row.

PATI TEST 1

8. If the blue flag can only be one flag away from a green flag, what position could a red flag be? a. b. c. d. e. Position #3 Position #4 Position #5 Position #7 Position #4 or #5

9. What arrangement follows the conditions above and puts the most flags next to a differently colored flag? a. b. c. d. e. G,G,G,W,B,R,R B,R,R,W,G,G,G R,G,B,W,B,G,R R,B,R,W,G,G,G R,W,B,G,G,G,R

Situation: The campers in a summer camp are broken up into five groups: the Nightingales, the Owls, the Peacocks, the Swallows, and the Turtle Doves. Each summer, three of the five groups participates in the mid-summer talent show. The rules specifying the order of participation of the groups in the show are as follows: Each group must participate at least once in any two consecutive years. No group participates for three consecutive years. Participate takes place in cycles, with each cycle ending when each of the five groups have participated three times. Only then does a new cycle begin. No group participates more than three times within any cycle. 10. If the groups participating in the first year of a given cycle are the Nightingales, the Owls and the Peacocks, which of the following could be the groups participating the second year of the cycle? a. b. c. d. e. Nightingales, Owls, Swallows Nightingales, Owls, Turtle Doves Nightingales, Peacocks, Turtle Doves Owls, Peacocks, Turtle Doves Owls, Swallows, Turtle Doves

PATI TEST 1

11. Any cycle for the groups' participation in the talent shows must be completed at the end of exact how many years? a. b. c. d. e. five six seven eight nine

12. Which of the following must be true about the three groups that participate in the talent show in the first year? a. b. c. d. e. At most two of them participate together in the third year At least two of them participate together in the second year All three of them participate together in the fourth year All three of them participate together in the fifth year None of them participate in the third year

13. If in a particular cycle, the Nightingales, Owls and Swallows participate in the talent show in the first year, which of the following must be true? a. b. c. d. e. Nightingales participate in the second and third years Owls participate in the third and fourth years Nightingales and Owls participate in the third year Peacocks and Turtle Doves both participate in the fifth year Swallows and Turtle Doves both participate in the fifth year

14. If in a particular cycle, the Nightingales, Owls and Turtle Doves participate in the first year and if Owls and Peacocks participate in the fourth year, any of the following could be a clan that participates in the third year except: a. b. c. d. e. Nightingales, Owls, Peacocks Nightingales, Owls, Swallows Nightingales, Peacocks, Swallows Owls, Peacocks, Swallows Peacocks, Swallows, Turtle Doves

PATI TEST 1

15. One form of reasoning holds that by eliminating all possible explanations until only one remains, that one should be accepted. Critics argue that the flaw in this form of reasoning is that one cannot know about all possible explanations. Which of the following examples best supports this criticism? a. b. c. d. e. the possible causes of heart disease the possible results of rolling dice the possible family members who left the house unlocked the possible candidates running for mayor of Atlanta, Georgia the possible countries with nuclear weapons

16. Doctor: The law of genetics holds that if both parents have brown eyes, then they can have only brown-eyed children. Patient: That is not true; my mother has blue eyes, and I have brown eyes. The patient has misinterpreted the doctor's statement to mean that a. b. c. d. only brown-eyed people can have blue-eyed children brown-eyed people cannot have blue-eyed children people with blue eyes invariably have blue-eyed children parents with the same eye color have children with a different eye color e. parents with different eye color have children with the same eye color 17. Certain similarities between prehistoric art and the art of children has led some people to the mistaken conclusion that either early humans had the mentality of children or that they were as unskilled as children. These conclusions assume which of the following? a. Art that is considered sophisticated today must always have been considered sophisticated. b. What is easy for humans today must always have been easy. c. The significance of art is consistent over time. d. Prehistoric humans painted in the same way that children now paint. e. Modem humans have learned from prehistoric man.

PATI TEST 1

18. During the cultural revolution in China under Chairman Mao, thousands of "enemies of the republic" were killed. When Mao's critics accused him of confusing his personal enemies with enemies of the republic, he responded, "I deny the accusation, and the proof is that you are still alive." Which of the following assumptions was Mao making? a. b. c. d. e. All the enemies of the republic are dead. His critics are his personal enemies. Some personal enemies are also enemies of the republic. Enemies of the republic are not personal critics. Those killed were personal enemies.

19. Harry typically vacations in Tahoe. Two years ago, Harry spent his vacation in Madrid. In relation to the first sentence, what does the second sentence do? a. b. c. d. e. It clarifies an assumption. It notes an exception. It adds emphasis. It draws a conclusion. It makes a comparison.

20. No one who has a sore throat need consult a doctor, because sore throats will recover without medical intervention. In recent years several cases of epiglottitis have occurred. Epiglottitis is a condition that begins with a sore throat and deteriorates rapidly in such a way that the throat becomes quite swollen, thus restricting breathing. Sometimes the only way to save a patient’s life in these circumstances is to insert a plastic tube into the throat below the blockage so that the patient can breathe. It is highly advisable in such cases that sufferers seek medical attention when the first symptoms occur, that is, before the condition deteriorates. Which one of the following is the best statement of the flaw in the argument? a. The author draws a general conclusion on the basis of evidence to a particular instance. b. The author assumes that similar effects must have similar causes.

PATI TEST 1

c. The author uses a medical term, “epiglottitis,” and does not clarify its meaning. d. The author makes two claims that contradict each other. e. The author bases her conclusion at the end of the passage on inadequate evidence. 21. The end of overcrowding at colleges and universities provides them with the opportunity to improve the quality of the educational services they offer. As enrollment declines, services and campus facilities should better serve student needs. If true, which of the following statements most weakens the above conclusion? a. The quality of educational services does not depend on the variety of services offered. b. Fees paid by students are the major source of funding for educational services. c. Educational services are a critical factor in a student's choice of school. d. As campus facilities grow older, their maintenance becomes more expensive. e. Student needs are different than they were when colleges and universities were overcrowded. 22. When pregnant laboratory rats are given caffeine equivalent to the amount a human would consume by drinking six cups of coffee per day, an increase in the incidence of birth defects results. When asked if the government would require warming labels on products containing caffeine, a spokesperson stated that it would not, because if the finding of these studies were to be refuted in the future, the government t would lose credibility. Which of the following is most strongly suggested by the government's statement above? a. b. c. d. e. A warning that applies to a small population is inappropriate. Very few people drink as many as six cups of coffee a day. There are doubts about the conclusive nature of studies on animals. Studies on rats provide little data about human birth defects. The seriousness of birth defects involving caffeine is not clear.

PATI TEST 1

23. Sam: Olive oil can help prevent heart attacks, according to physicians. Betty: It cannot. My mother cooked with olive oil her entire life, and she died of a heart attack last year. Betty's statement can best be countered by pointing out that a. b. c. d. Betty's mother was an exception other factors could have nullified the influence of the olive oil Betty does not know that her mother always cooked with olive oil It has never been scientifically proven that olive oil causes heart attacks e. Betty's mother might have used olive oil irregularly 24. Teresa has missed the last three practices of the dance step. She cannot perform the maneuver. In relation to the first sentence, what does the second sentence do? a. b. c. d. e. It states a consequence. It suggests a cause. It offers proof. It limits a preceding idea. It adds emphasis.

25. A study has shown that there are still millions of people who are unaware that they endanger their health by smoking cigarettes. This is so despite government campaigns to warn people of the dangers of smoking. Reluctantly, one has to draw the conclusion that the mandatory warnings that tobacco companies are required to print have had no effect. Which one of the following, if true, would refute the argument in the passage? a. Many people who continue to smoke are aware of the dangers of smoking. b. Some people smoke cigarettes for legitimate reasons. c. Government has had to force companies to warn potential customers of the dangers of their products. d. Some people who are aware of the dangers of smoking were made aware of them by the mandatory warnings.

PATI TEST 1

e. Smoking is clearly responsible for a substantial proportion of preventable illness in the country.

Police 25 Question Practice Test

Page 1 of 18

Police 25 Question Practice Test 1 Answer Key

Multiple Choice Identify the letter of the choice that best completes the statement or answers the question. To check your score fill in name on the top and click the “check your results” button on the bottom. You have 50 minutes to complete the test. Reading Comprehension

1.

According to the rule: Officer Burton should... a. b. c. d. ANSWER: D

Police 25 Question Practice Test

Page 2 of 18

NOTES: REF:
2.

The first face, on the left, is a sketch of an alleged criminal based on witnesses' descriptions at the c scene. One of the four sketches to the right is the way the suspect looked after changing his/her appearance. Assume that NO surgery has been conducted on the suspect. Select the face to the righ is most likely to be that of the suspect on the left:

a. b.

Mug shot A Mug shot B

c. Mug shot C d. Mug shot D

ANSWER: D

NOTES: REF:
3.

The correct answer is choice D. All critical features match the features of the suspect. In addition, all other answer choices have at least one critical difference. Choice A is incorrect because the lips are too small AND the eyebrows are too bushy. Choice B is incorrect because the lips are too small, the eyebrows are too bushy, AND the no too short (or high on the face). Choice C is incorrect because the chin is rounded AND the eyebrows are too bushy.

Answer this question on the basis of the following sketches. The first face that appears on the left is sketch of an alleged criminal, based on witnesses' descriptions at the crime scene. One of the four sketches to the right is the way the suspect looked after changing his/her appearance. Assume that surgery has been conducted on the suspect. Select the face that is most likely to be the suspect.

Ans A a. Mug shot A b. Mug shot B ANSWER: A

c. Mug shot C d. Mug shot D

Police 25 Question Practice Test

Page 3 of 18

The correct answer is choice A. All critical features match the features of the suspect. In addition, all other answer choices have at least one critical difference. Choice B is incorrect because the nose is too high on the face (resulting in too much space between the nose and mouth) AND the features on the neck are different. Choice C is incorrect because the lips are not only bigger, but also shaped differently AND there are two distinct lines between the upper lip and the bottom of the nose. Choice D is incorrect because both the ears AND the nose are shaped differently than the suspect's ears and nose.

NOTES: REF:
4.

Answer this question on the basis of the following sketches. The first vehicle, which appears on the a sketch of a vehicle used in a recent crime, based on witnesses' descriptions at the crime scene. On the four sketches to the right is the way the vehicle looked after its appearance was changed. Assum bodywork was performed when changing the vehicle's appearance. Select the vehicle that is most li to be the vehicle used in the crime.

a. b.

Truck A Truck B

c. Truck C d. Truck D

ANSWER: B The correct answer is choice B. Although the truck has been painted a darker color and a bed cap has been added, the body of the truck matches the picture of the vehicle used in this crime. All of the other answer choices have at least one discrepancy that could only have occurred as a result of bodywork to the truck. Choice A is incorrect because the bed of the truck is too long.

Choice C is incorrect because the truck has a high back end that extends to the top of the cab, similar to a rental truck. Choice D is incorrect because the vehicle also has a high back end that includes an extra door and window, resembling a Range Rover or Suburban type vehicle.

NOTES: REF:

Use the MAP to answer the following questions:

Police 25 Question Practice Test

Page 4 of 18

5.

You are at the Police Station preparing to go out on patrol when you are dispatched to respond to a vandalism incident in progress at the auto school on High Street (between Dudley Boulevard and Jefferson Avenue). The most direct route to the auto school, from the Police Station, without break any traffic laws, is as follows:
a. b. c. d. Travel north on Spruce, west on Marquette, north on Dudley, and east on High to the auto school. Travel north on Spruce, west on Garfield, north on Dudley, east on Marble, north on Jefferson, and west on High to the auto school. Travel north on Spruce, west on Garfield, north on Dudley, and east on High to the auto school. Travel north on Spruce, east on Marquette, north on Jefferson, and west on High to the auto school.

ANSWER: D Choice D provides the only route to the incident without breaking any traffic law.

Police 25 Question Practice Test

Page 5 of 18

Choice A is incorrect because it suggests the candidate travel in the wrong direction (i.e., east) on High. Choice B is incorrect because it suggests the candidate travel in the wrong direction (i.e., west) on Garfield. Choice C is incorrect because it suggests the candidate travel in the wrong direction (i.e., west) on Garfield AND (that is, east) on High.

NOTES: REF:
6.

You are located at the intersection of Oak Street and Jefferson Avenue and are out on patrol in you district. You turn north and travel for 1 block, then turn west and travel for 4 blocks, then turn no and travel for 2 blocks, then turn east and travel for 5 blocks, and finally turn south and travel for block before stopping. You have stopped at the intersection of: a. b. Jefferson and Tucker. Jefferson and Stover. c. Broad and Tucker. d. Ridge and Broad.

ANSWER: C The correct answer is Choice C, Broad and Tucker Streets. Beginning from Oak and Jefferson, you travel north 1 block to Jefferson and Market. Then, travel west 4 blocks to Market and Spruce, north 2 blocks to Spruce and Stover, and east 5 blocks to Stover and Broad. Finally, you travel south 1 block to Broad and Tucker. NOTES: REF:
7.

A police officer is talking to a local citizens' group on the role police play in problems of domestic violence. The officer has explained that police officers are often called because a domestic conflict i disturbing neighbors. In addition, the officer has pointed out that many calls occur prior to a crime being committed. This is important since one of the objectives of police officers is to stop crime befo starts. The most effective way for the officer to accurately sum up the role of the police in domestic conflicts is as follows:
a. b. "Since frequently no crime has been committed, the role of the police in domestic conflicts is fairly limited." "In resolving domestic conflicts, the officer's function is to restore order and prevent possible crimes from occurring." "The officer's main goal in responding to domestic crimes is to protect the innocent." "Many people involved in domestic conflicts call the police in order to have an objective authority help settle disputes."

c. d.

ANSWER: B Choice B is the correct answer. It correctly reflects the two roles police play in problems of domestic violence: 1) handling a conflict that disturbs the neighbors; AND 2) stopping crime before it starts. Although answer choices A, C, and D do relate (in varying degree) to other aspects of police involvement in domestic violence issues, none of these aspects are directly mentioned in the information in the question and none include the notion of the dual roles of the police.

Police 25 Question Practice Test

Page 6 of 18

NOTES: REF:
8.

Police Officer Turner is writing a report regarding the criminal homicide he investigated this morn The report will include the following sentences: (These sentences are NOT listed in the correct orde
1. I noticed a display case for handguns was open, but there were no guns in it.

2. We got a call reporting that the front door of a pawnshop was open, but the owner was not there.
3. We interviewed tenants of the apartments in the upstairs sections of the same building. 4. On arrival, we discovered the body of the shop owner, apparently dead from knife wounds to the chest. 5. I called an ambulance to pick up the victim.

Officer Turner could communicate the above sentences most effectively if he presented them in the following order: a. b. 4, 5, 2, 3, 1. 2, 1, 4, 5, 3. c. 2, 4, 5, 1, 3. d. 2, 5, 1, 3, 4.

ANSWER: C

NOTES: REF:

The correct answer is choice C. To identify the correct order, the sentences for the report need to be ordered into the proper time sequence. The clearest clues involve sentences 4 and 2. Sentence 4 begins "On arrival..." Therefore, sentence 4 must separate the actions that occurred prior to the arrival at the scene from those events occurring after the arrival. Sentence 2 is the only statement regarding actions prior to arrival, that is, getting the call. Thus, sentence 2 must occur first and be followed by sentence 4. Only answer choice C begins with the sequence 2, 4. In addition, the rest of the sequence (5, 1, 3) is also appropriate.

Reading Comprehension Passage

Use the information in the following passage to answer the next two questions. At 9:47 p.m., two patrol units were dispatched to a suburban home on the evening of June 12. The dispatcher stated that the owner had reported a robbery, which took place earlier that evening whe and his wife were not at home. The owner also stated that the suspects were in the home at the time their arrival, but that they fled the scene in a dark blue sedan.

Upon arrival at the home, Officers Baker and Reigner began to question Mr. Corneal, the owner o home. In the meantime, Officers Lucas and Gentry conducted a preliminary investigation of the ho determine the activities of the suspects and the extent of damage. The questioning revealed that Mr Corneal collected exotic guns and had accumulated an extensive collection. In addition, Mrs. Corne

Police 25 Question Practice Test

Page 7 of 18

had several sets of antique jewelry. When questioned as to individuals who knew about the collectio Mr. Corneal stated that only friends of the family and members of the rifle club were aware of them. He was further questioned to determine whether an unusual events had occurred recently that might be related to this incident.

Mr. Corneal stated that they had extensive remodeling done to the interior of the home during the month and a half by several different companies. Officer Reigner asked if they had reason to suspe of the workers. Mr. Corneal stated that several of the workers had shown an interest in seeing and discussing his collection of guns, but that it would be very difficult to determine which individuals, any, may have been involved. Officer Reigner requested the names of the companies and the servic they performed.

After a few minutes, Officers Lucas and Gentry returned from their search of the home. Since the other areas that were disturbed were ones that typically contain money (e.g., dresser top and dress drawers), they determined that the robbers concentrated their efforts on the two gun collections. T concluded that the robbers must have been aware of the gun collections before entering the home.
9.

According to the above passage, when the Corneals arrived home on the evening of June 12, they discovered that their home had been: a. b. c. d. ransacked, but nothing was stolen. entered, and the television, stereo, and computer were stolen. entered, and the television, stereo, and computer were stolen. robbed, and five antique guns and several pieces of exotic jewelry were the items that were taken.

ANSWER: C

NOTES: REF:
10.

Choice C is the correct answer. As stated in the passage, "The owner of the home had reported a robbery..."AND"... that the suspects were in the home at the time of their arrival." Although the information in the passage does not indicate the exact number and type of questions taken from the Corneal home, Officers Lucas and Gentry did determine that the robbers concentrated their efforts on the exotic gun collection, the antique jewelry collection and the areas of the house that typically contain money. Therefore, answer choice A is incorrect because it states that nothing was stolen; answer choice B is incorrect because the wrong questions are listed; and answer choice D is incorrect because there was no mention of money being stolen from the kitchen.

According to the robbery and subsequent investigation described above, the ONLY statement that accurately reflects the information gathered is:

a. b. c. d.

Officer Reigner requested the names of the companies doing the remodeling. Mrs. Corneal seemed to be the one to answer all of the police officer's questions. Officer Baker helped with the search of the home. The robbers were reported to have fled the scene in a dark blue van.

ANSWER: A Choice A is the correct answer. Officer Reigner was the officer who "requested the names of

Police 25 Question Practice Test

Page 8 of 18

companies and the services they performed."

Choice B is incorrect. Mr. Corneal seemed to answer all of the officers' questions. Choice C is incorrect because Officers Lucas and Gentry searched the home. Officer Baker assisted with the questioning of the owners. Choice D is also incorrect. The robbers were reported to have fled the scene in a dark blue sedan, not a dark blue van.

NOTES: REF:
11.

Police officers should follow the guidelines presented below when dealing with individuals suspecte driving while under the influence of alcohol:
1. A SEPARATE citation shall be issued for any traffic offense that originally brought the offender to the attention of the officers (e.g., reckless operation of motor vehicle). 2. If the driver refuses to submit to a blood alcohol test OR submits to such tests and scores above the limit, the arresting officer should order the driver to surrender his/her operator's license and issue a citation for driving under the influence. 3. If the violator scores below the legal limit on the blood alcohol test, the operator's license SHALL NOT be confiscated. The results of field tests (e.g., walking heel to toe on a straight line) have no influence on this guideline. 4. Even if the violator scores below the legal limit, the violator can still be charged with driving under the influence if the officer can justify the charge through the use of field tests.

Officer Kelley notices a car swerving in and out of its lane. Officer Kelley stops the car and asks th driver to submit to a test for driving under the influence of alcohol. The driver submits to the test a scores below the legal limit. However, based on several field tests, Officer Kelley still charges the dr with driving under the influence. Officer Kelley writes a citation for reckless operation (i.e., swervi from lane to lane), a citation for driving under the influence, and a citation for failing the field tests Officer Kelley then confiscates the operator's driver's license and takes the driver to the station for booking.
According to the guidelines presented above, Officer Kelley's actions were:

a. b. c. d.

appropriate because they were consistent with the guidelines provided. problematic because the driver was forced to surrender his license. problematic because Officer Kelley did not have a witness to the field tests. problematic because Officer Kelley gave the driver a citation for driving under the influence even though the driver passed the blood alcohol test.

ANSWER: B 11. Choice B is the correct answer. The driver submitted to the blood alcohol test and scored below the legal limit. According to guideline 3, under these conditions, "the operator's license SHALL NOT be confiscated." Officer Kelley did take the operator's license and that was problematic. Given the fact that there was a problem with the license (choice B), choice

Police 25 Question Practice Test

Page 9 of 18

NOTES: REF:
12.

incorrect. Choice C is incorrect because the original guidelines did not mention any need for witness to the field tests. Choice D is incorrect because (according to guideline 4) even if a v passes the blood alcohol test, "the violator can still be charged with driving under the influen the officer can justify the charge through the use of field tests." Both conditions have been m since Kelley's citations included driving under the influence AND failing the field tests.

Officer Johnson received the following four statements from witnesses who were in The First Natio Bank during a robbery:
Witness #1 - "There were two of them. Both male, about 6'2" tall with medium builds. One of them had dark hair and was wearing jeans, a leather jacket, and sunglasses. The other had short blond hair and was wearing jeans and a red lightweight jacket. Only the one with the sunglasses spoke, and he had a southern accent." Witness #2 - "Both of the men were about the same size, maybe 6', about 180 lbs. One of them was wearing a leather jacket and the other one was wearing a lightweight red jacket. I could not see their faces because one had on a ski mask and the other had on dark sunglasses. The one with the sunglasses had dark hair." Witness #3 - "The two men were both about 6'1" and 180 lbs. One had on a leather jacket, the other a red windbreaker. Both men were wearing jeans and tennis shoes. One had dark hair and was wearing sunglasses. The other one was wearing some kind of stocking cap that was pulled down over his face." Witness #4 - "Both men had on jeans, tennis shoes and jackets. One jacket was leather, the other bright red. One of the men had dark hair and I could not see the other one's face because of a ski mask. They both looked a little over 6 feet tall. Only the one with the dark hair spoke."

Based on the description presented above, Officer Johnson should recognize that there is a problem the account provided by Witness #: a. b. 1 2 c. 3 d. 4

ANSWER: A Choice A is the correct answer because witness #1 gave the only inconsistency between the various accounts. There is agreement that there were two males with similar builds (about 6' tall and 180 lbs.). In addition, from all accounts, the first robber had dark hair, jeans, a leather jacket, sunglasses, tennis shoes and spoke AND the second robber had jeans, a red jacket and tennis shoes. The only difference is witnesses #2, #3 and #4 state that the second robber's face was hidden by a ski mask/stocking cap. Witness #1 not only failed to mention the mask but also was the only witness who claimed that this individual had blonde hair. All other details were confirmed by at least two of the four witnesses. NOTES: REF:
13.

Use ONLY the definitions provided below to answer this question.

Police 25 Question Practice Test

Page 10 of 18

Aggravated Burglary - The crime of aggravated burglary is committed when an individual trespasses by force or deception into an occupied structure with intent to commit a theft or felony. Also, during the trespass, the individual must harm another or carry a deadly weapon. Burglary - The crime of burglary is committed when an individual uses force or deception in order to trespass in an occupied structure with intent to commit a felony or theft. Breaking And Entering - The crime of breaking and entering is committed when an individual, intent on committing a theft or felony, trespasses in an unoccupied structure by force or deception.

Consider the following situation: A janitor enters a busy office building, identifies himself as a jani and begins cleaning. While cleaning one office, he notices an appointment book on the floor. When mops, he picks up the book and places it in his pocket while he finishes the job. Once completed, he leaves the building. It turns out that this janitor was really one of the competitors of the company occupying the building and that he planned to take any documents he could get.
Given the definitions described above, this scenario is best categorized as:

a. b.

aggravated burglary. burglary.

c. breaking and entering. d. none of the above.

ANSWER: B

NOTES: REF:
14.

Choice B is the correct answer. When comparing these three crimes, the common elements are: 1) trespassing by force or deception; AND 2) intent to commit a theft or felony. This individual did both by the use of deception to gain entry and his plan to "take any documents he could get.” Because none of the definitions describe any exceptions if items are found on the floor, choice D is incorrect. Choice A is incorrect because there was no mention that the intruder harmed another OR carried a deadly weapon. Choice C is also incorrect because the structure was occupied. (Upon entering the building, the individual "identified himself..." to someone else in the building "...as a janitor.")

The following dress code guidelines apply to Police Officers. Consider only the information present here when answering the question.
1. The Antron Jacket will be worn for outside duty during the months of November through February and whenever the temperature is expected to drop below 55 degrees. 2. The Eisenhower Jacket shall be worn for outside duty during the months of March, April, May, September, and October only if the temperature is below 65 degrees. Officers with the rank of Captain or higher may wear the Double Breasted Blouse instead of the Eisenhower Jacket. 3. A navy blue long sleeve uniform shirt shall be the standard to be worn with the jackets. All individuals at the rank of Captain or above will substitute a white

Police 25 Question Practice Test

Page 11 of 18

shirt. 4. The short sleeve shirt may be worn whenever the temperature is to rise above 70 degrees during the months of May through September. 5. Officers assigned to indoor duty may also substitute the short sleeve shirt for the long sleeve shirt. 6. Regulation trousers shall be worn. The black trouser braid or stripe shall be worn by individuals at the rank of Lieutenant and above. Typical Police Officer Ranks (from highest to lowest): Police Chief Deputy Chief Captain Lieutenant Sergeant Police Officer

Consider the following circumstance: The temperature for the day is expected to stay right around degrees. The date is September 15th. According to the regulations outlined above, Captain Cross w be dressed appropriately for outside duty if he wore:
a. b. c. d. an Eisenhower Jacket, a white long sleeve shirt, and regulation trousers with a black stripe. a Double Breasted Blouse, a navy blue long sleeve shirt, and regulation trousers. an Antron Jacket, a white long sleeve shirt, and regulation trousers with a black braid. a Double Breasted Blouse, a white short sleeve shirt, and regulation trousers with a black braid.

ANSWER: A Choice A is the correct answer. Based on the Captain's work situation and the date and temperature, Captain Cross may wear either the Eisenhower Jacket or the Double Breasted Blouse, a white long sleeved shirt, and regulation trousers with either the black trouser braid or stripe.

Choice B is incorrect because, according to guideline 3, a Captain "will substitute a white shirt" for the navy blue long sleeved shirt.
Choice C is incorrect because, according to guideline 1, it is too early in the year (September) and too warm (60 degrees) to justify wearing the Antron Jacket.

NOTES: REF:
15.

Choice D is incorrect because, according to guidelines 4 and 5, the Captain CANNOT wear the short sleeved shirt because the temperature is too cold (60 degrees) and the Captain is serving outside duty.

Police Officer Thompson has noticed that in the district of the city that he patrols, all of the assault

Police 25 Question Practice Test

Page 12 of 18

occur in the eastern and northern sections, all of the auto thefts occur in the southern and western sections, and all of the traffic accidents occur in the western section.

The majority of the auto thefts take place between 3 a.m. and 7 a.m. Most of the traffic accidents oc either between 7 a.m. and 9 a.m. or between 5 p.m. and 8 p.m. Most of the assaults occur between 7 and 9 p.m. or between 11 p.m. and 4 a.m. In addition, the traffic accidents almost always occur on Mondays and Fridays, the assaults take place on any day from Wednesday through Saturday, and thefts typically take place on weekday mornings.
Police Officer Thompson would be most likely to reduce the number of assaults by patrolling the:

a. b. c. d.

eastern section between 5 a.m. and 1 p.m. eastern section between 5 a.m. and 1 p.m. eastern section between 11 a.m. and 7 p.m. northern section between 5 p.m. and 1 a.m.

ANSWER: D Choice D is the correct answer. It includes both an appropriate section of the city and an appropriate time of day. When examining the choices, the answer choices include the section of the city and the time of day. There is no indication of the day of the week. Thus, the information in paragraph 3 is irrelevant to this decision. According to the first two paragraphs, assaults occur "in the eastern and northern sections" and "between 7 p.m. and 9 p.m.” OR “between 11 p.m. and 4 a.m." Choice B is incorrect because it is the wrong section of the city for assaults. Choices A and C are incorrect because they do NOT include any of the hours of the day when assaults are more likely to occur. NOTES: REF:
16.

During the first half of the month of May, Police Officer Riggins received a series of reports from p who were mugged outside the South Side Shopping Plaza. The description of each suspect appears below:
#1 (May 7) - male, Black, early 20s, 5'9", 170 lbs., black hair, tattoo on his upper arm, blue jeans and T-shirt. #2 (May 13) - male, Black, 20-26, 5'10", 175 lbs., black hair, tattoo on left hand, tank top, jeans, and sneakers. #3 (May 15) - male, Black, late teens, 5'11", 190 lbs., brown hair, snake tattoos on both upper arms, and red tank top. #4 (May 20) - male, Black, 17-21, 6'1", 210 lbs., brown hair, brown pants, a three-quarter length sleeve shirt, and no jewelry. #5 (May 21) - male, Black, 16-20, 6'7", 230 lbs., black hair, an earring in the left ear, blue jeans, and a tank top. Officer Riggins referred to this information when examining the incident described below. On June 3rd, a Black male was arrested near the Plaza. In the description that led to the arrest, a witness described the suspect as a Black male who looked about 23 years old. He

Police 25 Question Practice Test

Page 13 of 18

was about 5'10" tall and weighed about 180 lbs. He had black hair, a tattoo of a cobra on his bicep, a muscle shirt, and blue jean cut-offs.

Based on this description and the information from the five incidents that occurred in May, this su should also be considered a suspect for incident number: a. b. 1 2 c. 3 d. 4

ANSWER: C Choice A is the correct answer. To answer this question it is helpful to create a table with all various features for incidents 1, 2, 3 and 5 (4 is NOT one of the answers) and for the June 3r incident. Using this method, the other three answer choices can be eliminated. Choice B is incorrect because the only tattoo is on the left hand, not on the bicep. Since this suspect had on a tank top, a cobra tattoo on the arm would be hard to miss. Choice C is incorrect because the suspect's hair is brown, not black. Choice D is incorrect because the suspect is too tall and heavy (6'7" and 230 lbs.). NOTES: REF:
17.

The technique for dusting for fingerprints involves the following procedures: (These techniques are presented in the correct order). 1. Choose a powder color in contrast with the surface to be dusted. 2. Dip a brush into the powder and work the powder into the fibers of the brush. 3. Lift the brush out of the powder, checking to make sure there isn't too much powder on the brush. 4. Holding the brush lightly, shake a light dusting of powder onto the suspected area. 5. If a light pattern shows up, brush more powder into the pattern lightly with the flow of the ridges. 6. Lightly clean up the pattern by brushing excess powder out of the voids between the ridges to define the print. 7. Photograph the impression and proceed to lift the impression with transparent tape.

An officer has followed correct procedure when dusting for fingerprints and finally notices a patter emerging. The next step the officer should do is: a. follow along the pattern with additional powder on the brush. b. shake a light dusting of powder over the entire area. c. shake the brush lightly to make sure there is no excess powder on it. d. try to define the print by brushing excess powder out of the voids.
ANSWER: A The correct answer is choice A. The key phrase in this question is "finally notices a pattern emerging." Procedure number 5 reads "If a light pattern shows up,..." the next step is to "...brush more powder into the pattern lightly with the flow of the ridges." Choices B, C and

Police 25 Question Practice Test

Page 14 of 18

NOTES: REF:
18.

D are incorrect because they correspond to procedures 4, 3 and 6 respectively.

When a prisoner is brought in to be booked, a basic search is conducted. The steps involved in a search are listed below: (These steps are NOT presented in the correct order.) 1. Lower both of your hands to the base of the prisoner's neck and proceed with the search by covering the chest, stomach, and back. 2. Have the inmate empty all pockets in trousers, shirt, coat, and jacket. 3. Be sure no other inmates are in the room when the search takes place. 4. From the waistline, proceed down the legs, using both hands on one leg, then the other. 5. Have the inmate stand facing a wall with his back to you, legs spread, and arms extended straight out. 6. From behind, using both hands, start at the forehead and run your fingers or a comb through the inmate's hair. The above procedures should be performed in the following order:

a. b.

2, 1, 4, 3, 5, 6. 3, 5, 2, 6, 4, 1.

c. 3, 2, 5, 6, 1, 4. d. 3, 5, 4, 2, 6, 1.

ANSWER: C The correct answer is Choice C. To determine the correct answer, the procedures must be listed in the correct time sequence. Procedures 2, 3 and 5 deal with the preparation for the search and procedures 1, 4 and 6 deal with the actual search. In terms of the last three tasks, 6 must be first because it instructs the officer to "start at the forehead." Procedure 1 must be next because it states: "Lower both of your hands to the base of the prisoner's neck and proceed… .” Procedure 4 is the continuation of procedure 1 since the officer finishes #1 at the mid-section and begins #4 at the waistline. Choice C is the only one ending with the sequence 6, 1, 4 AND having the appropriate order for the initial three procedures (3,2,5). NOTES: REF:
19.

The Divisions of Police and Fire have standard procedures for handling bomb threats and bomb incidents including the following: 1. Trained police personnel direct operations at the scene; fire personnel stand by and typically assist in rescue operations.
2. While radios may be left on at the scene of an unexploded device to receive instructions from the Communications Office, no radio transmissions are to be made from the scene, since radio signals can detonate an explosive device.

3. The decision to evacuate a building is to be made by the management of the building unless

Police 25 Question Practice Test

Page 15 of 18

an explosive device has been found; in such an instance, the police personnel in charge of the operation make the evacuation decision. 4. No public statements are to be made to the media by police or fire personnel.

5. If one device detonates, there is always the possibility of a second or third device, so police and fire personnel should stay clear of the area until it has been determined (usually by police bomb squad personnel) to be safe.

Given the above procedures, the most potentially dangerous mistake has been made in which of the following situations?
a. b. c. d. Immediately after a second bomb exploded in a bank, Firefighter Thomas entered the bank to conduct rescue operations. While standing by at the scene of a bomb threat, Police Lieutenant Caffey provided information regarding the incident to a reporter. While standing by at the scene of a bomb threat, Fire Lieutenant Griffin received instructions from the Communications Office via his radio. After an unexploded device was discovered in an apartment building, Assistant Fire Chief Johnson ordered the apartment building manager to evacuate the building.

ANSWER: A Choice A is the correct answer. Choices A and B represent the only mistakes. With respect to choice A, Firefighter Thomas should wait to enter the bank until the bomb squad personnel determine that it is safe (according to procedure 5). With respect to choice B, no police perso including Lieutenant Caffey, should make a public statement to the media (according to proc 4). Since the question asks for the "most potentially dangerous mistake," the best response is choice A since entering the bank without proper safety clearance is a direct risk to one's own and the life of anyone who follows. Choice C is incorrect because (according to procedure 2) okay to receive instructions via the radio as long as no transmissions are made. Choice D is incorrect because (according to procedure 3) the actions of Assistant Fire Chief Johnson wer appropriate. NOTES: REF:
20.

Police Officer Dunn interviews four witnesses to a murder which took place in a crowded bar and grill. Each of the witnesses observed the perpetrator of the shooting as he was lea the bar and grill. They described him as follows: Witness #1: "He was a White male, about 25 to 30 years of age, with brown shoulder length hair. He was about 6'0" and weighed about 185 pounds. He wore blue jeans and a brown leather jacket. He had a small scar on his forehead." Witness #2: "He was a male, White or Hispanic, late twenties, 5'10", 190 pounds, with long brown hair. He was stocky and wore dark pants and a brownish jacket." Witness #3: "He was a light skinned male, possibly White or Hispanic, in his mid twenties, about 6'0" and weighed about 180 pounds. He had a tattoo of a panther on his right forearm. He had moderately long hair and wore a brown shirt and dark pants." Witness #4: "He was a White male, around 25 years old, about 5'11" and weighed about 185

Police 25 Question Practice Test

Page 16 of 18

lbs. His hair was brown and over his ears, fairly long. He wore darkish clothes, I'm not sure of the color of his jacket or pants." Based on the above information, Officer Dunn should recognize that there is a problem with the description given by Witness #: a. b. 1 2 c. 3 d. 4

ANSWER: C Choice C is the correct answer. There is agreement that the perpetrator was a White or Hispa male in his mid to late twenties with long brown hair who was approximately 6' tall and 185 addition, his pants and jacket were dark in color. There were only two discrepancies across t descriptions. Witness #1 reported that the perpetrator had a small scar on his forehead. Witne reported a tattoo of a panther on the perpetrator's right forearm. Of the two, it is much harder explain how a tattoo of a panther could be overlooked by three of four witnesses AND how o witness could see it when the other three witnesses all stated that the perpetrator had a jacket during the incident. Thus, choice C is the best answer. NOTES: REF:
21.

When a police officer arrived at the scene of a disturbance in an abandoned warehouse, she noticed van leaving the site. Since her investigation revealed that the warehouse had been vandalized, the o felt she should include her observation of the van in her report. The most effective way for the polic officer to report this fact is by saying: a. b. c. d. "When I arrived at the warehouse, I saw a van driving away from the site." "A van which should not have been there was at the warehouse." "The vandals escaped in a van." "I do not know if it's important but when I got there, I saw a vehicle leaving the scene."

ANSWER: A A is the correct answer because it correctly reflects the facts given in the question. Answers C are incorrect because they provide details that have not been established (the van should n have been there AND the vandals escaped in the van). The police officer was only able to sta the vehicle left the site as she arrived. Answer D is not the best answer because it leaves out important fact that the vehicle was a van. NOTES: REF:
22.

Police Officer Wilkins is preparing a report after leaving the scene of an accident.

The report will include the following five sentences: (These sentences are NOT listed in the correct order.)
1. The Dodge struck the right rear fender of Mrs. Smith's Ford, and continued on its way. 2. Mrs. Smith stated she was making a left turn from 40th St. onto Third Avenue. 3. As the car passed, Mrs. Smith noticed the dangling rear license plate #412AEJ. 4. Mrs. Smith complained to police of back pains and was taken by ambulance to Bellevue Hospital. 5. An old green Dodge traveling up Third Avenue went through the red light at 40th St. and

Police 25 Question Practice Test

Page 17 of 18

Third Avenue. The most logical order for the above sentences to appear is as follows:

a. b.

5, 3, 1, 2, 4. 1, 3, 2, 5, 4.

c. 4, 5, 1, 2, 3. d. 2, 5, 1, 3, 4.

ANSWER: D The correct answer is D. To identify the correct order, the sentences for the report need to be organized according to the proper time sequence for the accident. Sentences 1, 2, 3 and 5 des the events of the accident. The proper sequence for these four events is 2, 5, 1, 3. (Mrs. Smit attempting a left turn, a Dodge ran the red light, struck her vehicle and continued on its way, as it passed she noticed its license plate number.) Answer D is the only answer choice with th sequence.

When working with a question like this, look at the content of each statement or phrase separately and determine whether it can stand alone or if it must precede or follow another statement. If it cannot stand alone, look for the statement that contains the information you need. However, do not forget to evaluate that statement in the same way as the first. Ask the questions, what happened first and what happened next? Similarly, consider whether there ar statements that the statement under consideration cannot precede or follow. This also will he to narrow down the choices.

NOTES: REF:
23.

While you should identify the pairs of statements that must (or cannot) go together, you should not attempt to determine the correct order of all of the statements before checking the answer choices provided. There may be several logical ways in which to order the statement however, only one possibility will be included among the answer choices provided. Therefor should work through each of the answer choices presented one by one, keeping in mind the statements that must (or cannot) appear together. Answer choices that are not feasible should eliminated until you find the one answer choice that places the statements in the appropriate

Police Officer Crawford received a series of reports from several people who were mugged in the early evening as they were exiting from the Spruce Street subway station. The descr of each suspect is as follows: Report No. 1 (November 16): Male, White, early 30s, 5'10", 180 pounds, dark hair, mustache, one gold earring, blue jeans, black jacket, running shoes. Report No. 2 (November 20): Male, White, 25-30, 5'6", 120 pounds, dark hair, dark glasses, one gold earring, blue jeans, green sweat shirt, running shoes. Report No. 3 (November 21): Male, White, 40-45, 5'10", 130-140 pounds, dark hair, mustache, one gold earring, blue jeans, black jacket, running shoes. n November 23rd, another person was mugged by a male who was loitering near the subway station exit. The clerk selling tokens witnessed the mugging, called 911, and the male was apprehended two blocks away. The description of the suspect is as follows: Report No. 4 (November 23): Male, White, 25-30, 5'10", 175 pounds, dark hair, mustache,

Police 25 Question Practice Test

Page 18 of 18

blue jeans, black jacket, green ski cap, boots.

Based on the description of the suspects in the first three reports, the suspect in Report No. 4 shoul be considered a suspect in: a. b. c. d. Report No. 1, but not in Report Nos. 2 or 3. Report Nos. 1 and 2, but not in Report No. 3. Report Nos. 2 and 3, but not in Report No. 1. Report Nos. 1, 2, and 3.

ANSWER: A After setting up a table with the characteristics of the four reports, similarities are easily iden In addition, the critical dimensions can be identified. The important differences center aroun characteristics -- age, height, and weight. Report No. 4 does not match Report No. 2 because muggers differ substantially with respect to height and weight. Report No. 4 does not match Report No. 3 because the muggers differ substantially with respect to age and weight. Repor does closely match Report No. 1. Using this information, the correct answer is A. NOTES: REF:
24.

Add: 327+512

a. b.

475 510

c. 545 d. 839

ANSWER: D In this example, the correct answer is D, because the sum of 327 and 512 is 839. NOTES: REF:
25.

An arrested man’s bail was set at $3500. He had to put up 10% of that amount in cash in order to b released. If the man had $215 in his wallet, how much did he still need?

a. b.

$35 $125

c. $135 d. $145

ANSWER: C 10% of $3500 is $350 $350 - $215 = $135 NOTES: REF:

Police Practice Test 2

Page 1 of 38

Police Practice Test 2 Answer Key
Multiple Choice MAP QUESTIONS

Use the below MAP to answer the following questions. NORTH ^

1. You are a police officer assigned District 10, located at Means Ave and Elm St. You currently are at the district finishing roll call. You are dispatched to a Traffic accident on N. Oneal Ave between W. 2nd St and W. 1St. The most direct route to take without violating any traffic laws is; a. Drive West on Means Ave to N. c. Drive West on Means Ave to N. Jessup Ave, North on N. Jessup to 1 Jessup Ave, South on N. Jessup to 1 St, East on 1St to W. Oneal And St, East on 1St to W. Oneal And North On W. Oneal Ave to the South On W. Oneal Ave to the Accident. Accident. b. Drive West on Means Ave to N. d. Drive West on Means Ave to N. Jessup Ave, South on N. Jessup to 1 Jessup Ave, South on N. Jessup to St , West on 1St to W. Oneal and 1 St , West on 1St to W. Oneal And

Police Practice Test 2

Page 2 of 38

South On W. Oneal Ave to the Accident.

South On W. Oneal Ave to the Accident.

ANSWER: B NOTES: REF: 2. You’re at Elm St and Clark St. You travel South from that location for one block, you turn E. and travel 2 blocks, you turn S. and travel 2 blocks, turn W. and travel 2 blocks before stopping. When you stop what intersection are you at? a. Clark & Elm c. Means & Elm b. Oneal & 1 St d. 2nd & Means ANSWER: C

NOTES: REF:

OBSERVATION AND MEMORY QUESTIONS

Accident Scene 1:

Answer the following questions based on the following accident scene photo. You' have three minutes to examine the photo. The questions will test your ability to ll recall facts and events occurring in the photo.

You' have three minutes to examine the above photo. The questions will test your ability ll to recall facts and events occurring in the photo.

Police Practice Test 2

Page 3 of 38

Don’t scroll past this point until the three minutes are up.

3. How many firemen are in the photo? a. 3 c. 5 b. 4 d. 6

ANSWER: C NOTES: REF: 4. The car involved in the accident hit a...

a. truck b. fire hydrant ANSWER: D NOTES: REF: 5. What were the firemen doing? a. All Five were standing by the vehicle involved in the accident.

c. tree d. pole

c. Four were standing by the vehicle involved in the accident and one holding the hose. b. Four were standing by the vehicle d. Three were standing by the vehicle involved in the accident and one involved in the accident and two was kneeling on the ground several were kneeling on the ground feet away. several feet away.

ANSWER: B NOTES: REF: 6. How many Traffic cones are in the photo? a. 2 c. 4 b. 3 d. 5

ANSWER: A NOTES: REF:

Accident Scene 2:

Answer the following questions based on the following accident scene photo. You' have three minutes to examine the photo. The questions will test your ability to ll recall facts and events occurring in the photo.

Police Practice Test 2

Page 4 of 38

You' have three minutes to examine the above photo. The questions will test your ability ll to recall facts and events occurring in the photo.

Don’t scroll past this point until the three minutes are up.

7. Where was the damage on the car? a. front bumper b. left side door

c. left front quarter panel d. right front quarter panel

ANSWER: D NOTES: REF: 8. What color is the fire truck door? a. red and white b. blue

c. red d. blue and white

ANSWER: D NOTES:

Police Practice Test 2

Page 5 of 38

REF: 9. What are the letters on the fire truck? a. BL c. FD b. XA d. LA

ANSWER: D NOTES: REF: 10. What were the firemen doing? a. Both were looking at the car c. One was looking at the car and the other one was facing the police officer b. Both were facing the police officer d. One was looking at the car and the other one was standing next to the fire truck

ANSWER: C NOTES: REF: 11. Officer Riley interviews four witnesses to an armed robbery of a gas station. Each witness observed the suspect leaving the scene of the crime. They described the suspect as follows: Witness 1 He was a white male, 20 to 30 years of age with brown curly hair. About 6' “, 190 lbs. 1 He wore blue jeans and a black leather jacket. Witness 2 He was white or Hispanic, in his late 20' 5' ,195 lbs. with curly hair. Who was s, 11" wearing dark pants and a dark colored jacket. Witness 3 He was a thin white male, in his late 20' about 6 “, 185 lbs. with a tattoo of a cross on his s right bicep. He had brown curly hair. Witness 4 He was a light skinned Hispanic male, with dark curly hair, 5' to 6' about 200 10" 1" pounds. Wearing a black or brown jacket and blue jeans. Officer Riley should recognize the problem with which witnesses description?

a. 1 b. 2

c. 3 d. 4

ANSWER: C All the witnesses’ descriptions are similar EXCEPT witness number 3, who stated he observed the suspect with a tattoo on his right bicep. Witnesses 1,2 & 4 all agreed that the suspect was wearing a jacket , which made witness number 3' s

Police Practice Test 2

Page 6 of 38

NOTES: REF:

observation of a tattoo on the suspect' right bicep, not possible. s

12. Situation: When arriving at a scene, where a person needs medical assistance, a police officer should respond in the following order: 1. Render reasonable aid to the injured or sick person. 2. If needed call for Medical Assistance including an ambulance or a doctor. 3. Notify the dispatcher if the person is wearing a medic-alert emblem, indicating the person suffers from a serious medical problem such as heart disease, diabetes etc... 4. Make sure the ambulance gets directed to the scene when it arrives.

5. If the ambulance does not arrive in a reasonable time, depending on the seriousness of the medical problem, place a second call for the ambulance. 6. Record all the pertinent information including, time, names of persons notified, medic alert information etc.. . .
Based on the given information answer the following question:

Officer Hunt is advised by a citizen that an elderly woman just collapsed, just around the corner. Officer Hunt responds to the scene and immediately renders first-aid. At this time he notices the woman wearing a Medic-Alert emblem indicating heart disease and Officer Hunt calls for an ambulance. What should Officer Hunter do next?
a. Make arrangements to direct the ambulance to the scene. b. Wait for the ambulance. c. Record the information in his daily log. d. Notify the dispatcher of the medical alert.

ANSWER: D Notify the dispatcher of the medical alert. NOTES: REF:

13. Use the following information to answer the below questions : Traffic Accidents are classified in the following categories: 1. One vehicle accidents with no injuries. 2. Two or more vehicle accidents with no injuries. 3. One or more vehicle accidents with injuries that are not life threatening. 4. One or more vehicle accidents with injuries that are life-threatening. Officer Allen responds to an accident at N. 20th Street and W. Logan. Upon arrival, Officer Allen is informed by a witness that a Ford traveling south on 20th St. struck a boy on a bicycle at the intersection of 20th and Logan. Officer Allen observes a boy standing by a bike. He appears to have minor injuries. Based on the given information what category accident is this?

Police Practice Test 2

Page 7 of 38

a. 1 b. 2

c. 3 d. 4

ANSWER: C The correct answer is C (3). One or more vehicle accidents with injuries that are not life-threatening. The easiest way to answer this question would be by the process of elimination. Eliminate the bad choices that don'pertain to the given t situation. NOTES: REF:

14. What is it called when a police investigation begins to focus upon a specific suspect? a. a proactive law enforcement strategy b. the accusatory stage c. the beginning of formal proceedings d. the high point of the criminal process

ANSWER: B NOTES: REF: 15. Which part of the crime detection steps typically follows an arrest?

a. booking b. accusation ANSWER: C NOTES: REF:

c. search d. diversion

16. Which of the following is NOT part of a drug courier profile? a. arriving late at night from a source c. listening to certain kinds of music city b. acting nervous or too calm d. walking rapidly or aimlessly

ANSWER: C NOTES: REF: 17. Which of the following would NOT justify a frisk? a. the officer is alone and backup has c. the stop is for a crime where not yet arrived weapons are commonly used b. the officer is scared because his/her d. the stop occurs in a high-crime area partner got shot last week

ANSWER: B NOTES:

Police Practice Test 2

Page 8 of 38

REF: 18. What is the purpose of Miranda rights? a. to reform the police so they change c. to eliminate unnecessary pressure, their procedures bordering on coercion b. to outlaw torture in any form, d. to neutralize distinct psychological physical or psychological disadvantages

ANSWER: D NOTES: REF: 19. Which of the following crimes has a better chance for suspect identification?

a. auto theft b. burglary

c. robbery d. vandalism

ANSWER: C Robbery is a crime where the suspect is, in most cases, observed by the victim therefore, making identification of the suspect easier. The other crimes listed ,auto theft , burglary, and vandalism are usually more difficult to solve because they' re usually committed without victim identification. This is a common sense question.

NOTES: REF:

MATH

20. Find the sum of 205, 87, 4352, and 361 a. 5004 d. 6005 b. 5001 e. 6015 c. 5005

ANSWER: C NOTES: REF: 21. Find the sum of 3412, 1234, 2143 and 4322 a. 12,110 d. 11,111 b. 11, 222 e. 10,011 c. 12,111

ANSWER: D

Police Practice Test 2

Page 9 of 38

NOTES: REF: 22. What' 798 X 450? s a. 366.976 b. 231,908 c. 259, 100

d. 359, 100 e. 459, 200

ANSWER: D NOTES: REF: 23. Divide 66,456 by 72 a. 923 b. 823 c. 913

d. 843 e. 123

ANSWER: A NOTES: REF: 24. 3/4 from 9/10 = a. 3/20 b. 1 c. 2/7

d. 3/5 e. 7/40

ANSWER: A NOTES: REF: 25. Find the sum of 3/5, 2/3, and 1/4 a. 3/4 b. 27/20 c. 4/5

d. 91/60 e. 1 6/7

ANSWER: D NOTES: REF: 26. It takes one man one day to dig one hole. How many days does it take 5 men to dig 5 holes? a. 1 d. 4 b. 2 e. 5 c. 3 ANSWER: A On day one, each man will be able to dig one hole. Therefore at the end of the first day each of the five men will have dug one hole making a total of 5 holes and the job is complete. NOTES: REF:

Police Practice Test 2

Page 10 of 38

27. A shirt is discounted from $49.95 to $44.95. The percentage discount is approximately;

a. 15% b. 14% c. 12%

d. 11% e. 10%

ANSWER: E We can round the price to $50, the discount is $5 which is 10% of the original price. Answer E is the closest answer. NOTES: REF:

28. What percentage of 220 is 33?

a. 0.15% b. 1.5% c. 5%
ANSWER: D

d. 15% e. 1500%

NOTES: REF:

1. 22 is 10% of 220 2. Therefore 11 is 5% of 220 3. So 33 must be 15% of 220

What Constitutes An Arrest?
The central idea of an arrest is the taking or detaining of a person by word or action into custody so as to subject his/her liberty to the actual control and will of the person making the arrest. There must exist the intent to take into custody and a corresponding understanding by the person arrested that he is in custody, although, no formal declaration of the arrest is required. An arrest also implies not only custody, but also the aim of bringing the person into the judicial process to answer for an offense.
Answer the following question(s) based on the above passage: Questions Regarding Arrest:

29. An officer, in uniform, firmly grips the person and says, "you’re going down to the station". This is:

a. not an arrest b. an arrest ANSWER: B NOTES: REF:

c. a stop and frisk d. none of the above

30. Same as above, except the officer is in plain clothes and identifies himself/herself as an

Police Practice Test 2

Page 11 of 38

officer. This is;

a. an arrest b. not an arrest

c. a stop and frisk d. all of the above

ANSWER: A The correct answer is A, an arrest. In both situations the officer subjects the liberty of the subject to his actual control and will . NOTES: REF:

31. An officer makes no physical contact and says, "Would you mind coming down to the station to answer some questions?" The person asks the officer if he/she must, and officer says, "You don' have to, but it would help get this thing cleared up. " This is: t a. an arrest because the officer asks c. a stop and frisk because it is not an the subject to go to the station arrest b. not an arrest because the officer told d. both A and C the subject he did not have to go to the station ANSWER: B The correct answer is B, because the officer does not subject the liberty of the person to his actual control and will. The officer told the subject he did not have to go to the station. NOTES: REF: READING COMPREHENSION PASSAGE:

Probable Cause:
The probable cause required for a warrantless arrest according to the Wisconsin Supreme Court, is no less than would support a warrant. The definition of probable cause in most jurisdictions is: That quantum of evidence that would lead a reasonable police officer to believe that the defendant committed a crime. It' more than a hunch or s suspicion, but less than the evidence required to convict at trial. The facts on which Probable Cause is based must be legally obtained. The Fresh-needle puncture marks on an addict' arm discovered by forcing, without further justification, the addict to remove s his/her jacket will not support an arrest. The test for probable cause is objective, not subjective. The arresting officer must believe the defendant probably committed a crime. It must be sufficient evidence to convince hypothetically, that a reasonable officer would find Probable Cause:

If an officer arrives at the scene of the crime in progress by chance or in answer to a call for assistance, there is no question that a warrantless arrest based on probable cause may be made when the suspect is apprehended, after a chase. The same is not true when an officer answers a call and upon arrival sees a person fleeing from the scene of what appears to be a completed crime. At this point the officer certainly has the right to stop the person for a reasonable time and question him/her regarding the crime but it is not

Police Practice Test 2

Page 12 of 38

sufficient basis to arrest without verifying some facts. The difference here is a crime in progress as opposed to a completed crime.

Answer the following questions based on the above passage:
32. Questions Regarding Probable Cause:

An officer receives a tip from an unknown source that X , a known heroin user, will be purchasing a quantity of heroin from Z, an unknown person, at a certain time and place (which would be in public view). The officer conceals himself near the place and, at the time predicted, observes X receive a package from a person in return for paper money.
Regarding Probable Cause: a. There is no probable cause b. There is probable cause to arrest only X c. There is probable cause to arrest both persons after the transaction d. none of the above

ANSWER: C The correct answer is C, there is probable cause to arrest both persons after the transaction Note: In the above situation, if the officer arrives at the previously disclosed location, and observes X alone and walking along the street, holding a small, unconcealed package, there is no probable cause for the arrest.

NOTES: REF:

33. A police bulletin reports a robbery and describes the suspect and the suspect’s car. An officer, hearing the description, observes a car traveling away from the scene, a very short time after the reported incident. The observed car matches the description, and as the officer gives chase, the suspect attempts to flee at a high rate of speed. When caught a. the suspect may not be arrested c. based on the fact no probable cause has been established for committing the robbery. b. the suspect may be arrested based d. on probable cause for committing the robbery. the suspect may not be arrested based on the fact the officer did not observe the robbery. all of the above

ANSWER: B The correct answer is B, the suspect may be arrested based on probable cause for committing the robbery. NOTES: REF: READING COMPREHENSION

Passage: Use Of Deadly Force

Police Practice Test 2

Page 13 of 38

An officer may use deadly force when he/she believes it is necessary to prevent death or great bodily harm to himself/herself or others. An accepted definition of deadly force applied by police officers is: The use of any means or instrumentality intended to or likely to cause death. Therefore, the justification for deadly force is the immediate threat of death or great bodily harm. While the application of deadly force is the action which is likely to cause death, there are limited number of instances where an officer may use deadly force in attempting to make an arrest or prevent escapes.
The arrest must be for a serious crime in which the suspect used or threatened to use deadly force against some member of the public, such as murder, rape, assault with a deadly weapon etc. Also, the officer must reasonably believe there is NO OTHER WAY to make the arrest in order to retain custody of person once arrested.

Even if the suspect is fleeing from officer, there has to have been a use or threat of use of deadly force by the suspect for the officer to use deadly force. In any situation the officer should not use deadly force unless he/she reasonably believes it is necessary and ONLY AS A LAST RESORT.
Can an officer use deadly force when no crime is committed? Yes, an officer may use deadly force at any time even when no crime has been committed. For example: whenever the officer believes it necessary to prevent imminent death or great bodily harm to himself/herself or another person(s) as seen as being in danger. Use Of Deadly Force In Minor Crime Situations:

In all other cases, the officer may not use deadly force, even if this restriction makes it impossible to arrest the person. For example, a fleet footed suspect out running the officer and successfully escaping. It makes no difference whether the person flees or resist arrest if the resistance is not with deadly force. However, even in minor crime situations the officer has a duty to attempt to make the arrest and if the person resists using deadly force, the officer may use deadly force in self-defense. EVEN IN SELF DEFENSE OF DEADLY FORCE however, the officer must reasonably believe that force less than deadly force will not be effective. 34. Question- Regarding The Above Passage and The Use Of Deadly Force:

A murder suspect, known to be armed and to have previously shot an officer is seen by Officer Mack, who calls on the suspect to stop and throw down his weapon. The suspect flees on foot from the officer and is escaping.
The Officer: a. Cannot shoot the suspect because c. no shots were fired at him (Officer Mack) b. Cannot shoot the suspect because he d. (Officer Mack ) did not observe the suspect commit the murder Can shoot the suspect because the suspect is known to be armed and is fleeing Can shoot the suspect because the officer knows the suspect is wanted for murder, is known to be armed and is escaping.

Police Practice Test 2

Page 14 of 38

ANSWER: D A and B can be eliminated. Officer Mack does not need to have been a witness to the murder or does he need the suspect to shoot at him for him to use deadly force on this occasion. (Here' where the history of the suspect comes into play). Even s though the suspect did not display a gun or shoot on this occasion, the officer knows the suspect is wanted for murder , has shot an officer in the past , is known to be armed is fleeing and will escape if deadly force is not used. These things add up to the probability that the suspect will continue to use extreme violence, and possibly causing death or great bodily harm again if he escapes.Therefore, the officer is justified in using deadly force on this occasion. NOTES: REF:

35. A robbery suspect has shot at the pursuing officer and then runs away. The officer: a. may shoot the suspect c. may not shoot the suspect because he knows where the suspect lives and can arrest him later b. may not shoot the suspect because d. may shoot the suspect only if the the suspect stopped shooting suspect turns and shoots at him again ANSWER: A The officer may shoot the suspect because it' reasonable to assume the suspect s will shoot again. It should also be considered a case of self-defense. NOTES: REF:

36. A suspect, wanted for a traffic offense, resists arrest, lunging at the officer with a knife. Regarding this statement which of the following is true: a. The officer cannot use deadly force c. The officer can use to deadly force because the suspect is not wanted because the suspect is resisting for a serious crime. arrest. b. The officer can use to deadly force d. The officer can use deadly force because the suspect is resisting because even though the suspect is arrest. wanted for a minor crime, the suspect is resisting using deadly force against the officer. ANSWER: D Even though it is a minor offense , deadly force is justified in self defense, to protect the officer' life. s NOTES: REF:

37. Which word means the OPPOSITE of JUDICIOUS?

Police Practice Test 2

Page 15 of 38

a. legal b. imprudent ANSWER: B NOTES: REF:

c. lawful d. criminal

38. Which word means the OPPOSITE of PREJUDICED?

a. bias b. impartial ANSWER: B NOTES: REF:

c. reason d. sorry

39. Which word means the OPPOSITE of CONCRETE?

a. difficult b. abstract ANSWER: B NOTES: REF:

c. simple d. false

40. Which word means the SAME as EXPOSE ?

a. reveal b. cover ANSWER: A NOTES: REF:

c. favor d. pretend

41. Which word means the SAME as INFLUENCE?

a. cause b. affect ANSWER: B NOTES: REF:

c. probe d. deem

Problem Analyses
It is suggested you use scratch paper to help answer these questions. Baseball Situation Questions:

J and B minor league baseball team has four pitchers, Mays, Craig, Henry and Manthy. Each of the four is best known for throwing one type of pitch: fastball, curveball, slider and screwball. Each of the four has his own style of delivery in pitching; overhand, three

Police Practice Test 2

Page 16 of 38

quarters, side arm on or underhand. * Henry is best known for the slider * Neither Craig nor Manthy uses a three-quarters style of delivery. *The pitcher who uses the underhand delivery is best known for throwing a fastball. *Manthy is best known for throwing the screwball. *Mays uses the overhand delivery.

Answer the below questions based on the above information:
42. Which of the following matches a pitcher with his best known pitch and his delivery style? a. Mays (curveball-three-quarters) d. Manthy-(curve ball-underhand) b. Henry (slider-side arm) e. Mays-(screw ball-sidearm) c. Craig-(fast ball-underhand) ANSWER: C Answer/Key With this problem, create a three column chart listing pitchers, pitches and styles of delivery. The information given can easily be charted and diagrammed as follows: Pitches ______ H Manthy Mays Pitch ______ SL F SC Delivery ______ U O

Now using the process of elimination and the given information you can complete the diagram/chart like this: Pitches Pitch Delivery ______ ______ ______ H SL T C F U Manthy SC S Mays C O
Answers and explanations:

NOTES: REF:

Answer is C (Obtain this answer from the chart )

43. During a game, if the starting pitcher is ineffective he will be replaced by another pitcher. All the following are possible pitching changes EXCEPT: a. The curveball picture being d. The slider pitcher being replaced replaced by the pitcher who uses the by the picture who uses the side overhand delivery. arm delivery.

Police Practice Test 2

Page 17 of 38

b. The screw ball pitcher being e. Craig being replaced by the curve replaced by Henry. ball pitcher. c. Mays being replaced by the fastball pitcher. ANSWER: A A is the correct answer because A cannot occur, because the curveball pitcher and the pitcher who uses the over hand delivery are one and the same. This difficulty does not apply to any of the other choices. (Refer to the chart on answer key 42) NOTES: REF:

44. In a four games series, the manager of the team decides to pitch the fastball pitcher first, the pitcher who uses the three-quarters delivery second, curve ball pitcher third and the pitcher who uses the side arm delivery fourth. In which order will pitchers appear? a. Manthy, Craig, Mays, Henry d. Craig, Mays Manthy, Henry b. Craig, Henry, Mays, Manthy e. Mays, Henry, Manthy, Craig c. Mays, Craig, Henry, Manthy ANSWER: B (Refer to the chart on answer key 42) NOTES: REF: PROBLEM ANALYSES Situation:

Ivan, Jack, Ken, Larry, Mike, Nancy, and Sue Seven police officers, Ivan, Jack, Ken, Larry, Mike, Nancy, and Sue, apply for training positions at the police academy. There are seven training positions available, in two different fields, firearms and search and seizure. Each field may take either three or four officers. The police academy has to consider the following: * Ivan and Jack may not work in the same training fields. * Nancy and Larry must work in the same training field. * If Jack and Ken work in the same training field, there must be four in that field. * Mike must work in search and seizure. * Firearms may have only four officers IF Jack is one of them.

45. Which of the following is a possible placement of the officers? FIREARMS a. Jack, Ken, Ivan, Sue b. Jack, Nancy, Larry c. Ivan, Ken , Larry Sue d. Jack, Ken, Sue SEARCH & SEIZURE Larry, Mike, Nancy Ivan, Ken, Mike, Sue Jack, Mike, Nancy, Ivan, Larry, Mike,

Police Practice Test 2

Page 18 of 38

Nancy e. Ivan, Ken, Larry, Nancy

Jack, Mike, Sue

ANSWER: B ( use the process of elimination) Choice A is incorrect because Jack and Ivan may not work together in the same field. C is incorrect because Nancy and Larry must work in the same field. D is incorrect because if Jack and Ken work together, then that training field must have four officers. E is incorrect because firearms may not take four officers without Jack. Therefore B is the only option that can be true.

Given Statements And Diagrams
Officers: Ivan, Jack, Ken, Larry, Mike, Nancy, and Sue *Jack and Ivan cannot work together *Nancy and Larry must work together *If Jack and Ken work together, then there must be four officers in that training field *Mike must work in the search and seizures *Firearms may only have four officers if Jack is one of them. FIREARMS either 3 or 4 officers may have 4 with Jack must have 4 with Jack & Ken Jack & Ivan cannot work together together Nancy & Larry must work together together SEARCH AND SEIZURE either 3 or 4 officers may have 4 with Jack must have 4 with Jack & Ken Jack & Ivan cannot work

Nancy & Larry must work Mike must work in S&S

NOTES: REF:

46. If Jack works in firearms then which of the following statements are true? a. Ken works in firearms b. Sue works in firearms c. Nancy may not work in search and seizure d. Larry and Sue work in the same training field e. Sue and Ivan work in the same training field

ANSWER: C (use the process of elimination) If Jack works in firearms, then Ivan and Mike must both work in search and seizures. Since Ivan and Jack may not work together and Mike must always work in search and seizure. Nancy and Larry who must work together may not work in a search and seizure, since that would give search and seizure, four officers without Jack. As a result firearms will have Jack, Nancy and Larry and search and seizure must have Mike and Ivan. Ken and Sue will then be split into two different

Police Practice Test 2

Page 19 of 38

NOTES: REF:

groups, one working for firearms and the other one for search and seizure. C is the correct answer and the only statement that can be true. The other statements must be false.

47. If Nancy is one of the four officers placed in search and seizures, which of the following statements could be True?

a. Ivan works in search and seizure d. Ken works search and seizures b. Larry and Sue work in the same e. Ken and Sue both work in firearms field c. Jack and Sue both work in firearms
ANSWER: E If Nancy works in search and seizure, then Larry also must work in search and seizure.Mike must also work in search and seizure. That leaves one more officer to work in the search and seizures , with the remaining three to work in firearms. Jack and Ivan cannot both work in firearms, because they are not allowed to work together. Jacking and Ken cannot work in firearms because if they were to put them both in firearms there would be 4 officers in firearms. The only way Jack can be separated from Ivan is to have Jack work in search and seizures and Ivan, Ken and Sue work in firearms. As a result E is the only true statement. NOTES: REF: READING COMPREHENSION PASSAGE:

While working the 8:00 a.m. to 4:00 p.m. shift on January 14, 1999, Police Officers Marks and Smith received a radio call at 1:45 p.m. to investigate a report of a man with a gun in front of 103 Lexington Avenue. Mary Parks had called 911 from home at 1:43 p.m. and explained that two days ago while on the way home from work, she had been threatened by a man with a gun in front of her home at 113 Lowell Street. She told the police operator that the same man now was standing in front Harry' Lounge at 103 Lexington s Avenue. She described him as being 30 to 40 years of age, 5 ' , 165 lbs, wearing a gray 6" coat, a gray baseball cap and eye glasses. The officers responded to the location and observed a male fitting the description given by Mrs. Parks. The officers approached the suspect and while searching his right front waistband, Officer Marks found a 38 caliber revolver, that was licensed and registered under the name of Joseph Jackson. Mrs. Parks was brought to the scene and identified the suspect as the person who had threatened her. Officer Smith then place the man, identified as Joseph Jackson under arrest.

48. On what day did the suspect threaten Mrs. Parks?

a. January 10th b. January 12th

c. January 14th d. January 16th

Police Practice Test 2

Page 20 of 38

ANSWER: B Answer Key for 48 through 51: The easiest way to answer these questions is to Read The Questions First and then look for the answers in the passage. You might want to jot down keywords from each question such as; 1. When was Parks threatened? 2. Where did Marks recover the gun? 3. Where was Parks threatened? 4. Jackson arrested because? By jotting down the keywords in the questions it will make it easier for you to find the answers in the passage.

NOTES: REF:

49. Officer Marks recovered the gun from the suspect’s:

a. left front waistband b. Right rear waistband ANSWER: D NOTES: REF:

c. Left rear waistband d. Right front waistband

50. Mrs. Parks stated that the suspect had threatened her in front of ...

a. Harry' Lounge. s b. her home. ANSWER: B NOTES: REF:

c. a bar. d. the police station.

51. Joseph Jackson was arrested because he...

a. possessed an unregistered gun. b. pointed a gun at the bartender. ANSWER: D NOTES: REF:

c. was wanted for robbery. d. was identified as the suspect who threatened the victim.

52. Officer Johnson arrested an elderly man for loitering. Officer Johnson doesn' handcuff t the elderly man before he places him into the paddy wagon. This is a... a. good policy, because in general, elderly people should not be c. bad policy, because Officer Johnson is subject to a greater

Police Practice Test 2

Page 21 of 38

handcuffed. potential risk of being assaulted. b. good policy because under the d. bad policy, because Officer circumstances Officer Johnson has Johnson would appear too soft or full control of the situation. lacking control in the way he handled the incident.
ANSWER: C The best policy is handcuffing all suspects because the officer is protected from being assaulted and the suspect is deterred from escaping. This holds true of all persons arrested, including elderly people. NOTES: REF:

53. Officer Jackson comes across a man who is unconscious, in an alley. Further investigation reveals that the subject has an empty syringe in his hand. Officer Jackson attempts to wake the man but is unable to do so. What should Officer Jackson do next? a. Ignore the situation and let the man c. Take the man into custody and wake-up on his own accord. transport him to nearest the detoxification treatment center. b. Arrest the man for possession of d. Call for an ambulance immediately illegal drug paraphernalia. to transport the subject to medical facility. ANSWER: D Selection D is the only reasonable choice considering the fact that the man could not be awakened, and the presence of the empty syringe which gives the indication of drug use. Including possible drug overdose. Officer Jackson should assume that this is a life-threatening situation. Also, taking into consideration the most important police priority is protection of life and limb, calling for an ambulance is the only option. Also, checking for vital signs and performing first aid should be factored in. NOTES: REF:

54. As an officer on routine patrol you observe what appears to be a drug deal going down. You observe two individuals in a shopping mall exchange drugs for money. Both suspects observe you and start to run away in opposite directions. What should you do? a. Give a description of the suspects on the police radio and chase the suspect who has the money. b. Fire a warning shot and order the suspects to halt. c. Write down a complete description of the suspects, and attempt to seek a warrant for their arrest d. Give a description on the police radio of the suspects involved and chase the suspect with drugs.

ANSWER: D It' easier to build a case against someone who has possession of illegal drugs s

Police Practice Test 2

Page 22 of 38

NOTES: REF:

than it is to build a case against someone in possessions of money, alone. It' not s illegal to be in possession of money. The officers best chance of apprehending the suspects would be by giving immediate chase and giving the dispatcher and other officers a description of the suspect(s) being chased. This would allow other officers in the area to help apprehend the suspects. Firing warning shot would present too much of a risk in regards to the general public.Firing warning shots in most jurisdictions is not an acceptable practice.

55. While you' on duty a young female approaches you and tells you that a street vendor re has ripped her off for $45. She demands that he be arrested immediately. What should you do? a. Approach the vendor and demand that he returns the $45 to female. c. Conduct further investigation by talking to the street vendor in question and then decide what action should be taken. b. Explain to the woman there' s d. Place the street vendor under arrest nothing you can do because this is a immediately. civil matter and the police only handle criminal matters. ANSWER: C Always remember there are two sides to a story. Don'jump to conclusions and t don'immediately take sides of one party without finding out what the other party t has to say. You should find out the entire story from both sides. Only after conducting an investigation should you decide on what steps to take next. NOTES: REF:

56. You and your partner arrest a suspect for domestic abuse. You feel your partner failed in several regards to adequately assist you in this arrest. You felt his lack of action placed you in unnecessary danger. How should you handle the situation? a. Report your partners lack of action c. File a written request for a different to a supervisor. partner. b. Confront the officer directly and d. None of the above discuss the problem. ANSWER: B Confronting your partner directly is the best course of action at this point. If the question indicated that this was a continued problem then A would have been the best response. In most cases by confronting the person directly you can resolve these type of problems quickly. If the problem continues then you take the next step, which is to report it to a supervisor. NOTES: REF:

57. An officer conveys two prisoners to the station house to be booked. After dropping them off, he finds a $20 dollar bill in the back seat of the patrol car. What should the officer

Police Practice Test 2

Page 23 of 38

do ? a. Ask both prisoners if they lost the money. b. Keep the money.

c. Give the money to your immediate supervisor and request that it be donated to the Police Athletic League d. File a formal written report indicating how the money was found and place the money on inventory.

ANSWER: D The Officers best course of action in this incident is to place the money on an inventory report and file a report on how the money was found. This would protect him/her or the police department from any claims of theft. Any other action would be unethical and inappropriate. NOTES: REF:

58. Officer Roberts observers two men fighting. One of the men stabs the other in the chest with a knife. The victim falls to the ground and he is bleeding profusely. Under the circumstances, what should Officer Roberts do? a. Pursue the suspect and attempt to c. Attempt to slow bleeding and call arrest him. for an ambulance . b. Radio for an ambulance and attempt d. None of the above to locate witnesses. ANSWER: C It' obviously important to call for an ambulance ASAP in this situation. Although, s the FIRST priority is to stop the bleeding otherwise the victim could bleed to death before an ambulance arrived. NOTES: REF:

59. While on routine patrol, Officer Reynolds observers a downed power line in the roadway. What should Officer Reynolds do regarding this situation? a. Move the wire to the side of the c. road where it will not create a hazard to traffic. t b. Don'touch the wire, secure the area d. and divert the traffic away from the power line and contact the Electric Utility Service

Don'touch the wire, place a traffic t cone by it to divert traffic and contact the Electric Utility Service None of the above

ANSWER: B Power lines are dangerous and should not be left unattended. An Officer should not make any attempt to move it himself/herself. This should be left up to the Electric Utility Company personnel. NOTES: REF:

Police Practice Test 2

Page 24 of 38

60. Which of the following crimes would be least likely affected by increased police presence?

a. burglary b. criminal trespass

c. murder d. robbery

ANSWER: C Murder in most situations is a spontaneous event, occurring without premeditation. The other crimes usually present a pattern that can be deterred by additional police presence. NOTES: REF:

61. While on patrol Officer Matthews discovers a fire on the first floor of a two-story apartment building. What should Officer Matthews first response to the fire be? a. Do whatever he can to extinguish the fire. b. Enter the building and attempt to alert the occupants of the fire and help evacuate the building c. Immediately contact the fire department and use the PA system in patrol car to alert the occupants of the fire. d. Call the Fire Department and check for witnesses.

ANSWER: C Alerting the Fire Department immediately is the best course of action. They' re better equipped and more qualified to handle these types of situations. The safety of the occupants is a much higher priority than checking for witnesses eliminating D. Alerting the occupants using the PA system is an acceptable practice. NOTES: REF:

62. 99 + 88+ 77+ 66+ 55 =

a. 384 b. 485 ANSWER: C NOTES: REF:

c. 385 d. 345

63. Officers must be accurate yet brief when broadcasting on the police radio. What is the best reason for the above statement? a. It makes officers sound more professional b. It saves the department money on radio air time expense c. Tying up the airwaves with long messages on the police radio may prevent emergency transmissions from getting through. d. Shorter messages are easier to understand.

Police Practice Test 2

Page 25 of 38

ANSWER: C It' difficult for Officers to get on the police radio with important and emergency s transmissions at times because of the busy Airways. That' why offices are told to s keep their messages short and brief. NOTES: REF:

64. Officer Smith is sent to a silent burglary alarm at a warehouse at 3 am. Officer Smith should take the following steps when responding to the alarm, EXCEPT: a. Make a cautious approach to the c. Approach the building with your building, using stealth so as not to red lights and sirens activated, so alert the suspects. you can a arrive at the building as soon as possible . b. Attempt to contain the building d. Do not allow the owner of the upon your arrival until further help building or any employees to enter arrives the building until the premise has been searched. ANSWER: C This is a common sense question. By approaching with your red lights and sirens activated you would obviously alert any of the suspects and therefore defeat the purpose of a silent alarm. All the other answers are correct procedures to take when responding to burglar alarms. NOTES: REF:

65. Officer Jones sees a motorist commit a traffic violation at a construction site where street signs have recently been changed. What action should Jones take? a. Write down license plate and if he c. Stop the offender and issue a sees the motorist commit this warning. violation again issue him/her a citation. b. Ignore the incident because of the d. Stop the offender and issue a recent change. citation. ANSWER: C Here again common sense plays a factor. In this situation issuing a warning would be the best course of action. Based on the fact that a recent change in traffic signs has occurred. This mistake can happen to anybody. It would be an officer' job to s bring it to the motorist' attention by stopping him and giving him a warning. s NOTES: REF:

66. While attending a high school football game off-duty, Officer Smith observers some kind of incendiary device connected to the timer beneath the bleachers. What is Officer Smith' best course of action at this time? s a. Attempt to disarm the device. c. Notify the stadium officials and have them stop the game and evacuate the public, then call the bomb squad.

Police Practice Test 2

Page 26 of 38

b. Attempt to evacuate the area by d. Call the bomb squad and let them telling everybody there' a bomb in s handle the entire matter. the stands. ANSWER: C Always remember the number one priority is safeguarding human life. By announcing a bomb in the stands to the crowd you may create mass hysteria and unsafe evacuation. In most situations like this stadium officials have a game plan on safe evacuation. NOTES: REF:

67. Officers are told not to appear in places within their patrol areas at regular intervals. What' the best reason for this policy? s a. Staggering inspection times gives c. Staggering patrols makes it more difficult for the criminal element to the public the impression that there are more officers then there really predict when and where the police are. will be. b. Regular routines inspire d. None of the above. complacency. ANSWER: C A regular routine by officers who patrol a particular area can help the criminal element predict an officer' time schedule enabling them to commit certain crimes s easier. NOTES: REF:

68. An officer should know how to take a good defensive position when conducting field interviews to help ward off unexpected attacks. All the following are considered good defensive posture, EXCEPT: a. feet moderately spread and staggered b. legs and back kept straight c. knees someone bent

d. lowering of the buttocks

ANSWER: B All the answer choices are correct except B. If your legs and back are straight it would be easier for someone to knock you off balance. NOTES: REF:

69. Officer Reynolds arrives at the scene of a disturbance in an abandoned warehouse. She notices a van leaving the site. Her investigation reveals the warehouse had been vandalized. Officer Reynolds will file a report regarding this incident. What is the most effective way Officer Reynolds should indicate in her report her observations of the van and leaving the scene? a. A van which should not have been c. The suspects escaped in a van. there, was at the warehouse. b. Upon my arrival at the warehouse, I d. All the above observed a van driving away from

Police Practice Test 2

Page 27 of 38

the site.
ANSWER: B Answers A and C are incorrect because they provide details of which have not been substantiated. Based on the information given, the only thing Officer Reynolds should put on her report is she observed a van leaving the scene. Remember, JUST THE FACTS. NOTES: REF:

70. Officer Miller has completed an investigation of a traffic accident. Officer Miller is about to file a report regarding the accident. The following contains sentences that will be on Officer Miller' report. s What is the correct order the sentences should be in? 1.The Ford struck the right rear fender of Mr. Ricks Dodge and continued on its way. 2. Mr. Ricks stated that he was making a left turn from 35th Street onto 2nd Street. 3. Mr. Ricks stated he observed the license plate of Ford and it was XZY163. 4. Mr. Ricks complained of back pain as a result of the accident and was conveyed by ambulance to St. Luke' Hospital. s 5. A Ford with license plate number XZY163 was observed traveling north on 2 Street when it went through a red light on 35 St.

a. 1,3,2,5,4 b. 2,5,1,3,4

c. 4,5,1,2,3 d. 5,3,1,2,4

ANSWER: B Time sequence is the key. To establish the correct order of the sentences in the report you must determine what order the events took place. The order of events indicates that correct answer is B (2,5,1,3,4). NOTES: REF:

PASSAGE: Travel Agents
Travel agents assist prospective travelers in a number of ways. They provide information and suggestions regarding travel plans, make transportation and hotel reservations, and consult with clients regarding other details associated with traveling, such as passport regulations, foreign currency, and hotel rates. While an agent who works in a small office usually performs all of these duties, some, who work in large agencies with many employees specialize in a single function. Because travel agents make many detailed reservations, they must be capable of the utmost accuracy. Some experienced and highly skilled agents develop package tours. This involves planning an itinerary, finding a tour leader, making travel and hotel reservations, and publicizing the tour. Sometimes travel agents serve as tour guides. This gives them a chance to acquire personal travel experience and to become thoroughly acquainted with people who may be potential customers for future trips. 71. READING COMPREHENSION

Police Practice Test 2

Page 28 of 38

Travel agents help their clients to:

a. develop package tours b. make travel plans ANSWER: B NOTES: REF:

c. meet other travelers d. serve as tour guides

72. According to the passage, travel agents must be especially capable of accuracy when they:

a. convert foreign currency b. serve as tour guides ANSWER: D NOTES: REF:

c. arrange publicity d. make reservations

73. In general, the passage describes a travel agent' s: a. work environment c. method of making detailed reservations d. duties and experiences

b. education and training ANSWER: D NOTES: REF:

74. Situation: Police officers are to call for a supervisor in the following situations; Whenever force is necessary in making an arrest. Whenever a shooting occurs. All assignments where the suspect is armed. All traffic accidents involving life-threatening injuries. All accidents involving Department vehicles. ------------------------------------------------Based on the above information; A supervisor should be called to the following assignments, EXCEPT: a. armed robbery in progress b. burglary in progress c. traffic accident involving a car and a bicycle where the bicyclist has head injuries d. arrest of a burglary suspect who resisted and was pepper sprayed

ANSWER: B It' the only situation where a supervisor would not be called. Using the process of s elimination, A can be eliminated because it involves an armed suspect. C can be eliminated because it includes life threatening injuries. D can be eliminated because it involves the use of force. NOTES:

Police Practice Test 2

Page 29 of 38

REF:

Problem Analyses
Classroom Questions Situation: An assistant principal must assign classes K,L,M,N,O,P and Q to the 7 classrooms lining the North corridor of his high-school. The rooms are numbered 1 through 7 from east to west. There is a broom closet between rooms 4 and 5. Only 1 and 7 have taping facilities. L and M may not be adjacent rooms. N and O must be adjacent rooms. L must be in room 3 Language classes must be in rooms with taping facilities. Start making a list and a diagram. 1 2 ____ _____ (taping ) 3 ____ L 4 ____ BC ____ 5 ____ 6 ____ 7 _____ (taping)

LIST GIVEN INFORMATION --------------------------------------------------------L not adjacent M N is adjacent to O L is in room 3 Language classes are in rooms with taping facilities. Rms 1 & 7 have taping facilities A broom closet between 4 and 5 ----------------------------------------------------------------In this exercise, you' need to list the known facts for every individual question. ll Every question presents a different set of circumstances. Note: Keep the following in mind when answering the questions: You' notice the ll questions don' supply you with concrete information. You' not reminded about some t re important factors given in the stem/situation. Such as: Where' the language classes? s 1 2 ____ _____ (taping ) (Lang) 3 ____ L 4 ____ BC ____ 5 6 ____ ____ 7 _____ (taping) ( Lang)

The challenge is keeping track of special conditions. IE: broom closet, language classes, taping rooms.
----------------------------------------------

Police Practice Test 2

Page 30 of 38

75. If K and M are in 2 rooms adjacent to the broom closet and if O is in room 2 , which the following must be true?

a. N is in Room 7 d. N and P are in are adjacent rooms b. P and M are in are adjacent rooms e. P is in a room with taping facilities c. Q and P are in adjacent rooms
ANSWER: C The stem of this question hints that K and M are interchangeable between the 4th and the 5th spot. But that' not possible. M can'be 4th because it would be right s t next to L, which is forbidden. So M is 5th and K is 4th. O is second which means M must be 1st. ( 0=N) , leaving Q in either order. The answer is C. E could be true but A & D cannot be true. 1 2 ____ ____ N O (taping) 3 4 ____ ____ L K BC ___ 5 ____ M 6 ____ Q 7 _____ P (taping)

NOTES: REF:

76. Situation: If M and N are both language classes and if O and L are in adjacent rooms, it must be true that M is in:

a. room 1 b. room 7 c. a room adjacent to Q' room s

d. a room adjacent to O' room s e. a room adjacent to K' room s

ANSWER: B Since rooms 1and 7 are taping rooms, the first if-clause in this question means that either M is first and N is seventh or vice versa. However, you see that Vice versa is what it must be. N seventh would require that O be in 6, yet doing so does not allow O to be adjacent to L which the stem specifies. The only possible combination is : 1 2 ____ _____ N O (taping) 3 ____ L 4 ____ BC ____ 5 ____ 6 ____ 7 _____ M (taping)

NOTES: REF:

You' notice these are the only placements that must be true, which makes the ll correct answer B. C & E are possible while A is impossible.

77. If K and O are both in the language classes, and if M is not in a room adjacent to the broom closet, all of the following must be true; EXCEPT:

a. O is in room one b. N is in room 2

d. P and L are in adjacent rooms e. Q is in a room adjacent to the broom closet

Police Practice Test 2

Page 31 of 38

c. M is in Room 6
ANSWER: D The question starts by telling us that K and O are either first or seventh or seventh and first, respectively. Where ever O is N must be next to it, so we can set up the options: 1 ____ K OR 1 ____ O 2 _____ 3 ____ L 4 ____ 5 ____ 6 ____ N 7 _____ O

___ BC

2 _____ N

3 ____ L

4 ____

____ BC

5 ____

6 ____

7 _____ K

The first option must be eliminated, if M is prohibited from being next to the broom closet, M can'be 5th and M can'be 2nd or 4th. Option 2 is the only t t prospect, as M must be 6th for reasons discussed. P and Q, then will occupy the position next to the broom closet, though we don'know in which order. The t answer is D: D cannot be true.

NOTES: REF: 78. Situation: Officer Jackson has issued a summons to a driver and has obtained following information: Place of occurrence: Corner of Foster Rd and Woodrow Avenue Time of occurrence: 7:10 p.m. Driver: William Grant Offense: Driving through a red light Age of driver: 42 Address of driver: 23 Richmond Avenue -------------------------------------------------Officer Jackson is making an entry into his memo book regarding the incident. Which one of the following expresses the above information most clearly and accurately? a. William Grant lives at 23 Richmond c. William Grant age 42, was issued a Avenue at 7:10 PM went through a summons on the corner of Foster red light. He was issued a summons Road and Woodrow Avenue for at the corner of Foster Rd and going through a red light. He lives Woodrow Avenue. The driver is 42 that 23 Richmond Avenue at 7:10 years of age. p.m. b. William Grant, age 42 who lives at d. A 42 year-old man who lives at 23 23 Richmond Avenue, was issued a Richmond Avenue, was issued a summons for going through a red summons at 7:10 p.m. William light at 7:10 p.m. at the corner of Grant went through a red light at Foster Road and Woodrow Avenue. the corner of Foster Rd and Woodrow avenue.

Police Practice Test 2

Page 32 of 38

ANSWER: B B. provides the clearest description of the events in this scenario. NOTES: REF:

79. Situation: Officer Kent has completed an investigation regarding a report of an auto theft and obtained the following information: Date of occurrence: October 26, 1988 Place of occurrence: 51St and 8th Ave Time of occurrence: 3:30pm Crime: Auto theft Suspect: Michael Wadsworth The action taken: Suspect arrested ------------------Which of the following expresses the above information most clearly and accurately? a. Arrested on October 26, 1980 was a c. stolen auto at 51 street and 8th Avenue at 3:30 p.m. driven by Michael Wadsworth. b. For driving a stolen auto at 3:30 d. p.m. Michael Wadsworth was arrested at 51st Street and Eighth Avenue on October 26, 1988. On October 26, 1988 at 3:30 p.m. Michael Wadsworth was arrested at 51st Street and 8th Avenue for driving a stolen auto. Michael Wadsworth was arrested on October 26, 1988 at 3:30 p.m. for driving at 50 St and 8th Ave. The auto was stolen.

ANSWER: C C provides the clearest description of the events in this scenario. NOTES: REF:

80. Which word means the OPPOSITE of REVERENCE?

a. loyalty b. sane ANSWER: C NOTES: REF:

c. disrespect d. balance

81. Which word means the OPPOSITE of AUDIBLE?

a. optical b. planned ANSWER: D NOTES: REF:

c. easy d. silent

82. Which word means the OPPOSITE of DETAIN?

Police Practice Test 2

Page 33 of 38

a. hold b. release ANSWER: B NOTES: REF:

c. repeat d. fail

83. Which word means the OPPOSITE of CHRONIC?

a. infrequent b. fatal ANSWER: A NOTES: REF:

c. forever d. seldom

84. Which word means the OPPOSITE of CERTAIN?

a. ambiguous b. almost ANSWER: A NOTES: REF:

c. reveal d. predict

85. Which word means the OPPOSITE of MAR?

a. break b. confuse ANSWER: C NOTES: REF:

c. repair d. incur

86. Which word means the OPPOSITE of LUCID?

a. unclear b. apathetic ANSWER: A NOTES: REF:

c. ordinary d. bolster

87. Which word means the SAME as UNINTELLIGIBLE?

a. insane b. garbled ANSWER: B NOTES:

c. retain d. savor

Police Practice Test 2

Page 34 of 38

REF: 88. Which word means the SAME as HIDEOUS?

a. hidden b. uncovered ANSWER: C NOTES: REF:

c. grotesque d. famous

89. Which word means the SAME as COMPEL?

a. rely b. coerce ANSWER: B NOTES: REF:

c. repent d. release

90. While conducting a burglary investigation, Officer Jones interviews several witnesses. All the witnesses observe a vehicle leaving the scene of the burglary. Which of the description offered by the witnesses should Officer Jones consider most likely to be correct? a. A brown GMC truck, License plate c. A Brown Ford truck, License # G-1843 plate# G -1845 b. A black Ford truck, License plate d. A Brown Ford truck, License plate #G-1854 # C-1845 ANSWER: C If you look at all the description you' notice answer C has the most details in ll common with the other descriptions. NOTES: REF:

READING COMPREHENSION
PASSAGE: Officer Harris, after studying the crime trend analysis data noticed a significant increase in the number of certain crimes in his patrol area over the past three months. The data indicated that most of the rapes took place on East 98 Street between Lott Avenue and Herk Place: assaults took place on Lott Avenue between Chester Avenue and East 98 Street: and the majority of the robberies occurred on Lott Avenue between 98th Street and Hughes PLace. The assaults took place between 1:00 a.m. and 3:00 a.m.. All the robberies happen between 1:00 a.m. and 6:00 a.m. and most of the rapes occur between 8:00 a.m. and 11:00 a.m.. The rapes usually occur on Mondays and Wednesdays, the robberies on Fridays and Saturdays and the assaults on Saturdays and Sundays.

91. Based on the above information answer the following question. Officer Harris would most effectively reduce the number of robberies by patrolling...

Police Practice Test 2

Page 35 of 38

a. Lott Avenue between E. 98th Street c. and Hughes Pl. on Fridays and Saturdays between 1:00 a.m. and 8:00 a.m. b. Lott Avenue between East 98th d. Street and Chester Avenue on Saturdays and Sundays between 1:00 a.m. and 6:00 a.m.

East 98th Street between Lott Avenue and Herk Place on Saturdays and Sundays between 1:00 a.m. and 3 a.m. East 98 Street between Herk Place and Chester Avenue on Mondays and Wednesdays between 8:00 a.m. and 11:00 a.m.

ANSWER: A The times and areas listed in this answer choice would most closely correspond with the crime analysis data. NOTES: REF:

92. Officer Harris has been informed by supervisors that he will be assigned to a patrol area each week that would allow him to concentrate on reducing the number of rapes. What would be the most appropriate patrol for Officer Harris to work? a. Tuesdays through Saturdays, 8:00 c. p.m. to 4:00 p.m. b. Monday through Friday, 7:30 a m to d. 3:30 p.m. Wednesday through Saturday, noon to 8:00 p.m. Monday through Friday 3:00 p.m. to 11:00 p.m.

ANSWER: B Although these are not the exact times the crime analysis report listed it is the closest and most appropriate times of occurrence in comparison with the other times listed.

The best way to answer these questions is to create a diagram or a list of all the pertinent information described in the passage.
Example:

Rapes East 98 St. between Lott Avenue and Herk Place between 8:00 a.m. and 11:00 a.m on Mondays and Wednesdays

Assaults on Lott Avenue between Chester Avenue and East 98 St between 1:00 a.m. and 3:00 a.m on Saturdays and Sundays

NOTES:

Robberies on Lott Avenue between 98th Street and Hughes PL between 1:00 a.m. and 6:00 a.m. on Fridays and Saturdays

Police Practice Test 2

Page 36 of 38

REF: 93. In order to conceal her guilt, Linda told a blatant lie.

a. harmless b. subtle ANSWER: D NOTES: REF: 94. 837+ 415 minus 1035 =

c. careless d. obvious

a. 332 b. 245 c. 213 ANSWER: D NOTES: REF: 95. What' 526 X 317 ? s

d. 217 e. 226

a. 154,298 b. 166,742 c. 166.244 ANSWER: B NOTES: REF:

d. 266, 742 e. 66, 742

96. Police officers are required to remove potentially dangerous property from prisoners prior to placing them into jail cells. Which the following should be considered a potentially dangerous property?

a. an empty wallet b. shoelaces

c. cough drops d. a wedding ring

ANSWER: B This is another common-sense question. Prisoners can use shoelaces to hang themselves. NOTES: REF:

97. Answer the following question based on the following information: Situation: When a OFFICER makes an arrest, the officer should do following in the order given: 1. Advise the person he/she is being arrested.

Police Practice Test 2

Page 37 of 38

2. Handcuff the prisoner' hands behind his back. s 3. Search the prisoner and the nearby area for weapons and evidence. 4. Advise the prisoner of their constitutional rights before questioning him/her. -----------------------------------Police Officer Silver arrests Larry Tims for robbing a grocery store . He informs Mr. Tims that he is under arrest. He then handcuffs Mr. Tims, placing the handcuffs behind Mr. Tim' back. While handcuffing Mr. Tims Officer Silver notices a s small handgun in Mr. Tims back pants pocket. The next step Officers Silver should take is to: a. Inform the prisoner of his rights b. Question the prisoner c. Return, with the prisoner to lead to the serve store d. Search the prisoner and the immediate area for weapons and evidence

ANSWER: D The next step according to the information given is, search the prisoner and the immediate area for weapons and evidence. NOTES: REF:

98. Civilization started to move ahead more rapidly when people freed themselves of the shackles that restricted their search for the truth. The sentence above best supports the statement that the progress of civilization a. came as a result of people’s dislike c. has been aided by people’s efforts for obstacles. to find the truth. b. did not begin until restrictions on d. is based on continually increasing learning were removed. efforts. ANSWER: C The search for truth has sped up the progress of civilization. Civilization moved ahead slowly and began even before restrictions on learning were removed. NOTES: REF:

99. Irresolute means:

a. wavering b. insubordinate

c. impudent d. unobservant

ANSWER: A Irresolute means indecisive or wavering. “The couple was irresolute as to the choice of next summer’s vacation.” NOTES: REF:

Police Practice Test 2

Page 38 of 38

100. Jim is arrested in a criminal case. He refuses to answer any questions about the alleged crime, claiming that he is “taking the Fifth Amendment.” “Taking the Fifth Amendment means that... a. Jim is pleading innocent to the charges against him. c. Jim does not have to answer questions that might incriminate him. b. Jim has not had the opportunity to d. Jim is too young to be arrested for discuss the case with a lawyer. this crime.

ANSWER: C

Police Practice Test 3

Page 1 of 34

Police Practice Test 3 Answer Key
Multiple Choice Identify the letter of the choice that best completes the statement or answers the question.

1. Police officers have the discretion to give warnings instead of citations at traffic stops. Which of the following situations would a warning be more appropriate to give than a citation? a. A man runs through a stop sign because there's no other cars. b. A woman is late for work and is caught driving 10 mph over the speed limit. c. A man has a heart attack and runs through a stop sign causing a minor accident. d. A woman unfamiliar with the area makes a left turn disregarding a no left turn sign.

ANSWER: C This is an obvious common sense answer. A person having a heart attack has no control of their actions. Choice d could be another situation where a warning could be issued, especially if the person was from out of State. Although it's expected, people are to be alert to all traffic signs. Driving in another state can a bit of an adjustment and should be taken into consideration. NOTES: REF:

2. A car was seen leaving the scene of a burglary. Witnesses observed the license plate. Which of the following is most likely the correct license plate number given by the witnesses?

a. TLC 519 b. BIU 559

c. BIU 519 d. BIC 519

ANSWER: C The correct answer is C. C, as it contains the most commonly repeated letters and numbers offered by the witnesses. NOTES: REF:

3. A witness observes an Armed Robber run out the door of a gas station. Which part of the witness' description would be most useful in identifying the suspect? a. The suspect was wearing a green d. The suspect had a 2" inch scar on jacket. his left cheek. b. The suspect had on a blue baseball e. The suspect walked with a limp. cap.

Police Practice Test 3

Page 2 of 34

c. The suspect had a large nose and brown eyes. ANSWER: D The correct answer is d. Like most difficult questions this has two choices that could be correct. e would also be a reasonable choice. However, d is the better choice and here's why; Although both a limp and a scar are descriptive elements that are very distinguishable, a 2" scar on the left cheek is even more unusual than a limp. Also, a limp may be temporary where a scar is permanent short of plastic surgery. NOTES: REF:

4. Officer Miller is advised by his sergeant that a shopping Mall in her area had a rash of auto thefts reported in the last two weeks. Officer Miller indicates she will intensify patrol in the area. Regarding the auto thefts what situation observed in the Mall by Miller should be investigated? a. Two juveniles sitting on a parked c. A man and woman sitting in a car smoking cigarettes. parked car with the engine running. b. A man walking in the parking lot d. A juvenile walking through the pulling on the handles of several parking lot, carrying a shopping car doors. bag. ANSWER: B The correct answer is b. The man trying several car doors constitutes the most suspicious activity in regards to auto theft complaints. The other answer choices indicate activity that is commonly seen at shopping malls. NOTES: REF:

5. Officer Maxwell and Timms respond to a domestic dispute. Neighbors called police and stated they heard a man and women arguing. They heard the man threaten his wife and thought he was going to beat her. The officers arrive at the scene, and notify the dispatcher of their location. They then exit their squad and approach the house. What should they do next? a. Talk to the neighbors who called toc. get more information. b. Knock on the door and request d. entry. ANSWER: C Talking to the neighbors should be done afterwards unless they want to remain anonymous. To do so before hand would waste valuable time in the event there is an assault going on. Stop, look, listen and evaluate should always dictate your actions. Answer c is the correct answer because you must know what your getting yourself into. Listen at the door to see if they can hear what's going on. Call for an ambulance to stand by in case there's anybody injured.

Police Practice Test 3

Page 3 of 34

NOTES: REF:

What you hear inside should dictate your actions. If you hear nothing it could be a false call or the people inside may resolve the problem. If you hear screaming, you may have to do a forced entry to prevent injury. Veteran officers will tell you that time and again. An ambulance would not be called until you knew there were injuries. They are just too busy to stand by at every call.

6. Officer Jackson responds to a domestic dispute and approaches the residence. Officer Jackson should... a. approach the house with his gun drawn. b. stand off to the side of the doorway before knocking. c. knock on the side or rear door of the residence. d. leave and return to patrol if all the lights are off in the residence and he doesn't hear a disturbance.

ANSWER: B An officer should ALWAYS stand off to the side of a doorway , no matter what type of call it is. There have been numerous officers shot through the doorway at minor disturbance calls. NOTES: REF:

7. Officer Lukes and Daniels respond to a family trouble. Upon approaching the house, they hear what sounds like breaking glass coming from inside the residence. What is the first thing the officers should do? a. run into the house c. advise the dispatcher that you have arrived at the scene and you hear the glass breaking inside b. listen at the door to see if they can d. call for a supervisor tell what's happening ANSWER: C Always advise the dispatcher of your arrival at a call. Just in case you know what hits the fan. The next step would be to knock on the door and identify yourself as the police. If no one answers and (or) you continue hearing a disturbance inside the residence, entry ( including a forced entry if needed) would be in order to prevent injuries. NOTES: REF:

8. After arriving at the scene of a family fight, Officer HUNT notifies the dispatcher of his arrival. When approaching the house, a women comes running out onto the porch bleeding from a deep cut on her arm screaming, "help". The next thing Officer Hunt should do is; a. run into the house after the suspect c. advise the dispatcher of the

Police Practice Test 3

Page 4 of 34

b. advise the dispatcher of the bleeding woman ,call for an ambulance and apply first aid immediately on the porch

bleeding woman, call for an ambulance, move the woman away from the house and apply first aid d. call for supervisor

ANSWER: C If you apply first aid on the porch, you may put yourself and the woman in a precarious position and subject to attack from a possible suspect with a knife.Your first concern is the safety of all parties involved. NOTES: REF:

9. Officer Hendly is sent to a purse snatch. Officer Hendly interviews four witnesses who observed the purse snatch suspect. Which of the witness's statement is probably the most accurate? a. He was medium height, husky, in c. He was short, husky, early 20s his mid 30s b. He was short, husky, in his mid d. He was short, medium built, mid 30s 30s ANSWER: B The correct answer is b. b contains the most repeated descriptive elements provided by all the witnesses. TIP: The best approach to eyewitness account questions is: Compare the similarities in each witness account and pick the answer that has the most common elements. NOTES: REF:

10. Police sometimes have to exceed the speed limit in order to perform their duties effectively. Which of the following incidents would be appropriate for a police officer to go faster than the speed limit? a. responding to a burglary that occurred the day before b. responding to a hospital to interview a victim of an assault c. pursuing a vehicle wanted in an armed robbery d. attempting to stop a speeder e. both c and d

ANSWER: E Both c and d would be situations where a police officer would have the right to exceed the speed limit. You can eliminate both a and b. No need to rush to a burglary that occurred the day before. Going to a hospital to interview a victim of an assault would NOT be a situation where a fast response time would be necessary. Most

Police Practice Test 3

Page 5 of 34

NOTES: REF:

likely the suspect is long gone and the victim is already receiving medical treatment.

11. Officer Raymonds, while conducting an investigation of a convenient store robbery, interviews 4 witnesses who observed the getaway car. Which of the descriptions is probably the right one?

a. dark green with gray roof b. dark blue with white roof

c. black with gray roof d. dark green with tan roof

ANSWER: A The correct answer is A, (dark green with gray roof) as it has the most repeated witness observations. NOTES: REF:

12. Burglar Alarms Officer Doran gets dispatched to a silent burglar alarm at Lincoln Elementary School located at 1st and Madison. When the call came, Officer Doran was finishing a traffic stop two blocks from the school. She responds to the call and parks the squad in a strategic location away from the school. What should Officer Doran do next? a. Check the building for signs of entry. b. Call for back up. c. Notify the dispatcher of her arrival and location. d. Call for a supervisor.

ANSWER: C Always let the dispatcher know, when you arrive at a call. In case the "you know what" hits the fan, the dispatcher will know where to send help. Let's face it when officers are forced to act quickly, such as chasing suspects, they're not always coherent. That's why it's important for the dispatcher to know your location. NOTES: REF:

13. Officer Winston is sent to a burglar alarm at a residence at 204 S. 10 St. It's 4pm., traffic is heavy at the time and he is about 2 miles away from the scene. Officer Winston should respond to the alarm in the following manner: a. Proceed to the scene WITH c. Proceed to the scene WITH emergency lights and siren emergency lights and siren activated to get to the scene quick activated, than turn them off far as possible enough away from the scene as not to alert the suspect. b. Proceed to the scene WITHOUT d. None of the above. emergency lights and siren activated so as not to alert the suspects.

Police Practice Test 3

Page 6 of 34

ANSWER: C The objective should be to get to the alarm as soon as possible without alerting the suspects. NOTES: REF:

14. At 2:00 am Officer Harris is sent to a burglar alarm at a shoe store at 204 N 10th St. This is the third time in 4 hours the alarm has gone off. All false alarms. The owner is on his way to the alarm. Officer Harris responds to the alarm parking a safe distance away and notifies the dispatcher of her arrival. What should Officer Harris do next ? a. Wait for the owner, it's probably another false alarm b. Call for a supervisor c. Check the building for signs of entry d. Call for a back up

ANSWER: C No matter how many false alarms occur, the officer should follow the proper procedure for handling burglar alarms. The next alarm could be the real thing and actually indicate a burglary in progress. NOTES: REF:

15. Officer Johns and Malloy were sent to a Domestic Disturbance at 1202 N. Henry. While walking up to the residence they hear breaking glass and screaming. What should the officers do next? a. Conduct a forced entry. b. Knock loudly and identify themselves as police. c. Advise the dispatcher of their observations. d. Listen at the door to see if they can tell what's happening

ANSWER: C Keeping the dispatcher advised should always be a top priority. You and your partner's safety depend on it. At this point the dispatcher can send the nearest available squad for back up. The other answer choices are steps that should be considered after The dispatcher is notified. NOTES: REF:

16. Officers Timm and Davis enter a residence of a Domestic Violence call. Inside they observe women bleeding from the arm profusely. The suspect is on the scene. They notify the dispatcher of the situation and request an ambulance. What should the officers do next?

a. Perform first aid. c. Call for a supervisor. b. Take control of the suspect(s) and d. Interview the victim. check for and secure weapons within reach of all parties in the residence.
ANSWER: B

Police Practice Test 3

Page 7 of 34

NOTES: REF:

Although it's a high prior to perform first aid, it would do you no good If while performing first aid you, your partner or the victim was attacked by the suspect or someone else in the residence with a weapon.

17. In the United States the law has imposed strict consequences on any abuse of search and seizure procedures. Therefore, evidence discovered as a result of any search that does not comply with the procedures and standards laid down by the Supreme Court and by other courts, interpreting the various amendments to the U.S. Constitution collectively known as the Bill of Rights, is not admitted in the trial, even though; a. It clearly establishes the guilt of c. The suppression of the evidence the accused person, and even may prevent the conviction of a though the suppression of the person who is indigent. evidence may prevent the conviction of a person who is innocent. b. It clearly establishes the guilt of d. It clearly establishes the the accused person, and even innocence of the accused person, though the suppression of the and even though the suppression evidence may prevent of the evidence may prevent the conviction of a person the conviction of a person who is plainly guilty. who is innocent. ANSWER: B The correct answer is b. Using the process of elimination and common sense b is the best answer. Evidence obtained without complying to the courts standards for proper search and seizure would most likely hurt the prosecution and prevent the conviction of a person who is guilty. NOTES: REF:

18. Common Sense Factors To Consider When Handling Evidence: Officer Keen, while investigating a burglary at Washington High School, observes what appears to be evidence inside one of the classrooms. He finds a crowbar and a flashlight near a broken window. Officer Keen places the crow bar into a paper bag. What should Officer Keen do next? a. Place the bag with the crowbar intoc. Put the flashlight into the bag with his squad. the crowbar. b. Seal the bag with evidence tape. d. Write his name and ID number on the bag ANSWER: B

NOTES: REF:

The correct answer is b. Seal the bag with evidence tape.

19. Officer Charles is investigating a hit and run accident at 124th and Andrews Lane. He

Police Practice Test 3

Page 8 of 34

finds a piece of a tail lamp belonging to the hit and run vehicle. He also found an auto mirror belonging to the same hit and run vehicle, several feet from the tail lamp. Officer Charles takes photos of the evidence and than proceeds to collect it. What is the first thing Officer Charles should do? a. Place both pieces of evidence in c. Place both items in a cardboard the same evidence container than box and transport them to the seal them. property division. b. Place both pieces of evidence in d. Place the items in a cardboard box separate evidence containers and and mark them with his ID seal them. number. ANSWER: B Place the items in separate containers, then mark them. NOTES: REF:

20. Officer Peters has collected evidence at a shooting scene. She placed the evidence in separate containers and sealed the containers. Officer Peters should place the following information on the evidence tag, EXCEPT; a. Her name. b. The case number. c. The date and time the evidence was collected. d. The date and time the evidence was collected. e. Name of the first officer at the scene.

ANSWER: E The correct answer is e. All the other information should be listed on the tag. The 1st officer on the scene may not be involved in the actual investigation just a back up unit. Although, the observations of the 1st officer on the scene would be valuable in court, he/she may not have anything to do with the collection of evidence. NOTES: REF:

21. Officer Krans arrests a suspect for shoplifting. While he is on route to the jail, he hears another squad broadcast a vehicle pursuit several miles away. What should officer Krans do? a. Proceed to the chase with redlights c. and siren activated. b. Proceed to the chase using normal d. traffic speed, obeying all traffic laws. Proceed directly to the jail with the prisoner. Remain at the location of the arrest in the event the pursuit comes that way.

ANSWER: C The Correct answer is c. The prisoner should be conveyed directly to a lock up facility. If the officer responds to the pursuit and the prisoner is injured, the officer and

Police Practice Test 3

Page 9 of 34

NOTES: REF:

his department would be held liable. The officer is responsible for the prisoners’ safety while he/she is in custody.

22. Officer Thomas arrests a drug dealer at 17th and Howard. He places handcuffs on the suspect, placing the handcuffs behind the suspect's back. What should Officer Thomas do next? a. Check the area of the squad where c. Search the prisoner the suspect will be seated for weapons. b. Place the suspect into the squad d. Advise the prisoner of his and seatbelt the suspect. Miranda rights. ANSWER: C The correct answer is c. The suspect at this point should be searched for weapons, contraband and evidence. Miranda rights can be given after the suspect has been conveyed to the station house, prior to interviewing the suspect. NOTES: REF:

23. When a prisoner is arrested and conveyed the officer should do the following, EXCEPT: a. Handcuff the prisoner securely. b. Search the prisoner. c. Convey the prisoner directly to a lock up facility. d. Let the prisoner contact his attorney from the location of the arrest

ANSWER: D The officer should do all of the above EXCEPT d, let the prisoner contact his attorney from the location of the arrest. The officer does not have to let the suspect contact his attorney until the suspect is in secure custody.

NOTES: REF: 24. Police Officer Thompson is unsure about what factors should be considered when deciding if non-deadly physical force is reasonable. Police Officer Thompson asks Field Operations Lieutenant Hernandez to clarify factors that should be considered in a possible situation. Field Operations Lieutenant Hernandez should inform Police Officer that when deciding whether non-deadly physical force is reasonable, one (1) of the factors that should be considered is the individual's:

Police Practice Test 3

Page 10 of 34

a. race b. age c. religion

d. sexual preference e. social status

ANSWER: B Using the process of elimination, B is the only reasonable answer.You'd probably take in consideration when deciding whether non-deadly physical force is reasonable, the person's age. For instance the amount of force used when dealing with an elderly person as opposed to a young adult would probably not be the same.. NOTES: REF:

25. Officer Harris responds to a sexual assault at 2334 S. 10st. Upon arrival, Officer Harris observes the victim with her clothing torn, sitting on the rear porch at that address. The victim says to the officer she was just rapped and the suspect ran south on 10th Street. What should Officer Harris do first? a. Obtain and broadcast a description. c. Administer first aid and call for ambulance if needed. b. Return to the squad and check the d. Check the scene of the crime for area for the suspect evidence. ANSWER: C First priority should always be the safety and well being of all people involved. NOTES: REF:

26. Officer Sanders is sent to investigate a sexual assault at the Gas Station at 35th and Vine. Upon arrival at the scene she is met by the victim who is uninjured and appears calm. The victim informs the officer that she knows the suspect's first name and gives the officer a detailed disruption. She tells the officer she met the suspect at a night club on 27th street. He followed her out of the night club and raped her. She drove to the gas station and called the police. She thinks the suspect returned to the night club. The next thing the officer should do is: a. Go to the night club to check for the suspect. b. Broadcast the description and request that a squad check the night club for the suspect. c. Check the area for evidence. d. Transport the victim to a medical facility.

ANSWER: B Since the victim is not injured, transporting the victim to a medical facility can wait. Checking for evidence can also wait. Police officers should rely on team work and not try to do everything themselves.That's why (a) should be eliminated. NOTES: REF: 27. A supervisors should be called to the following assignments, EXCEPT:

Police Practice Test 3

Page 11 of 34

a. armed robbery in progress b. burglary in progress

c. traffic accident involving a car and a bicycle where the bicyclist has head injuries d. arrest of a burglary suspect who resisted and was pepper sprayed.

ANSWER: B It's the only situation where a supervisor would not be called. Using the process of elimination, (A) can be eliminated because it involves an armed suspect. (C) can be eliminated because it includes life threatening injuries. (D) can be eliminated because it involves the use of force. NOTES: REF:

28. Officer Kent responds to the welfare of an elderly man living alone at 1983 S 10th. Upon arrival he finds the elderly man inside the residence deceased in a bedroom. The neighbors advise Officer Kent that the deceased Mr. Rose lived alone. Officer Kent should take the following steps to protect the prisoners property, EXCEPT: a. In the presence of a supervisor, d. Call for back up squads. search the body and the residence for valuables and identification. b. Count all money in front of the e. Deliver the property to the police supervisor. Remove all property property room and place the except clothing. property recovered on a property inventor report. c. Enter in Memo Book a list of all property removed. ANSWER: D All the above choices should be done EXCEPT (D), call for back up squads. Finding a dead body in itself does not present a situation where a back up squad would be required. If a homicide were indicated, then the need for more squads would be required for crime scene protection and investigation. Furthermore, the term back up signifies an emergency situation requiring a quick response. NOTES: REF:

29. Officer Johnson investigates a report of a man found dead of natural causes in his apartment at 1903 W. Brantly. No foul play is suspected. Upon arrival at the scene, Officer Johnson finds the deceased man and calls for a supervisor. What should Officer Johnson do next? a. Enter in Memo Book a list of all property removed. b. Count all money in front of the supervisor. c. Remove all property except clothing. d. In the presence of a supervisor, search the body and the residence for valuables and identification. e. Request another squad to search the apartment

Police Practice Test 3

Page 12 of 34

ANSWER: D The correct answer is D. Searching the deceased and the house next, with a supervisor present, would guard against false claims of lost or stolen valuables. NOTES: REF:

30. Officer Doyne is sent to investigate a missing person at 231 E. Milton. Upon arrival at the scene, she gets a detailed physical and clothing description from the caller. The caller informs Officer Doyne that the missing has no physical disabilities. The next thing officer Doyne should do is... a. Obtain a photograph if available. c. Determine if the person has a mental disability. b. See if the missing person has a d. File a missing person report wanted status ANSWER: C Although all the steps listed in the answer choices should be done, finding out if the person has a mental disability is an important factor and should be done next. This information may indicate that the missing person presents an immediate danger to her/ himself or others.

NOTES: REF: 31. Officer Stoker arrests a man for auto theft. He handcuffs the prisoner's hands securely behind his back. He notices a slight bulge in the suspects left sock. What should Officer Stoker do next? a. Search the suspect front pocket. c. Search the area within reach of the suspect's handcuffed hands. b. Search the suspect's socks. d. Search the suspect's cap and hair. ANSWER: C Search the area of the immediate reach of the suspect. This area presents the most danger. NOTES: REF:

32. Officer Slater and Gains arrest a suspect for assault. The suspect attempts to run. Both officers chase the suspect and tackle him. What should the officer do next?

a. Search the suspect for weapons. b. Advise the suspect of his rights.

c. Put handcuffs on the suspect. d. Put the suspect in the squad

ANSWER: C The goal of the officers at this time is to gain control of the suspect and prevent injuries to themselves and the suspect as well as preventing the suspect from escaping. The best way to do that, would be to secure the suspect in handcuffs. NOTES: REF:

Police Practice Test 3

Page 13 of 34

33. Officer Ford is inside a residence investigating a domestic dispute. An angry husband pulls out a knife and starts moving toward the officer in a threatening manner. What's the FIRST thing the officer should do?

a. call for back up squads b. draw his firearm

c. seek cover d. pepper spray the man

ANSWER: B A knife threat is considered a threat of deadly force. An officer has the right to defend himself against deadly force with deadly force. If you can defuse the matter without deadly force, then you should. Pepper spray is not always effective and it may be too late to find out if it would decentralize the assailant. Actually drawing your weapon and creating distance would be the appropriate response. NOTES: REF:

34. How Should An Off Duty Officer Respond To A Crime In Progress. An off duty officer who is having lunch in a restaurant observes two men enter the restaurant pulling out guns, pointing them at the cashier and demanding money. What should the off duty officer do? a. He should do nothing. He's off duty and is the same as a private citizen. b. Attempt to stop the robbery and arrest the robbers. c. Attempt to get a detailed description and relate the information to the police dispatcher as soon as the robbers leave. d. Shoot the robbers as soon as the opportunity presents itself.

ANSWER: C The correct answer is C. It would present too much of a risk to confront the suspects in the restaurant, especially being out numbered. Officers are in fact real heroes, but not fools. It would be foolhardy to risk the lives of the customers, employees and yourself for what may be just the loss of property. The robbers will most likely be apprehended, eventually. Attempting to apprehend the suspects when they exit the restaurant may be an option if you can gain a favorable position on the suspects without endangering citizens. If the suspects start shooting people in the restaurant then the game changes. If the off duty officer is armed he/she would be compelled to use deadly force to prevent serious injury or death to him/herself or others. NOTES: REF:

35. While directing traffic at a busy intersection you observe a man walking across the intersection grabbing his chest in obvious pain. What should you do, FIRST? a. Perform first aid. c. Help the person get out of the roadway to prevent him from

Police Practice Test 3

Page 14 of 34

b. Divert traffic and instruct the person to lie down in the street.

being hit by a car. d. Inform the person that you will help him after you’re finished directing traffic.

ANSWER: C Help the person to get across the street. It does no good to administer first aid and have you or the man get hit by a car. If the man falls unconscious then you have no choice, you'll have to divert traffic and administer first aid. You'll need the help of private citizens. Until other officers or medical assistance arrives, you can protect the area where the man has fallen with a vehicle, with the vehicle's flashers activated. NOTES: REF:

36. The LEAST appropriate situation for an officer to respond as an emergency vehicle, with redlights and siren activated is... a. Armed robbery in progress at a c. A gang fight. service station. b. A traffic accident involving seriousd. A hospital reporting that a victim injuries. of a stabbing was just brought in. ANSWER: D A hospital reporting a victim of a stabbing was just brought in, is the LEAST appropriate situation for an officer to respond as an emergency vehicle. Although it is a serious matter, it's unlikely the suspect will be at the hospital and medical attention is already being taken care of. All the other situations require immediate response. NOTES: REF:

37. A person making a complaint of a theft seems confused and the facts related seem a bit farfetched and somewhat unrealistic. The Officer taking the complaint should; a. Inform the citizen that his complain is unreal. b. Listen to the person's complaint,attempt to clarify the facts and inform the person you will investigate the matter. c. Arrest the person for obstructing an officer. d. Tell the person there is nothing you can do for him.

ANSWER: B Rule of thumb, treat every complaint as legitimate, UNLESS you have evidence it is not. Victims of crimes are generally upset and may not have some of the facts perfect, such as times and dates. But it doesn't mean the crime didn't occur. Also, NEVER tell a complainant, there is nothing you can do for them. NOTES: REF: 38. Officer Brown is sent to an armed robbery of a gas station at 35th and National. Upon

Police Practice Test 3

Page 15 of 34

arrival at the scene Officer Brown is informed by the complainant that she was just robbed by two white male suspects, armed with handguns who fled South on 35th Street, in a brown pickup truck, with a burned out tail lamp. What should Officer Brown do next? a. Obtain a full description of the c. Broadcast the short description armed robbers and the vehicle, given to him by the complainant, then broadcast it to all squads. immediately. b. Returned to his squad to drive south on 35th street to look for the suspects. ANSWER: C Always broadcast initial descriptions of the basic information , immediately. This gives responding squads and squads in the area the best chance of apprehending the suspects during flight. This is especially true if a vehicle is involved. NOTES: REF:

39. Officer Zens investigates an armed robbery where two white males, wearing leather jackets and blue jeans, armed with a knife and a hand gun, fled the scene in a blue Chevy. What information in the above description will best help responding squads locate the suspects?

a. type of weapon b. suspects race and sex

c. auto involved d. clothing description

ANSWER: C Autos involved in a crime should always be one of the first things given in the description because it is the easiest thing to spot and identify by responding squads. NOTES: REF:

40. Officer Mills and Butler are the first officers at a shooting scene. They interview a witness, who describes the suspect as a white or Hispanic male with a beard, a scar on his left cheek, wearing a red T-shirt ,blue jeans, and white tennis shoes. The suspect was armed with a silver semi auto handgun. What part of this description would be the most useful to other squads in identifying the suspect?

a. white or Hispanic male b. red T-shirt

c. scar on left cheek d. beard

ANSWER: C The scar on the left cheek is the most distinctive identification detail offered. Scars are permanent. NOTES: REF:

Police Practice Test 3

Page 16 of 34

OBSERVING AND RECALLING FACTS AND INFORMATION TEST TASK:

OBSERVING AND RECALLING FACTS AND INFORMATION TEST TASK: During a 3 minute time period you must try to memorize as much as you can about the people, objects and events shown in the above photograph.You will not be allowed to take notes during this time period. Look closely at the people in the scene: who they are (e.g., correction officer, inmate or instructor), what they are wearing, what they are doing, etc. Note the setting of the scene: what objects are present, how objects are positioned, what people are doing with objects, etc. After the 3 minutes the photograph should be taken away and you will have to answer questions about them.

Don’t scroll past this point until the 3 minutes are up. 41. What was the group’s focus of attention? a. a weightlifting bench c. a barbell on the ground b. an inmate holding his fist d. an inmate lifting weights

Police Practice Test 3

Page 17 of 34

ANSWER: B NOTES: REF: 42. The barbell in the photo; a. was being held by an inmate b. was on the barbell rack

c. was on the ground d. there was no barbell

ANSWER: C NOTES: REF: 43. The man being shown the fist: a. was bald b. was wearing a baseball cap

c. was wearing a knit hat d. had a hood on

ANSWER: C NOTES: REF: 44. How many of the men had sweatshirts with hoods? a. 1 c. 3 b. 2 d. 4

ANSWER: C NOTES: REF: 45. How many people were in the photo? a. 4 c. 6 b. 5 d. 7

ANSWER: D NOTES: REF: 46. Officer Johnson arrives at a traffic accident scene, at 7th and Mills, with no injuries involved. There are two vehicles involved with minor damage. He positions the squad behind the accident with emergency lights activated. What's the next thing Officer Johnson should do? a. Call for a tow truck. c. Put the reflective vest on and divert traffic around the accident. b. Have the drivers move the vehicles d. Interview the witnesses before out of traffic. they leave. ANSWER: B The correct answer is b. Getting the vehicles out of traffic to prevent more collisions should be the number one priority at this point, if there's no injuries. It would also open up the natural flow of traffic at that intersection NOTES:

Police Practice Test 3

Page 18 of 34

REF: 47. You’re on routine patrol and see two men fighting. One of the men punches the other man knocking him to the ground. When you approach the men regarding the altercation, they feel embarrassed by it and state that they're friends, just joking around. The victim who fell to the ground says he does not want to file charges. What do you do about this public disturbance that has occurred in front of you? a. Arrest the man who threw the punch for assault. b. Arrest both men for disorderly conduct. c. Don't arrest either of the parties involved. d. Issue both men citations for disorderly conduct and let them go.

ANSWER: C Generally speaking in assault cases with minor injuries, an element of the crime is consent. For instance, if someone gives another person consent to punch him in the shoulder, there is no crime. People often goof around, but it doesn't necessarily mean you must arrest when you see such an act take place in public. Common sense indicates no harm no foul. If there are no injuries and the victim does not want to press charges then there is really no crime. Although if people started to gather thinking it was a real fight it could constitute Disorderly conduct. In which case citations could be issued and arrest for disorderly conduct could be made. The deciding factor would be whether the incident created a disturbance or not. NOTES: REF:

48. When dealing with children a Police Officer should: a. Treat them the same way you treat c. Take an authoritative approach. adults. b. Explain to them how the law can d. Attempt to gain their confidence. punish them if they do wrong. ANSWER: D Whether it's protecting them or taking them into custody a police officer should ALWAYS attempt to gain the confidence of children. NOTES: REF:

49. Police Officers should always maintain the following attitude:

a. defensive b. constantly on the alert

c. hostile d. authoritative

ANSWER: B A Police officer should ALWAYS be alert to his or her surroundings. NOTES: REF:

Police Practice Test 3

Page 19 of 34

50. As a new Police Officer you should LEAST expect your immediate supervisor to; a. Help you avoid errors. c. Make all necessary and difficult decisions for you. d. Inform you of your progress.

b. Give you specific instructions.

ANSWER: C Don't expect a supervisor to make all your tough decisions for you. Police officers are in the decision making business. The first day you walk out of the police academy, you are expected to take on responsibilities immediately. NOTES: REF:

51. As the police officer on routine patrol, you find a man laying unconscious on the ground near a bus station. The man smells like alcohol. He does not immediately respond when you try to revive him. The next thing you should do is... a. Handled the situation yourself by c. Do nothing, as intoxicated people continuing to attempt reviving often pass out. him. b. Call for medical aid d. Place the man under arrest for public drunkenness. ANSWER: B You’re a police officer, not a doctor or a paramedic. Leave medical care to the professionals. Common-sense indicates that if the man does not revive, there might be something seriously wrong with him . NOTES: REF:

52. Officer what would you do, IF? You find a two-year old child, standing outside a grocery store, crying. The people standing outside the store don't know who the child belongs to. What would you do next? a. Broadcast on the police radio that c. Take the child into the store and you have located a lost child and try to find his parents. give a description. b. Take the child to the precinct and d. Ask the dispatcher if there has been a report of a missing child in try to locate the parents. the area. ANSWER: C Once again, Common-Sense plays a key role in your decision. Small children often wonder away from their parents. Most likely, the parents can be found in the store. NOTES: REF:

53. Officer what would you do, IF? You stop a motorist for a traffic violation. He's very rude and indicates he has friends in high places. He demands your name and badge number. What would you do?

Police Practice Test 3

Page 20 of 34

a. Inform him that the information he c. Tell him you'll give him your wants will be on the citation. badge number but not your name. b. Give him both your name and d. Completely ignore his request. badge number. ANSWER: B Police officers are required to act professionally. Always give your identification to anyone who asks. You have nothing to hide. People are not generally happy when they receive traffic tickets. They often vent. A good police officer will remain calm in this situation and respond professionally.

NOTES: REF: 54. Officer Henry observes a red Station Wagon drive through a red light. He attempts to stop the vehicle regarding the traffic violation. The operator of the vehicle takes off at a high rate of speed through busy rush-hour traffic. Officer Henry recognizes the driver and was able to observe the vehicle's license plate number. Officer Henry's best course of action would be to: a. pursue the vehicle because traffic c. not pursue because identification violators should not be allowed to has been made and arrest can be escape made later b. not pursue because the risks out d. both B and C weigh the violation ANSWER: D Due to traffic volume, the minor traffic violation involved and the suspect being known by the officer, the pursuit should be discontinued. NOTES: REF:

55. Officer Johnson is involved in a high speed vehicle pursuit. Regarding continuing or discontinuing the pursuit, Officer Johnson should consider the following conditions, EXCEPT: a. The nature of the violation b. The type of headlights on the fleeing vehicle c. The volume of the traffic d. The nature of the area, whether residential, commercial, school zone, open highway etc. e. The population density.

ANSWER: B All the OTHER answer choices should be considered in regards to continuing a pursuit. NOTES: REF:

Police Practice Test 3

Page 21 of 34

56. An officer initiating a pursuit, shall in all cases notify the dispatcher as soon as possible that a pursuit is underway and provide the following information EXCEPT: a. Police unit identification b. Location, speed and direction of travel c. Pursued vehicle's description, including license number, if known d. The last time the officer was involved in a vehicle pursuit

ANSWER: D D is NOT pertinent in aiding the dispatcher or assisting squads in the apprehension of the suspect. All of the other answers describe information that is necessary for responding squads and the dispatcher, to give assistance to the pursuing squad. NOTES: REF:

57. Officer Smith observes a burglary suspect jump from a rear window of a warehouse. The suspect runs into an alley. Officer Smith starts chasing the suspect and radios for assistance. The suspect runs around the corner of an abandoned building. Officer Smith looses sight of the suspect. What should Officer Smith do next? a. run around the corner after the suspect b. slow down and peek around the corner from a safe distance c. fire a warning shot d. stop chasing the suspect

ANSWER: B If a suspect runs around a corner, slow down, get as far back from the corner as possible and peek around to ensure the suspect is not waiting to attack you. Don't get ambushed. NOTES: REF:

58. When involved in hand to hand combat, as a police officer you should consider the safety of the following people, in what order? 1. Criminals 2. Fellow officers 3. Civilians 4. You

a. 1,2,3,4 b. 4,2,3,1

c. 2,4,3,1 d. 3,2,4,1

ANSWER: B In order to survive combat, your safety and survival should be the number one priority. If you don't have a strong need for survival you won't win and you will not be able to help fellow officers or civilians if you loose the fight. NOTES:

Police Practice Test 3

Page 22 of 34

REF: 59. Officer Riley interviews four witnesses to an armed robbery of a gas station. Each witness observed the suspect leaving the scene of the crime. They described the suspect as follows: Witness 1 He was a white male, 20 to 30 years of age with brown curly hair. About 6' 1 ft., 190 lbs. He wore blue jeans and a black leather jacket. Witness 2 He was white or Hispanic, in his late 20's, 5'11", 195 lbs. with curly hair. Who was wearing dark pants and a dark colored jacket. Witness 3 He was a thin white male, in his late 20's about 6 ft., 185 lbs. with a tattoo of a cross on his right bicep. He had brown curly hair. Witness 4 He was a light skinned Hispanic male, with dark curly hair, 5'10" to 6'1" ft about 200 pounds. Wearing a black or brown jacket and blue jeans. Officer Riley should recognize the problem with which witnesses description?

a. 1 b. 2

c. 3 d. 4

ANSWER: C All the witnesses’ descriptions are similar EXCEPT witness number 3, who stated he observed the suspect with a tattoo on his right bicep. Witnesses 1,2 & 4 all agreed that the suspect was wearing a jacket , which made witness number 3's observation of a tattoo on the suspects right bicep, not possible. NOTES: REF:

60. Entrapment occurs when: a. Officers arrange the opportunity for the commission of a crime c. Officers encourage the commission of the crime by a person who had no intent to commit a crime until persuaded to do so by the officers d. None of the above

b. Officers participate in criminal activity to gain criminal's confidence

ANSWER: C NOTES: REF:

Police Practice Test 3

Page 23 of 34

61. A suspect wanted for a traffic offense, resists arrest, lunging at the officer with a knife. Regarding this statement which of the following is true: a. The officers cannot use deadly force because the suspect is not wanted for a serious crime b. The officer cannot use deadly force because a knife is not considered a deadly weapon c. The officer can use to deadly force because the suspect is resisting arrest d. The officer can use deadly force because even though the suspect is wanted for a minor crime, the suspect is resisting using deadly force against the officer

ANSWER: D Even though it is a minor offense, deadly force is justified in self defense, to protect the officer's life. NOTES: REF:

62. A police officer arrives at the scene of an accident involving injuries. In what order should the officer do the following: A. Interview witnesses B. Determine if anyone needs medical attention C. Move the vehicles off the roadway D. Interview the drivers

a. B , D, C, A b. B, D, A, C

c. B, C, A, D d. D, B, C, A

ANSWER: B Using the process of elimination: You'll immediately eliminate d because now you know medical attention is the number one priority. Common-sense should dictate what comes next. Although it's important to get witness statements, it's more important at this time to talk to the people involved. Also you need to look at the evidence. As long as you have the traffic diverted around the accident, moving vehicles off the roadway can wait. You might want to take photos and measurements of the accident scene and keep the vehicles as is, depending on your Department's accident procedures and the seriousness of the accident. So b is the correct answer.

NOTES: REF:

63. Which of the following crimes has a better chance for suspect identification?

a. auto theft

c. robbery

Police Practice Test 3

Page 24 of 34

b. burglary

d. vandalism

ANSWER: C Robbery is a crime where the suspect is, in most cases, observed by the victim, making identification of the suspect easier. The others crimes listed, auto theft, burglary, and vandalism are usually more difficult to solve because they're usually committed without victim identification. This is a common sense question. NOTES: REF:

64. Officer Jackson comes across a man who is unconscious, in an alley. Further investigation reveals that the subject has an empty syringe in his hand. Officer Jackson attempts to wake the man but is unable to do so. What should Officer Jackson do next? a. Ignore the situation and let the manc. Take the man into custody and wake-up on his own accord. transport him to nearest the detoxification treatment center. b. Arrest the man for possession of d. Call for an ambulance illegal drug paraphernalia. immediately to transport the subject to medical facility. ANSWER: D Selection D is the only reasonable choice considering the fact that the man could not be awakened, and there is the presence of the empty syringe, giving the indication of drug use, including possible drug overdose. Officer Jackson should assume that this is a life-threatening situation. Also, taking into consideration the most important police priority is protection of life and limb, calling for ambulance is the only option. Also, checking for vital signs and performing first aid should be factored in. NOTES: REF:

65. As an officer on routine patrol, you observe what appears to be a drug deal going down. You observe two individuals in a shopping mall exchange drugs for money. Both suspects observe you and start to run away in opposite directions. What should you do? a. Give a description of the suspects c. Write down a complete on the police radio and chase the description of the suspects, and suspect who has the money. attempt to seek a warrant for their arrest b. Fire a warning shot in order the d. Give a description on the police radio of the suspects involved and suspects to halt. chase the suspect with drugs. ANSWER: D It's easier to build a case against someone who has possession of illegal drugs than it is to build a case against someone in possession of money, alone. It's not illegal to be in possession of money. The officers best chance of apprehending

Police Practice Test 3

Page 25 of 34

NOTES: REF:

the suspects would be to give immediate chase and give the dispatcher and other officers a description of the suspect(s) being chased. This would allow other officers in the area to help apprehend the suspects. Firing a warning shot would present too much of a risk in regards to the general public. Firing warning shots in most jurisdictions is not an acceptable practice.

66. You and your partner arrest a suspect for domestic abuse. You feel your partner failed in several regards to adequately assist you in this arrest. You felt his lack of action placed you in unnecessary danger. How should you handle the situation? a. Report your partners lack of action c. File a written request for a to a supervisor. different partner. b. Confront the officer directly and d. None of the above discuss the problem. ANSWER: B Confronting your partner directly is the best course of action at this point. If the question indicated that this was a continued problem then A would have been the best response. In most cases, by confronting the person directly you can resolve these types of problems quickly. If the problem continues, then you take the next step which is to report it to a supervisor. NOTES: REF:

67. Officer Roberts observers two men fighting. One of the men stabs the other in the chest with a knife. The victim falls to the ground and he is bleeding profusely. Under the circumstances, what should Officer Roberts do? a. Pursue the suspect and attempt to c. Attempt to slow bleeding and call arrest him. for an ambulance . b. Radio for an ambulance and d. None of the above attempt to locate witnesses. ANSWER: C It's obviously important to call for an ambulance ASAP in this situation. Although, the FIRST priority is to stop the bleeding otherwise the victim could bleed to death before an ambulance arrives. NOTES: REF:

68. Officers must be accurate yet brief when broadcasting on the police radio. What is the best reason for the above statement? a. It makes officers sound more professional c. Tying up the airwaves with long messages on the police radio may prevent emergency transmissions from getting through. b. It saves the department money on d. Shorter messages are easier to radio air time expense understand.

Police Practice Test 3

Page 26 of 34

ANSWER: C It's difficult for Officers to get on the police radio with important and emergency transmissions at times because of the busy Airways. That's why offices are told to keep their messages accurate and brief . NOTES: REF:

69. Officer Jones sees a motorist commit a traffic violation at a construction site where street signs have recently been changed. What action should Jones take? a. Write down license plate and if he c. Stop the offender and issue a sees the motorist commit this warning. violation again issue him/her a citation. b. Ignore the incident because of the d. Stop the offender and issue a recent change. citation. ANSWER: C Here again, common-sense plays a factor. In this situation issuing a warning would be the best course of action, based on the fact that a recent change in traffic signs has occurred. This mistake can happen to anyone. It would be an officer's job to bring it to the motorist's attention by stopping him and giving him a warning. NOTES: REF:

70. Upon arrival at the scene of a demonstration, Officer Todd observes two officers fighting with demonstrators. Officer Todd pulls out his baton and goes to help the other officers. A demonstrator, not involved in the fight swings a broken bottle at Officer Todd. What should Officer Todd do next? a. Grab the bottle out of the man's c. Call for more back up hand b. Administer several blows with the d. Shoot the man baton in the attempt to disarm him ANSWER: B The best option at this point is try to disarm the man with baton strikes. Although calling for back up may seem like a good answer and probably should be considered, the man swinging the bottle needs to be dealt with first. Deadly force should always be thought of as a last resort. If the officer thought his life was being threatened or he was in danger of great bodily harm, then deadly force could be used. NOTES: REF:

71. Officer what would you do if? You and your partner respond to a burglary in progress at an office building. Upon

Police Practice Test 3

Page 27 of 34

arrival you observe a broken window and you see someone moving around inside the building. You call for additional squads to help search the building. Until backup arrives what should you and your partner do? a. Position yourself at one corner of c. You and your partner enter the the building, while your partner building and conduct a search for positions herself at the diagonally the suspect(s). opposite corner of the building. b. You position yourself at the front d. One officer maintains surveillance entrance, while your partner on the perimeter of the building, positions herself at the rear exit. while the other officer enters the building to search for the suspects. ANSWER: A The best vantage point for surveillance of the building would be opposite corners. This would give you and your partner the best chance to see the entire building. If you position yourself at the front entrance, while your partner positions herself at the rear exit, you’ll be limiting yourselves as far as viewing avenues of escape. Attempting to search the building before additional officers arrive would increase the chance of escape. NOTES: REF:

72. Officers conducting missing persons investigation should include the following information in their report, EXCEPT: a. if the person has a medical emergency b. detailed physical and clothing description c. physical or mental disability d. religion e. wanted status

ANSWER: D (religion). All the other answer choices contain necessary information required in missing persons reports. NOTES: REF:

73. An officer conducting a search of a prisoner she just arrested, should consider the following, EXCEPT: a. Checking legs and ankles b. Searching the prisoner from the prisoner's side. c. Searching the prisoner from the rear. d. Checking neck area and both arms

ANSWER: B The best vantage point as far as officer safety is searching from the rear of the suspect. Not from the side or front.

Police Practice Test 3

Page 28 of 34

NOTES: REF: 74. When should a person be advised of their Constitutional Rights? a. Before they're arrested. b. Right after they're taken into custody. c. After they're taken into custody and before they're questioned about a crime. d. After they're taken into custody and before they are taken to court.

ANSWER: C After they're taken into custody and before they're questioned about a crime. NOTES: REF:

75. Officer Ryan arrives at the scene of a traffic accident where there are no injuries involved. Both vehicles have significant damage and cannot be driven. Regarding the vehicles involved, what should Officer Ryan do FIRST? c. Leave the vehicles in the roadway until the investigation is completed b. Have the drivers move the vehicles d. Position the squad behind the out of traffic. autos involved in the accident, with emergency lights activated. a. Tow the vehicles. ANSWER: D Position the squad behind the autos involved in the accident, with emergency lights activated, to protect the scene from on coming traffic. NOTES: REF:

76. A police officer, chasing a suspect on foot should do the following, EXCEPT:

a. Radio for assistance immediately. c. Pace the suspect. b. Tackle the suspect when he gets d. Maintain visual contact. close enough.
ANSWER: B An officer chasing a suspect should attempt to do all the answer choices EXCEPT (b) tackle the suspect. Tackling the suspect and ground fighting leaves you open for a gun grab. Always try to maintain strategic control and approach. NOTES: REF: RULE INTERPRETATION: This question is based on rule interpretation.

Police Practice Test 3

Page 29 of 34

RULE: Patrol vehicles should be checked at the start of each shift. Do not assume that the vehicle is in satisfactory condition. Check all of the lighting equipment, all emergency equipment, siren, engine, oil, transmission fluid, battery, radiator and gasoline levels, tire pressure and condition (including spare), lug wrench, jack, windshield wipers and windshield washer fluid level. Check the body of the vehicle for damaged or missing parts and report any problems, damage or discrepancies to your supervisor. At the end of your shift, leave the vehicle in optimum condition for emergency use by the next officer. SITUATION: Officer Burton is about to begin her patrol shift when she discovers that her police vehicle has a large dent in the left rear bumper. She knows that the vehicle did not have this dent yesterday, when she last drove it.

77. QUESTION: According to the Rule above, Officer Burton should most properly... a. request that she be assigned a different vehicle c. find out what other officers have used the vehicle since her last shift b. begin her shift and be alert to any d. inform her supervisor about the operating problems dented bumper ANSWER: D SOLUTION: The situation states that Officer Burton has discovered a dent in the bumper of her patrol vehicle that did not exist when she last used it. The question asks what she should do about it. To answer the question, evaluate all of the choices. Choice A states that the officer should request a different vehicle. Nothing in the rule states that the officer should do this. Choice A is incorrect. Choice B states that the officer should begin her shift and be alert to any operating problems. The rule states that the officer should report any problems with the vehicle to her supervisor. Choice B is incorrect. Choice C states that the officer should find out what other officers have used the vehicle since her last shift. Nothing in the rule states that the officer should do this. Choice C is incorrect.

NOTES: REF:

Choice D states that the officer should inform her supervisor about the damaged bumper. This is in conformance to the rule stating that the officer should report any damaged or missing parts to her supervisor. Choice D is therefore the correct answer.

PREPARING WRITTEN MATERIAL IN A POLICE SETTING: This question tests your ability to prepare the types of reports that police officers write. You are presented with a page of notes followed by several questions. Each question consists of four restatements of the information given in the notes. From each set of four, you must choose the version that presents the information most clearly and

Police Practice Test 3

Page 30 of 34

accurately. TEST TASK: You must determine which one of the choices presents all the information from a particular portion of the notes and whether the phrasing and the punctuation of the sentence(s) results in a clear and accurate presentation of the information. QUESTION: The following is a portion of the notes you took about an accident you were dispatched to. NOTES: Responded to a call from 26 Arbor Ave. Residence of Tessa and John Wynter. Pulled in driveway. Saw woman on Wynters’ porch. Identified herself as Mrs. Orvis, a neighbor.

78. Which one of the choices that follow expresses the facts presented in the notes? a. I responded to a call from 26 c. When I responded to a call from 26 Arbor Avenue, the residence of Arbor Avenue, the residence of Tessa and John Wynter. When I Tessa and John Wynter, I saw pulled into the driveway, I saw a pulling into their driveway a woman on their porch. She woman on their porch who identified herself as Mrs. Orvis, a identified herself as Mrs. Orvis, a neighbor. neighbor. b. Responding to a call from 26 d. Responding to a call from 26 Arbor Avenue, the residence of Arbor Avenue, I saw a woman on Tessa and John Wynter, and the porch of Tessa and John pulling into the driveway, I saw a Wynter’s residence. She identified neighbor on their porch, who herself as Mrs. Orvis, a neighbor. identified herself as Mrs. Orvis. ANSWER: A This is the only choice in which all the information is presented, and it is presented in the correct sequence. It says that the officer responded to a call from the Wynter residence, that he pulled into the driveway, saw a woman on the porch who told him her name and that she is a neighbor. Choice B: "I saw a neighbor on their porch" suggests that the officer knew that it was a neighbor on the porch before Mrs. Orvis told him who she was. In the notes and in choice A, the woman tells him both her name and that she is a neighbor. This choice is incorrect. Choice C: "I saw pulling into the driveway a woman on their porch" does not make sense, the woman could not be doing both. For C to be correctly written, there should be a period after Wynter, and the next sentence should begin: "Pulling into the driveway, I saw…" This choice is incorrect. Choice D: This choice does not say that 26 Arbor Avenue is the Wynters’ residence. It could be the address that the call came from. Also, another piece of information is missing; the officer does not say that he pulled into the driveway. In police writing every detail is important. This choice is incorrect.

NOTES:

Police Practice Test 3

Page 31 of 34

REF:
READING, UNDERSTANDING AND INTERPRETING WRITTEN INFORMATION: This question tests your ability to read, understand, and interpret the kinds of written information that police officers are required to read during their formal training TEST TASK: You are provided with brief reading selections and asked questions relating to the selections. All the information required to answer the questions is provided in the selections; you are not required to have any specific knowledge of the content areas covered in the selections. QUESTION: "The increasing demands upon our highways from a growing population, and the development of forms of transportation not anticipated when the highways were first built have brought about congestion, confusion, and conflict, until the yearly toll of traffic accidents is now at an appalling level. If the death and disaster that traffic accidents bring throughout the year were concentrated into one calamity, we would shudder at the tremendous catastrophe. The loss is no less catastrophic because it is spread out over time and space."

79. Which one of the following statements concerning the yearly toll of traffic accidents is supported by the above selection? c. It does not shock us as much as it should because the accidents do not all occur together. b. It has resulted in increased d. It has resulted mainly from the congestion, confusion, and conflict new forms of transportation. on our highways. ANSWER: C SOLUTION: To answer this question, evaluate all the choices. Choice A: Nowhere in the passage does it say that there has been any demand for safer means of transportation. Someone who picks this choice may believe that there could be or should be a demand for safer transportation, but there is nothing in the passage to base it on. This choice is incorrect. Choice B: The passage states that it is the congestion, confusion, and conflict which results in the high toll of traffic accidents and not the other way around. A person who picks this choice could either be confused as to which is the cause and which is the effect or not have read the choice carefully. This choice is incorrect. Choice C: This choice is based on the last two sentences in the passage. The writer says "If …, we would shudder. " (A shudder is a response to shock.) The implication is that we don’t, and we don’t because accidents do not all occur at the same time and place. The writer then points out that we should think of the yearly toll as being catastrophic (shocking) even though the accidents are a. It is increasing the demands for safer means of transportation.

Police Practice Test 3

Page 32 of 34

spread out over time and space. This choice is the only correct choice. Choice D: There are two reasons given in the paragraph for the high accident rate. One is the development of new forms of transportation; the other is the increased highway use from a growing population. Neither one is described as the main reason. It is clearly incorrect to say that the new forms of transportation are the main reason. This choice is incorrect.

NOTES: REF:

MEMORY FOR FACTS AND INFORMATION: This question tests how well you can remember facts and information presented in written form after you have been given a period to read and study the information. TEST TASK: You are first given a Memory Test Booklet containing a story. It will be considerably longer than the one presented here. You will have a limited period of time in which to study the details contained in the story. You will NOT be allowed to take notes while studying the story. At the end of the study period, the monitor will collect the test booklets containing the story and then hand out the test booklets containing the test questions. The first group of questions in this test booklet will ask you to recall the facts and information presented in the Memory Story. MEMORY STORY: Officer Gary Hanson of the Burke Police Department was questioning Mathew Meyers, the owner of Meyers Sporting Goods, located at 321 Payne Avenue about a burglary that occurred the previous evening. Meyers said that when he arrived at the store at 8:50 A.M., he noticed that the rear door had been broken into. Meyers said that, after he had checked his inventory, he was missing 20 rifles, 16 pellet guns, 12 shotguns and 8 pistols.

80. QUESTION: How many shotguns did Meyers tell the Officer were missing from his store? a. 8 c. 16 b. 12 d. 20 ANSWER: B SOLUTION: The question asks how many shotguns did Meyers tell the Officer were missing from his store. The last sentence in the Memory Story states, "… Meyers said that … he was missing 20 rifles, 16 pellet guns, 12 shotguns and 8 pistols." Choice A (8) is the number of missing pistols. Choice B (12) is the correct answer. Choice C (16) is the number of missing pellet guns. Choice D (20) is the number of missing rifles. NOTES: REF:

81. You are an off-duty police officer, invited to a party at a nearby home (private residence). While at the party, you observe a couple of people using illegal drugs. The best course of action would be: a. Do nothing.You are off-duty. Stay c. Call the dispatcher and have

Police Practice Test 3

Page 33 of 34

and enjoy yourself. b. Approach the people concerned and arrest them. Drugs are illegal.

whomever is on-duty to come and arrest the people concerned. d. Leave the party immediately.

ANSWER: C Using the process of elimination. Eliminate a & d. I'm sure, your police agency would not want you to look the other way if you see a crime committed. Next, common sense may help you choose the correct answer. I'd select c. Trying to make an arrest in what may be a hostile situation by yourself may be asking for trouble. Calling it in and having a squad meet you, would be the best option. When answering the questions always consider logic.

NOTES: REF:

82. The 4th Amendment to the Constitution provides a high degree of protection for individuals from searches and seizures made by the police. Searches and seizures carried out by officers without a warrant must be clearly justified by officers as falling into one of the well-defined exceptions to the Fourth Amendment requirement. To avoid suppression of evidence and possible personal liability an officer should: a. always obtain a warrant before instituting a search and a seizure b. only attempt to obtain a warrant whenever no other options are available before instituting a search and a seizure c. always attempt to obtain a warrant whenever reasonably possible before instituting a search and a seizure d. obtain a warrant 7 days before instituting a search and a seizure

ANSWER: C If you know nothing about the 4th Amendment common sense and the process of elimination can help you answer this question. You can eliminate answer a, (always obtain a warrant). Sometimes searches must be made immediately, in situations where evidence can be destroyed or in the case where officers search suspects for weapons. Eliminate answer b, because although a warrant cannot always be obtained it should be obtained, whenever possible. Eliminate answer d, because one of the factors in a search warrant is, the officer must believe the property in question will be at the location of the search when the search is made. There is a time element involved. If the warrant can only issued 7 days prior to the search, there is no guarantee the property will still be there when the search is made. NOTES: REF:

83. Use the map below to answer the sample problems.

Police Practice Test 3

Page 34 of 34

You are at the store on Lake and heading to the cleaners on Forest. What is the shortest way to get there? a. North on Lake to Forest, east on c. South on Lake to Indian, east on Forest. Indian to Meadow, north on Meadow to Forest, west on b. South on Lake to Indian, east on d. South on Lake to Indian, east on Indian to Black, north on Black to Indian to Pacific, north on Pacific Forest, east on Forest. to Forest, west on Forest. ANSWER: D The correct answer is d, since it is the shortest legal route. NOTES: REF:

Police Practice Test 4

Page 1 of 26

Police Practice Test 4 Answer Key
Multiple Choice Identify the letter of the choice that best completes the statement or answers the question.

1. Which of the following is NOT entrapment? a. An officer observes a car traveling d. A Patrolman in an unmarked car behind him. The officer speeds up on a deserted highway challenges a little and the car follows suit. a persons to race and arrests the The officer continues to increase person prior to actually drag his speed, clocks the following car racing. and arrests the driver for speeding. b. An undercover officer begs a drug e. All of the above are examples of entrapment. user to sell the officer some drugs claiming that he is suffering from withdrawal symptoms. c. An undercover officer acts as a drunken derelict in a high crime area and arrests two men attempting to rob him.

ANSWER: C NOTES: REF: 2. Which one, of the following search and seizure situations is exempt from requiring a warrant? a. searching hidden places in a d. a b & c dwelling for contraband b. automobile search e. a & b c. searching the person when no charges are likely be filed

ANSWER: B NOTES: REF: 3. When a person politely refuses to answer an officer's questions during a stop, what should the officer do? a. allow the person to go on his way. d. issue a citation for loitering. b. demand that the person comply e. arrest the person for obstructing with the request. an officer . c. search the person for

Police Practice Test 4

Page 2 of 26

identification.
ANSWER: A A stop is not an arrest. To detain a person against their will the officer should be able to articulate his/her belief the subject has committed a crime. In most jurisdictions refusal to give ones name in itself does not constitute a crime. NOTES: REF:

4. Officer Jones hears a shot and observes a subject running into a home with a gun. Which of the following should Officer Jones do? a. Obtain a warrant to enter the home c. Enter without a warrant because there is an inherent threat of injury or death to others b. Attempt to get consent to enter d. Ignore the situation

ANSWER: C NOTES: REF: 5. When are Miranda warnings required? a. when the investigation has moved c. When a person enters a police from the investigatory to custodial station and confesses to a crime stage b. where an officers stops a speeding auto

ANSWER: A NOTES: REF: 6. The critical element that constitutes entrapment is whether: a. the defendant was predisposed to d. participation on the part of the commit the crime in question officer in order to gain evidence occurred b. the opportunity to commit a crime e. the degree of cooperation by the was arranged by the police officer destroys the essential element of the crime c. deception was used to gain evidence. ANSWER: A The key word is predisposed. Entrapment occurs when the officer plants the idea of committing the crime into the person who had no inkling to commit the crime before hand. NOTES: REF:

7. Entrapment occurs when: a. Officers arrange the opportunity for the commission of a crime

c. Officers encourage the commission of the crime by a

Police Practice Test 4

Page 3 of 26

b. Officers participate in criminal activity to gain criminal's confidence

person who had no intent to commit a crime until persuaded to do so by the officers d. None all the above

ANSWER: C NOTES: REF: 8. Under what circumstances may an officer, on patrol, stop a person? a. The officer doesn't like the way d. The officer has mere suspicion the person looks b. The officer has reasonable e. None of the above suspicion c. The person says hello to the officer and walks away

ANSWER: B NOTES: REF: 9. Which of the following would constitute exigent circumstances? a. immediate threat of escape c. immediate threat of death of great bodily harm b. immediate threat of destruction of d. all the above evidence

ANSWER: D NOTES: REF: 10. An officer may arrest a subject on the belief that a _______ warrant has been issued in another state. a. Misdemeanor c. Misdemeanor or felony b. Felony d. None of the above

ANSWER: B NOTES: REF: 11. That quantum of evidence which would lead a reasonable police officer to believe the defendant has committed a crime, is called: a. an arrest c. probable cause b. a stop and frisk d. a warrant ANSWER: C Quantum of evidence is the key phrase. NOTES:

Police Practice Test 4

Page 4 of 26

REF:
READING COMPREHENSION This section of the test measures how well you understand what you read. It consists of a number of short paragraphs followed by questions. The information needed to answer the questions is contained in the paragraphs. In some instances, you may have to draw a conclusion based on the information given. Sample Passage There are several types of investigation or inquiries, which police typically undertake as part of their job. Investigations of incidents, which are violations of laws and ordinances, include crime and traffic accident investigations. Personnel investigations are conducted to evaluate the suitability of individuals for positions of public trust.

12. According to the passage, conducting investigations is a routine part of a police officer's job. a. True b. False ANSWER: A The correct answer for the first question is A, True, because the passage states that, conducting investigations, is part of a police officer's job. NOTES: REF:

13. According to the passage, police investigations of incidents which are violations of laws and ordinances, include: a. traffic accidents c. personnel b. crime d. A and B only ANSWER: D The correct answer for the second question is D, because the passage does not include personnel investigations as a violation of a law or ordinance. NOTES: REF:

14. Choose the answer which most closely describes the word POWERFUL

a. happy b. energetic ANSWER: C NOTES: REF: 15. Add: 327+512 a. 475 b. 510 ANSWER: D

c. strong d. blue

c. 545 d. 839

Police Practice Test 4

Page 5 of 26

NOTES: REF:

In this example, the correct answer is D, because the sum of 327 and 512 is 839.

16. A police officer has to search a warehouse that is 60 feet long and 40 feet wide. What is the perimeter of the warehouse? a. 100 feet c. 200 feet b. 150 feet d. 240 feet ANSWER: C In this sample problem the correct answer is C (200 feet) because the perimeter of the warehouse is calculated by adding 60 plus 40 and then multiplying by 2. NOTES: REF:

17. 3 X 9 + 15 = a. 27 b. 42

c. 72 d. 92

ANSWER: B The correct answer is B (42) because 3 multiplied by 9 is 27 and adding 15 equals 42. NOTES: REF:

18. Which of the following is not a mammal: a. bat c. whale b. horse d. shark

ANSWER: D NOTES: REF: 19. The permanent set of teeth in a human adult including wisdom teeth numbers:

a. 28 b. 30 ANSWER: C NOTES: REF:

c. 32 d. 34

20. Which one of these birds does not have feathers:

a. penguin b. ostrich c. eagle ANSWER: A

d. a robin e. a they all have feathers

Police Practice Test 4

Page 6 of 26

NOTES: REF: 21. When formulating goals into specific activities, one should: a. Give priority to activities done easiest. c. Make sure activities have a date, time, and place to ensure their fulfillment. b. Overrule any previously scheduledd. Wait two weeks or until no longer appointments. busy to start activities.

ANSWER: C NOTES: REF: 22. Teachers: a. are concerned with the welfare of c. are unfair every student b. can help students by encouraging d. are all critical thinkers critical thinking

ANSWER: B NOTES: REF: 23. Movies today: a. invariably promote death and violence and anarchy. b. are the cause of the increasing crime rate in America. c. seem to be much more graphic in nature. d. all of the above.

ANSWER: C NOTES: REF: 24. How many sides does the Pentagon have in Washington DC:

a. eight b. nine ANSWER: C NOTES: REF:

c. five d. seven

25. The skiing industry is more profitable in the following state:

a. Utah b. Florida ANSWER: A

c. Puerto Rico d. Louisiana

Police Practice Test 4

Page 7 of 26

NOTES: REF: 26. You can have hallucinations if you eat: a. raisin bran c. LDS b. LSD d. beans

ANSWER: B NOTES: REF: 27. The University of Michigan is in the ________ Conference:

a. Western Athletics b. Big Twelve's ANSWER: C NOTES: REF: 28. A species is a:

c. Big Ten d. General

a. bunch of similar looking c. group of organisms that live in organisms the same place b. genetically distinctive natural d. group of individuals that cannot population that shares a common breed with each other gene pool

ANSWER: B NOTES: REF: 29. Altruism, as defined by social scientists, is: a. selfless concern for others and c. connected series of rules that unselfish behaviors that foster the govern a complex pattern welfare of others b. social norm of reciprocity d. running away from the situation

ANSWER: A NOTES: REF: 30. Patrimony is: a. a father's relationship with his children b. a neutral nation c. property inherited from one's father or ancestors d. the cutting of a precious stone

ANSWER: C NOTES:

Police Practice Test 4

Page 8 of 26

REF: 31. Several hours before an athletic event, athletes should eat a lot of:

a. candy bars b. high protein foods ANSWER: D NOTES: REF:

c. fatty foods d. carbohydrates

32. In the following series, what comes next? 1, 2, 4, 8... a. 6 c. 32 b. 16 d. 15

ANSWER: B NOTES: REF: 33. In the following series, what comes next? 1, 3, 5, 7... a. 9 c. 11 b. 10 d. 13

ANSWER: A NOTES: REF: 34. In the following series, what comes next? 1, A, 9, I, 17... a. Q c. E b. Z d. U

ANSWER: A NOTES: REF: 35. The bar graph below lets us compare the sales of each fruit over a 2 day period, not just the sales of one fruit compared to another. We can see that the sales of star fruit and apples stayed nearly the same. The sales of oranges increased from day 1 to day 2 by ___ kilograms.

Police Practice Test 4

Page 9 of 26

a. 4 b. 14 ANSWER: C NOTES: REF:

c. 10 d. 21

36. The bar chart below shows the weight in kilograms of some fruit sold one day by a local market. We can see that ___ kg of apples were sold, 40 kg of oranges were sold, and 8 kg of star fruit were sold.

a. 32 b. 52 ANSWER: B NOTES: REF:

c. 67 d. 59

37. In the following series, which letter comes next? A, C, E, G, ..

Police Practice Test 4

Page 10 of 26

a. I b. K ANSWER: A NOTES: REF: 38. Which does not belong?

c. M d. H

a. grape b. apple ANSWER: C NOTES: REF:

c. corn d. orange

39. Fill in the blank in this series: 000, 001, 011,___.

a. 111 b. 101 ANSWER: A NOTES: REF: 40. Which is faster?

c. 010 d. 110

a. light b. sound ANSWER: A NOTES: REF:

c. fly d. eagle

41. If every day is a Sunday, and there is no work on Sundays:

a. Everyone goes to church b. No one works ANSWER: B NOTES: REF: 42. What color is the sky at night?

c. Everyone works everyday d. Tomorrow is Saturday

a. blue b. gray ANSWER: D NOTES: REF:

c. yellow d. black

Police Practice Test 4

Page 11 of 26

43. If a car takes 10 minutes to travel 60 km, how long would it take to travel 1800 km. at this same rate? a. 300 hours c. 50 minutes b. 4 hours d. 300 minutes ANSWER: D time = distance/rate 60km per 10 min = 6km per 1 min t = 1800km/6km 1800 divided by 6 (= 300 minutes)

NOTES: REF:

44. If all males are green, and all females are red then:

a. All boys are pink b. All girls are orange ANSWER: C NOTES: REF:

c. All boys are green d. All girls are green

45. If it takes 20 years for a tree to grown 1 meter. How long would it take for the tree to become 5 meters in height?

a. 4 years b. 100 meters ANSWER: C NOTES: REF:

c. 100 years d. 20 years

46. Arrange the following words to form a sentence and indicate whether this sentence is true or false. ( men, mortal, all, are.) a. true b. false ANSWER: A All men are mortal. NOTES: REF:

47. If it takes 8.3 minutes for the light of the sun to reach the earth, and the speed of light is 300,000km/s per minute. How far is it to the sun?

a. 149400000 km b. 24900000 km

c. 5 cubic cm d. 928328.56 MI

ANSWER: B 8.3 min x 300,000 km = 24900000 km

Police Practice Test 4

Page 12 of 26

NOTES: REF: 48. Which of the following is closest in meaning to: dipsomaniac a. sot c. hydrophobic b. roger d. repose

ANSWER: A NOTES: REF: 49. If one were to be pusillanimous one would be: a. dead c. a coward b. in love d. unconscious

ANSWER: C NOTES: REF: 50. Eros is:

a. darkness b. love ANSWER: B NOTES: REF:

c. the word for grape in Pakistan d. a natural cleanser

51. If you rearrange the letters "aber", what would you have the name of?

a. Automobile b. Town c. State
ANSWER: B (bear) Animal NOTES: REF:

d. Animal e. Vegetable

52. Which activity does not belong in this group?

a. jump b. run c. jog ANSWER: D NOTES: REF:

d. read e. ski

53. Which number doesn't belong in the following numerical sequence? 2 4 6 8 9 10 12 14

Police Practice Test 4

Page 13 of 26

a. two b. eight c. nine ANSWER: C NOTES: REF:

d. twelve e. fourteen

54. If you rearrange the letters "faegrfi", what would you have the name of?

a. Town b. State c. Animal
ANSWER: C Giraffe NOTES: REF:

d. Vegetable e. Mineral

55. Which word makes the best analogy? A IS TO Z as START IS TO:

a. go b. finish ANSWER: B NOTES: REF:

c. begin d. wait

56. Tom is Mary's aunt's nephew. Which of the following statements is most likely NOT to be true?

a. Tom is Mary's Father b. Tom is Mary's cousin c. Tom is Mary's half brother ANSWER: A NOTES: REF:

d. Tom is Mary's brother e. Tom is related by blood to Mary

57. Which word makes the best analogy? SWIM IS TO WALK as BOAT IS TO:

a. Airplane b. Ship

c. Jet d. Car

ANSWER: D The analogy is water to land. NOTES: REF:

Police Practice Test 4

Page 14 of 26

58. Which word makes the best analogy? PITCH IS TO THROW as STEP IS TO:

a. Touch b. Leg c. Fly ANSWER: E NOTES: REF:

d. Match e. Walk

59. Which number does not belong in the following sequence? 4 6 5 7 6 9

a. four b. five c. six ANSWER: E NOTES: REF:

d. seven e. nine

60. If you purchased an item for 64 cents and you received five coins in change from your one dollar bill, which kind of coin would you receive the most of?

a. Penny b. Nickel

c. Dime d. Quarter

ANSWER: C Change= 3 dimes, one nickle and one penny NOTES: REF:

61. Which item does not belong in this group?

a. Coat b. Watch c. Shirt ANSWER: B NOTES: REF:

d. Socks e. Dress

62. In another language, "clup sonk wita" means "dead bad enemies" and "zorn gin clup" means "enemies hate people" and "fal wita nomi" means "see dead animals." What does "sonk" mean?

a. Dead b. Bad

d. Nothing e. Enemy

Police Practice Test 4

Page 15 of 26

c. Hate ANSWER: B NOTES: REF: 63. Which word is least like the other four?

a. Cold b. Snow ANSWER: B NOTES: REF:

c. Mild d. Hot

64. Which of the five is least like the other four?

a. Car b. Bus c. Wagon ANSWER: E NOTES: REF:

d. Van e. Bicycle

65. If you rearrange the letters "giaccho", what would you have the name of?

a. Animal b. Vegetable c. City
ANSWER: C Chicago NOTES: REF: WRITING SKILLS

d. Country e. Sea

This test measures three aspects of good writing: clarity, vocabulary, and spelling. Instructions: In the following pairs of sentences, identify the sentence which is most clearly written. If sentence "A" is clearer than sentence "B", mark "A" on your answer sheet. If sentence "B" is more clear than sentence "A", mark "B" on your answer sheet. Make no marks in the test booklet.

66. Identify the sentence, which is most clearly written. a. The officer was hoping to get a b. The officer was hoping to get a new partner, one that had a great new partner. One that had a great deal of experience. deal of experience.

Police Practice Test 4

Page 16 of 26

ANSWER: A Alternative "B" is incorrect because "One that had a great deal of experience" is a sentence fragment. NOTES: REF:

67. Identify the sentence which is most clearly written. a. Bullet fragments were gathered by b. Bullet fragments were gathered in officers in envelopes. envelopes by officers. ANSWER: B Alternative "A" is incorrect because it contains a misplaced modifier. The way this sentence is written, the officers collecting the bullet fragments were themselves in the envelope. NOTES: REF:

68. Identify the sentence, which is most clearly written. a. The suspect disliked the officer, as b. The suspect disliked the officer as did the judge. much as the judge. ANSWER: A Alternative "B" is incorrect because it is confusing and ambiguous. Does the suspect dislike both the officer and the judge, or does the judge dislike the officer as much as the suspect? One can't tell the way the sentence is written. NOTES: REF:

69. Identify the sentence which is most clearly written. a. The officer had probable cause to b. When the officer arrived at the arrest the suspect when he arrived scene, he had probable cause to at the scene. arrest the suspect. ANSWER: B Alternative "A" is incorrect because it has an unreferenced pronoun. The way the pronoun "he" is used in this sentence makes it unclear if "he" is referring to the officer or the suspect. NOTES: REF:

70. Identify the sentence, which is most clearly written. a. Jail is not a pleasant place to be, but they do get their basic necessities met. b. Jail is not a pleasant place to be, but prisoners do get their basic necessities met.

Police Practice Test 4

Page 17 of 26

ANSWER: B Alternative "A" is incorrect because the pronoun "they" does not refer to an antecedent noun. In sentence "B", the proper noun "prisoners" makes the sentence clear. NOTES: REF: VOCABULARY Instructions: In each of the following sentences, choose the word or phrase that has the same meaning as the underlined word.

71. The witness corroborated the suspect's story.

a. verified b. contradicted

c. added to d. questioned

ANSWER: A verify means to corroborate, confirm, or attest to contradict means to express the opposite added to means to augment questioned means to cast doubt on NOTES: REF:

72. It was not a very pragmatic plan. a. plausible b. serious

c. practical d. sensible

ANSWER: C plausible means apparently valid or likely serious means earnest or sincere practical means pragmatic sensible means reasonable or wise NOTES: REF:

73. The police sequestered the suspect.

a. caught b. isolated

c. arrested d. released

ANSWER: B caught means to seize or trap isolate means to sequester arrest means to seize or bring into custody release means to let go NOTES: REF: 74. In order to conceal her guilt, Linda told a blatant lie.

Police Practice Test 4

Page 18 of 26

a. harmless b. subtle

c. careless d. obvious

ANSWER: D harmless means not harmful, inoffensive subtle means not obvious careless means done without care or concern obvious means blatant NOTES: REF:

75. He was ignorant of the proper procedures.

a. unaware b. conscious

c. uncertain d. cognizant

ANSWER: A unaware means ignorant conscious means aware uncertain means having questions regarding cognizant means aware NOTES: REF:

READING COMPREHENSION
Instructions Officers must read and understand a wide range of materials. This test is designed to measure your ability to read and understand various types of written material. Read each paragraph or passage and choose the statement which best answers the question. All questions pertain only to the material in the passage which precedes them. Therefore, you are to choose your answer solely on the basis of the material contained in the passage. Mark the letter which identifies your choice in the space on your answer sheet. Do not spend too much time on any one item. The case of Gideon v. Wainwright, decided by the Supreme Court in 1962, granted court-appointed counsel to people charged with felonies who could not afford to pay for their own attorneys. In a more recent case, the Supreme Court declared that the right extends to persons who are charged with any crime for which prison is a potential penalty. According to these court decisions, a defendant may waive his or her right to be represented by counsel at trial, but the state must at least provide the defendant with the opportunity to have free counsel.

76. Based on the above passage, which of the following statements is most correct regarding defendants who cannot afford an attorney: a. anyone who must appear in court c. anyone charged with a felony has the right to a free attorney must have an attorney present for his or her trial

Police Practice Test 4

Page 19 of 26

b. anyone whose charges may result d. anyone charged with an offence in prison must have an attorney that could lead to prison must representing him or her at trial have the option of a free attorney ANSWER: D Alternative "A" is incorrect because it states that "anyone who must appear in court has the right to a free attorney" while the text states that "persons who are charged with any crime for which prison is a potential penalty" have a right to a free attorney. Alternative "B" is incorrect because it states "anyone… must have an attorney representing him at trial." The passage, however, states that "a defendant may waive his or her right to be represented."Alternative "C" is incorrect because it states "anyone charged with a felony must be represented for his trial" while the passage only addresses individuals charged with a crime "for which prison is a potential penalty." NOTES: REF:

77. During the 1970s, rural America registered a population gain of almost 14 percent, while metropolitan growth was just under 10 percent. The surprising spurt in rural growth had nothing to do, however, with prolific reproduction. On the contrary, rural areas were mired in aging townsfolk. This fact, combined with trend toward smaller families, actually resulted in a 10 percent decline in the existing rural population between 1965 and 1970. Newcomers more than made up for that loss. The latest census tells us that one sixth of the U.S. population now lives in rural areas and 40 percent of all new housing in the 1970s was built on rural land. The rural areas are now growing faster than the metropolitan areas for the first time since the early 1800s. Based on the preceding passage, which of the following statements is most accurate? a. Rural areas were mired in aging c. The overall gain in the American townsfolk because the young and population was almost 14 farmers were immigrating to the percent. cities. b. There has been more than a 10 d. The trend toward small families percent increase of newcomers to contributed to the decrease in the rural areas. urban population growth rate. ANSWER: B Alternative "A" is incorrect because the passage says nothing about "the young and farmers immigrating to the cities. "Alternative "C" is incorrect because the distractor states that there was an "overall gain in the American population", but the passage speaks about rural and metropolitan growth, not overall growth. Alternative "D" is incorrect because the passage states that the trend toward smaller families resulted in a "decline in the existing rural population", while the distractor talks about a trend toward smaller families affecting urban growth. NOTES: REF:

78. Entrapment is defined as officers or agents of the government provoking a person to commit a crime, that he did not originally contemplate, in order to prosecute him.

Police Practice Test 4

Page 20 of 26

The goal of law enforcement is not to encourage nor to create an offense, but to prevent people from committing crimes and/or arrest people when they do commit offenses. Based on the preceding passage, which of the following statements describes an instance of entrapment? a. Property that has been marked by c. police so that they can trace it at a later time. b. A victim, learning that a person d. intends to rob him, does nothing to stop the crime, but instead allows the robber to carry through with the crime so that police will catch him in the act.

A door to a warehouse containing valuable merchandise is purposely left open by police. An undercover officer approaches a known drug dealer and tries, unsuccessfully, to purchase drugs.

ANSWER: C Alternative "A" does not constitute entrapment because there is no government provocation to steal the marked items. Alternative "B" is not entrapment because the activity described is undertaken by a "victim", not by the government. Alternative "D" is not entrapment because the officer is not provoking an individual to commit a crime that they were not already contemplating committing. NOTES: REF:

79. Community policing has gained increased acceptance during the past 10 years. The community policing model places service to the public and prevention of crime, as the primary role of police in society. This also emphasizes problem solving, with active citizen involvement in defining those matters that are important to the community, rather than crime fighting and arrest statistics. Officers at the patrol level are required to spend less time in their cars communicating with other officers and more time on the street communicating with citizens. Proponents of this style of policing insist that addressing the causes of crime, makes police officers more effective and at the same time enhances the quality of life in the neighborhood. Community policing concepts, if successfully implemented, offer the prospect of effective crime prevention and substantially improved community relations. Although community-based policing is not a panacea for the problems of crime in society, it does offer valuable opportunity to both reduce crime and enhance the police image. According to the preceding passage, which of the following statements is most accurate? a. Community policing is expected c. to increase the number of arrests made by patrol officers. b. Implementing community policing d. is expected to decrease the amount of crime. Community policing is not designed to lessen crime. The community policing model defers decision-making to citizens and community members.

Police Practice Test 4

Page 21 of 26

ANSWER: B Alternative "A" is incorrect because community policing is not focused primarily on enhancing arrest statistics but rather "places service to the public and prevention of crime as the primary role of police in society." Alternative "C" is incorrect because community policing is intended to lessen crime. Unlike traditional policing, however, this goal is to be achieved through crime prevention, not crime fighting. Alternative "D" is incorrect because decisionmaking is not deferred to the citizens and the community. Community policing does, however, stress "active citizen involvement in defining those matters that are important to the community." NOTES: REF: GENERAL ORDER 62 (Radio Codes) The purpose of this general order is to describe the coding system that identifies the various departmental and non-departmental units. Below you will find three components of police radio codes (A, B, and C). When a dispatcher sends officers out on a call, these 3 code components are used. Each call number shall be comprised of three components: 1. the first digit identifies shift 2. the alpha digit describes the operational assignment 3. the last two digits identifies unit or area A. Call numbers shall be assigned as follows: Shift designation 1 day 2 swing 3 graveyard B. Operational assignments A. patrol division, platoon 1 B. patrol division, platoon 2 C. patrol division, special activities D. patrol division, gang unit E. traffic division, motorcycle unit 1 F. traffic division, motorcycle unit 2 G. traffic division, administration H. investigation division, fraud I. investigation division, drugs J. administration division C. Unit or Area 1-13 Metropolitan area 13-20 Airport 21-40 Industrial parks 41-45 Residential areas

80. Using the above coding system, the call number 2-B-12 would identify a. swing shift, platoon 2, c. swing shift, administration, metropolitan area residential b. graveyard, platoon 1, airport d. day, traffic unit 1, airport

Police Practice Test 4

Page 22 of 26

ANSWER: A Alternatives "B", "C", and "D" are immediately recognized as incorrect because the last element of the code (12) refers to the metropolitan area, and only one of the choices, "A", meets this condition. NOTES: REF: SPELLING

81. His ______________ of the situation was incorrect.

a. analisys b. annalysis ANSWER: C NOTES: REF:

c. analysis d. anallysys

82. It was not _____________ who committed the crime.

a. apparent b. apperant ANSWER: A NOTES: REF: 83. She was a ____________ worker.

c. aparent d. aperant

a. conscienteous b. consceintious ANSWER: D NOTES: REF:

c. consceinteous d. conscientious

84. She did not actually _____________ the stolen goods.

a. receive b. recieve ANSWER: A NOTES: REF:

c. recieve d. recive

85. There was only one witness to the boy's ______________.

a. abducton b. abducsion

c. abduction d. abduckion

Police Practice Test 4

Page 23 of 26

ANSWER: C NOTES: REF: 86. Propaganda is a(n) — of truth, a mixture of half-truths and half-lies calculatedto deceive.

a. revision b. perversion c. dissension ANSWER: B NOTES: REF: 87. NOSE: HEAD::

d. perception e. invasion

a. hand: arm b. foot: toe c. eye: lid ANSWER: A NOTES: REF: 88. WHEAT: GRAIN ::

d. wrist: finger e. teeth: gums

a. cow: beef b. orange: lime c. carrot: vegetable ANSWER: C NOTES: REF: 89. COTTAGE: CASTLE::

d. coconut: palm e. hamburger: steak

a. house: apartment b. puppy: dog c. dory: liner ANSWER: C NOTES: REF: 90. OLD: ANTIQUE::

d. man: family e. poet: gentleman

a. new: modem b. cheap: expensive c. useless: useful

d. wanted: needed e. rich: valuable

Police Practice Test 4

Page 24 of 26

ANSWER: A NOTES: REF: 91. DIGRESS: RAMBLE::

a. muffle: stifle b. rust: weld c. introduce: conclude ANSWER: A NOTES: REF:

d. rest: stir e. find: explain

92. If a car can drive 25 miles on two gallons of gasoline, how many gallons will be needed for a trip of 150 miles?

a. 12 b. 3 c. 6 ANSWER: A NOTES: REF:

d. 7 e. 10

93. A school has enough bread to feed 30 children for 4 days. If 10 more children are added, how many days will the bread last?

a. 5 1/3 b. 1 1/3 c. 2 2/3 ANSWER: E NOTES: REF:

d. 12 e. 3

94. He dashed into the house, ran for the phone, and answered—, tripping over the cord.

a. hesitantly b. nobly c. soothinsly ANSWER: E NOTES: REF:

d. distantly e. breathlessly

95. The criminal record of the witness caused the juryto ------ his testimony.

a. affirm b. belie c. retract

d. acauit e. discredit

Police Practice Test 4

Page 25 of 26

ANSWER: E NOTES: REF: 96. Write .2% as a decimal.

a. .2 b. .02 ANSWER: C NOTES: REF: 97. Write 3.4% as a fraction.

c. .002 d. .2

a. 34/1000 b. 34/10 ANSWER: A NOTES: REF: 98. Write ¾ % as a decimal.

c. 34/100 d. 340/100

a. 75 b. .075 c. .0075
ANSWER: C ¾ % = .75% =.0075 NOTES: REF:

d. .00075 e. 7.5

99. Find 60% of 70.

a. 420 b. 4.2 c. 1162
ANSWER: E 60 % = 3/53/5of 70 = 42 NOTES: REF:

d. 4200 e. 42

100. By a stroke of luck, the troops -------, avoiding a crushing -------.

a. converged-blow b. transmuted-oblivious

c. prevailed-defeat d. diverged-siege

Police Practice Test 4

Page 26 of 26

ANSWER: C NOTES: REF:

Police Practice Test 1

Page 1 of 22

Police Practice Test 5 Answer Key
Multiple Choice Identify the letter of the choice that best completes the statement or answers the question.

1. In the following pairs of sentences, identify the sentence, which is most clearly written. If sentence "A" is clearer than sentence "B", mark "A" on your answer sheet. If sentence "B" is more clear than sentence "A", mark "B" on your answer sheet. a. Bullet fragments were gathered by b. Bullet fragments were gathered in officers in envelopes. envelopes by officers ANSWER: B "A" is incorrect because it contains a misplaced modifier. The way this sentence is written, the officers collecting the bullet fragments were themselves in the envelope. NOTES: REF:

2. Identify the sentence, which is most clearly written. a. The officer was hoping to get a new partner, one that had a great deal of experience. b. The officer was hoping to get a new partner.One that had a great deal of experience.

ANSWER: A "B" is incorrect because "One that had a great deal of experience" is a sentence fragment. NOTES: REF:

3. Identify the sentence, which is most clearly written. a. The suspect disliked the officer as did b. The suspect disliked the officer as the judge. much as the judge.

ANSWER: A "B" is incorrect because it is confusing and ambiguous. Does the suspect dislike both the officer and the judge, or does the judge dislike the officer as much as does the suspect? One can't tell the way the sentence is written. NOTES: REF:

4. Identify the sentence, which is most clearly written. a. The officer had probable cause to arrest the suspect when he arrived at b. When the officer arrived at the scene, he had probable cause to arrest the

Police Practice Test 1

Page 2 of 22

the scene.

suspect.

ANSWER: B "A" is incorrect because it has an unreferenced pronoun. The way the pronoun "he" is used in this sentence makes it unclear if "he" is referring to the officer or the suspect. NOTES: REF:

5. Identify the sentence, which is most clearly written. a. Jail is not a pleasant place to be, but b. Jail is not a pleasant place to be, but prisoners do get their basic necessities they do get their basic necessities met. met.

ANSWER: B "A" is incorrect because the pronoun "they" does not refer to an antecedent noun. In sentence "B", the proper noun "prisoners" makes the sentence clear. NOTES: REF: OBSERVATION This section will test your ability to attend to details in visual material. For each questions, you will examine a set of five sketches of faces. For each set, the first drawing will be identified as a sketch of an alleged criminal. The other drawings will be labeled “a”, “b”, “c”, and “d” For each set of drawings, one of the drawings labeled “a”, “b”, ”c”, or “d” will have near identical facial features as that identified as the alleged criminal. Your task is to identify this drawing. In making your decision, you are to assume that no plastic surgery has occurred.

To complete the task accurately, focus on features that are unlikely to be altered without plastic surgery. For example, examine the shape of the eyes, the size and appearance of the nose, the shape of the face, etc. Do not be distracted by features that can easily be altered. For example, hair style and color, facial hair, and clothing. Use a process of elimination when completing the task. Eliminate any pictures where you notice a difference in a feature that should not change. Once three pictures have been eliminated, the remaining picture should be the correct answer. 6. Identify the suspect:

Police Practice Test 1

Page 3 of 22

Which of the above drawings is most likely the suspect? a. Mug shot A c. Mug shot C b. Mug shot B d. Mug Shot D ANSWER: B In option “a”, notice that the nose is different. In option “c”, notice that the eyes are different. In option “d”, notice that the ears and the shape of the face are different. Each of these differences are unlikely to occur without plastic surgery. On the other hand, the only difference between option “b” and the suspected criminal is that the picture includes a mustache. This would be a very simple disguise. Option “b” is therefore the most likely match to the suspected criminal. NOTES: REF:

Police Practice Test 1

Page 4 of 22

7. For each set of drawings, one of the drawings labeled “a”, “b”, ”c”, or “d” will have near identical facial features as that identified as the alleged criminal. Your task is to identify this drawing.

Use a process of elimination when completing the task. Eliminate any pictures where you notice a difference in a feature that should not change.
Which of the above drawings is most likely the suspect? a. Mug shot A c. Mug shot C b. Mug shot B d. Mug shot D ANSWER: B In option “a”, notice that the shape of the face is different. In option “c”, notice that the

Police Practice Test 1

Page 5 of 22

NOTES: REF:

shape of the mouth is different. In option “d”, notice that the nose is different. Each of these differences are unlikely to occur without plastic surgery. On the other hand, the only difference between option “b” and the suspected criminal is that the picture includes eyeglasses. This would be a very simple disguise. Option “b” is therefore the most likely match to the suspected criminal.

STREET SCENE Answer the following questions based on the following crime scene sketch. Sketches often present unclear images so your task is not only to identify the events in the picture but also figure out what the sketches represent.

You'll have five minutes to examine the sketch. The questions will test your ability to recall facts, figures and events occurring on the sketch.

You have five minutes to examine the sketch.

8. Which one of the following vehicles is parked on Rock Ave ? a. A police car c. A tow truck b. A taxi d. A fire truck

ANSWER: B

Police Practice Test 1

Page 6 of 22

NOTES: REF: 9. What are the words printed on the side of the bus?

a. Northside bus b. Southside bus ANSWER: A NOTES: REF: 10. The man with the gun is wearing a:

c. Eastside bus d. Westside bus

a. white shirt with black stripes b. T-shirt with the words spot news ANSWER: D NOTES: REF:

c. sweatshirt with the word Ace d. white shirt with black dots

11. At what intersection is the man changing the tire?

a. Rock and Elm b. Hess and London ANSWER: A NOTES: REF:

c. Howard and Crawford d. Smith and First

12. The person in the second floor window is directly above;

a. Uncle Dan's Lunch b. Mandley Discount Clothing Outlet ANSWER: D NOTES: REF:

c. Cafeteria Europa d. M & P Music World

13. What is the man with the gun stealing from the woman?

a. a purse b. a necklace ANSWER: B NOTES: REF:

c. earings d. a car

14. The ability to identify and resolve problems, and also to apply the principles of logic to given situations, is needed to perform many state jobs.These types of questions are designed to measure the applicant's ability in these areas.

Police Practice Test 1

Page 7 of 22

New York time is three hours ahead of San Francisco time. What time would it be in New York if a San Francisco clock, one hour behind the time, shows 4 o'clock? a. 1 o'clock c. 7 o'clock b. 6 o'clock d. 8 o'clock
ANSWER: D The correct answer is D. If a San Francisco clock shows 4:00 and is one hour behind (slow), the actual time in San Francisco is 5:00. Therefore, the time in New York three hours ahead, would be 8:00, or choice D. NOTES: REF:

15. Most of the Gaspard family lives in South Louisiana. Many of the people in South Louisiana celebrate Mardi Gras. Frank is a member of the Gaspard family. Based on these facts, we can conclude that: a. Frank lives in South Louisiana. c. All of the Gaspards celebrate Mardi Gras. b. Frank does not celebrate Mardi Gras. d. None of the above can be concluded. ANSWER: D The correct answer is D. Choice a cannot be concluded, since most, but not all of the Gaspard family, lives in South Louisiana. Choice b is not supported by the passage at all. Choice c cannot be concluded, because the passage states that many, not all, people in South Louisiana, celebrate Mardi Gras. Therefore, the answer is none of the above, or choice D. NOTES: REF:

16. Bill must use Highway 19 to get to work. Bill has a meeting today at 9:00 AM. If Bill misses the meeting, he will probably will lose a major account. Highway 19 is closed all day due to repairs. Based on the above, we can conclude that: a. Bill will not be able to get to work. c. Bill will lose a major account. b. Bill will probably not be able to d. None of the above can be concluded. reschedule the meeting. ANSWER: A The passage does not address whether or not Bill will be able to reschedule his meeting, so choice b cannot be concluded. Choice c cannot be concluded, because the passage says he will probably lose a major account, but this is not a certainty. Choice a is a correct conclusion, because the passage states that Bill must use Highway 19 to get to work, and that Highway 19 is closed all day. Therefore, it can be concluded that Bill would not be able get to work that day. NOTES: REF:

17. Mrs. Sellers reports that she was in the elevator late yesterday evening after leaving her office on the 15th floor of a large office building. A man got on at the 12th floor, pulled her off the elevator, and assaulted her, stealing her purse. She believes that she has seen the man in elevators and hallways of the building before. She thinks that he works in the building. Study the following parts of Mrs. Sellers' description of the man. Which one would most useful in

Police Practice Test 1

Page 8 of 22

finding him, assuming that he is a regular occupant of the building? a. He had very bad breath. c. He had a scar on his left cheek. b. He was wearing a striped tie. d. He was carrying a blue backpack. ANSWER: C The correct answer is c. Choices a, b, and d all describe characteristics that could be easily changed or hidden. However, a scar on the face is noticeable and could not be easily disguised or removed. Therefore, the best answer is choice c. NOTES: REF:

Reading Comprehension

Many state jobs require skill in analyzing, understanding, and interpreting written material of varying levels of difficulty. The reading comprehension questions on our tests are designed to measure applicants' abilities to understand and interpret written material. These questions require an employee to read and understand a paragraph, and then, to choose an answer based on their understanding of the main concept put forth in the written passage. The correct answer will usually restate this main concept, using different wording. In some cases, the correct answer will be a conclusion that is drawn from the content of the paragraph. After reading the passage, choose as your answer the statement that is best supported by the contents of the passage. A viable affirmative action program must contain specific procedures designed to achieve equal employment opportunities for specified groups. Appropriate procedures, without necessary determination to carry them out, are useless. Determination, without well defined procedures, will achieve only partial success.

18. The paragraph best supports the statement that: a. Well defined procedures will assure thed. An agency may guarantee success of success of an affirmative action its affirmative action program by program. developing and implementing well defined procedures. b. A high degree of determination is e. Two important ingredients of a necessary and sufficient for a highly successful affirmative action program successful affirmative action program. are well defined procedures and a sincere resolve to implement those procedures. c. It is impossible for an agency to develop a viable affirmative action program. ANSWER: E The correct alternative, E, restates the idea presented in the paragraph. Statements A and B each contain only one of the ingredients. Alternative D overstates the implications of the paragraph. NOTES: REF:

19. Education in the United States is a state responsibility, a local function and a federal concern. Unlike other social service programs, this arrangement also places state governments between the federal government and local governing bodies. The paragraph best supports the statement that: a. The federal government plays an d. No federal funds are used to support

Police Practice Test 1

Page 9 of 22

advisory role only in matters concerning education. b. Enforcement of federal education policies is left to state discretion. c. Federal educational policies are generally implemented by local governments under the direction of the state.

local educational programs. e. Federal aid is often used to induce local school systems to implement federal policies.

ANSWER: C The correct alternative C, is supported by the paragraph. The ideas expressed in alternatives A, B, and D are not addressed in the paragraph. Although alternative E is probably true, it is not mentioned in the paragraph. NOTES: REF:

20. Technological and psychological conditions are changing so rapidly that most agencies and organizations must continually adapt to new situations in order to remain viable. The paragraph best supports the statement that: a. Changes in general conditions d. The effectiveness of an organization is determine the effectiveness of an equally dependent upon technological organization. advances and psychological changes. b. The effectiveness of an organization e. The effectiveness of an organization is depends more on technological dependent upon its technological and advances than on psychological psychological advances. changes. c. Organizations must be able to adapt to technological and psychological changes in order to maintain effectiveness. ANSWER: C Correct alternative C, effectively restates the essence of the paragraph. In contrast to alternatives A and B, the paragraph states that organizations must adapt to changes. Alternatives D and E imply that effectiveness of an organization depends on change; however, the paragraph states that effectiveness depends on an organization's ability to adapt to change. NOTES: REF:

21. Choose the sentence that represents the best English Usage. a. Of the two runners, John is the worst. c. John is the worst of the two runners. b. Of the two runners, John is the better. d. John is the best of the two runners. ANSWER: B The correct answer is choice B. The sentence with the best English Usage is choice B. The correct way to write choice A is "Of the two runners, John is worse." The correct way to write choice C is "John is the worse of the two runners." The correct way to write choice D is "John is the better of the two runners." NOTES:

Police Practice Test 1

Page 10 of 22

REF: 22. Choose the sentence that represents the best English Usage. a. We seldom ever receive this type of c. Each of these regulations apply to request anymore. your case. b. Neither of the employees are doing d. I have enclosed a copy of the file you what is expected of him. requested. ANSWER: D The sentence with the best English Usage is choice D. The correct way to write choice A is "We seldom receive this type of request." The correct way to write choice B is "Neither of the employees is doing what is expected of him." The correct way to write choice C is "Each of these regulations applies to your case." NOTES: REF:

23. Both women have made previous complaints, none of which were followed through because of insufficient evidence or strong fear of retaliation by the suspect. The suspect is attempting to intimidate the women and is known for irrationale behaviour. Which word in the sentences above is misspelled?

a. insufficient b. retaliation

c. intimidate d. irrationale

ANSWER: D The correct spelling for option d is “irrational” (i.e., without the “e” at the end). NOTES: REF:

24. On the night of the accident, coeficient of friction testing was conducted using police transport on a cycloidal skid mark. Which word in the sentence above is misspelled? a. accident c. friction b. coeficient d. cycloidal ANSWER: B The correct spelling for option b is “coefficient” (i.e., two “f”s). NOTES: REF:

25. Which word completes the next sentence? Once she became a public figure, she had to give __________ her anonymity.

a. about b. back

c. in d. up

ANSWER: D Webster’s Ninth New Collegiate Dictionary (1984) defines give up as “to cease to do some action: ABANDON”. This definition works well at completing a meaningful sentence. The

Police Practice Test 1

Page 11 of 22

NOTES: REF:

other options do not.

26. Which words complete the next sentence? My partner and _____ went to the home and _____ knocking on the door.

a. I, began b. me, began

c. me, begun d. I, begun

ANSWER: A The first blank is for one of the subjects of the verbs “to go” and “to begin”. “I” and not “me”, which is used as an object of a verb, is the appropriate pronoun to use as the subject of the verbs. Next, the sentence is written in the past (went is the past tense of “to go Began is the past tense of “to begin”. NOTES: REF:

27. Which of the following is the best definition of the word amendment?

a.

cancellation by making invalid or outdated b. identification by comparison and elimination

c. improvement by revision or correction d. protection by establishing rules and laws

ANSWER: C ITP Nelson Canadian Dictionary of the English Language: An Encyclopedic reference (1997) defines amendment as “the act of changing for the better; improvement.” Ninth New Collegiate Dictionary (1984) defines amendment as, “the act of changing or modifying for the better.” Option “c” is the best match of these definitions. NOTES: REF:

Reading Comprehension:
Passage :

Throughout its long history, China has seen the rise and fall of a large number of secret organizations. Originally, most of them had no criminal intent. They were simply brotherhoods based on shared political purposes and mutual assistance. However, some of them fell under the control of dishonest people. They then drifted into crime and ended up posing a threat to the social order. This was the case for two of the largest secret organizations of Chinese modern history, the Tsing and Hong organizations. Many criminal organizations such as the Tsing and Hong and many others controlled brothels, opium dens, casinos and drug trafficking operations. They forced business owners to give them a percentage of their profit. They also committed more serious crimes such as abduction, the trade of women and children, and even assassinations. To achieve impunity from the law, they corrupted government and colonial officials. It was not uncommon for leaders of these organizations to occupy legitimate positions in companies or even in government agencies. They used these jobs as a front for their illegal activities. Each organization had its own

Police Practice Test 1

Page 12 of 22

system of laws and punishments.

The internal law of the Hong organization had five possible punishments: capital punishment, corporal punishment, caning, degradation and banishment. A strict hierarchy existed and obedience to superiors was mandatory. New members, called apprentices, were placed under a master and were at his service. In the golden age of the Tsing and Hong organizations, a powerful leader could recruit thousands of apprentices. 28. According to the above passage, which one of the following statements is true? a. Chinese secret organizations have always threatened the social order. b. Financial assistance is encouraged in Chinese criminal organizations. c. Few criminal organizations are known in China; only two are known today. d. Some secret Chinese organizations have not turned to crime.

ANSWER: D Option “a” is wrong because the passage includes the statement stating, “Originally, most of them (secret organizations) had no criminal intent.” The concept of financial assistance as stated in option “b” is never mentioned in the passage. Therefore, it is not supported by the passage. Option “c” mentions that ONLY two criminal organizations are known. The passage actually implies that there are more than the two (“This was the case for the largest secret organizations of Chinese modern history”). Option “d” is the only option supported by the passage (“some of them [not all] fell under the control of dishonest people”). NOTES: REF:

29. According to the previous passage, which one of the following statements is true? Criminal organizations bribed c. The trade of women and children was government employees and senior the main activity of Chinese criminal officials to avoid facing consequences organizations. of their misdeeds. b. All leaders of these organizations had d. Prostitution is one of the rare illegal legitimate positions in businesses or in businesses in which Chinese criminal the government. organizations did not participate. ANSWER: A Option “b” is wrong because is states that all leaders had legitimate positions in businesses or in the government. The passage states that it was not uncommon for this to be, but not that it always happened. Option “c” is wrong because the passage makes no claim that the trade of women and children is the main activity of the secret organizations, only that it is one of the illegal activities performed. Option “d” is wrong because it is specifically mentioned in the passage that the organizations controlled brothels. Option “a” is the only option supported by the passage (“To achieve impunity from the law, they corrupted government and colonial officials”). NOTES: REF: a.

30. Which one of the following statements is supported by the above passage?

Police Practice Test 1

Page 13 of 22

a.

In the Hong organization, only c. In the Hong organization, the penalty apprentices had to obey their superiors. for disobedience was corporal punishment. b. In the Hong organization, a persons d. In the Hong organization, the leader rank could not be lowered as a form of recruited many new members. punishment. ANSWER: D Option “a” is wrong because the passage includes the statement, “A strict hierarchy existed and obedience to superiors was mandatory.” This statement is not limited to being an apprentice. Option “b” is wrong because the passage identifies degradation (i.e., a reduction in rank) as a form of punishment. Option “c” is wrong because the passage never specifies any penalty that is associated with a particular crime. Option “d” is the only option supported by the text (“a powerful leader could recruit thousands of apprentices (Apprentices was defined earlier in the passage as new members). NOTES: REF:

31. This section will test your ability to use appropriate resources and strategies to achieve objectives. You demonstrate good judgement by reaching sound decisions and taking the appropriate courses of action. The term judgement is used synonymously with the concept of common sense.

You are a police officer in a small town. You are on patrol at 2:00 a.m. when you see a car leaving town and moving very fast. You turn on the lights and sirens of your police car and attempt to pull the car over. The car does not pull over and a brief high speed chase results. After a short distance, the driver loses control of the car and it skids into the ditch. Four men jump out of the car and run into the forest. Of the following, which is the best initial course of action to take? Call for backup and immediately chase c. Determine the registered owner of the the suspects into the forest. car. The following day, go to the registered car owner's home and question him about the incident. b. Call for backup and broadcast a d. Draw your gun and order the men to description of the suspects. When stop. If they do not comply, fire a backup arrives, begin the chase of the warning shot into the air suspects into the forest.
a. ANSWER: B Option “b” is the most effective action because you have called for back up in a high risk situation. In addition, you have taken steps, once sufficient resources are obtained, to apprehend the individuals. Option “a” is wrong because you have placed yourself at considerable risk by pursuing four males at night in a forest. Option “c” is wrong because the vehicle may be stolen and you have made no attempt to apprehend the suspects at the time when options existed for you to do so. Option “d” is wrong because it is very dangerous to fire a shot into a dark forest. NOTES: REF:

32. You are a police officer in a large municipality. Your coworker and friend, Officer Grey, is in constant financial trouble as the result of loans that she obtained before joining the PD. Grey is

Police Practice Test 1

Page 14 of 22

married and has recently taken on a second job to pay her bills. You have noticed that since she has had this second job, the quality of her work has gone down considerably. Of the following, which is the best initial step to take?
a. Suggest to Officer Grey that she speak c. Speak to Officer Grey's spouse to to a financial consultant. identify possible solutions to her financial difficulties. b. Lend Officer Grey some money. d. Speak to Officer Grey and tell her about your concerns. ANSWER: D Option “d” is the most effective initial option because Officer Grey is your friend and you have knowledge of the situation. Options “a” is wrong because, although you have provided good advice, there is no indication that Grey has not already contacted a financial advisor. Option “c” is wrong because you have meddled too far into a private situation. The reason you should confront her is BECAUSE it’s effecting her work as a police officer. NOTES: REF:

33. At a meeting, your supervisor tells all the members on shift about a new police policy. The policy states that police officers are to wear their hats at all times while on duty in the downtown area. The rationale for the policy is that the wearing of hats projects a professional policing image and allows the public to easily identify the person as a police officer. Later that evening, you and your partner receive a call that a young female was just sexually assaulted by two males who are both well-known to yourself. The two males are to be arrested.

You believe that the two male suspects are at a fountain in the downtown area that is only approachable on foot. There is a music festival being held in the area that is attracting large crowds of people. You and your partner feel that, under these circumstances, it would be inappropriate to wear your hats; the suspects would likely see you approaching and may flee. Of the following, which is the best course of action to take?
Call your supervisor, explain the c. situation, and request permission not to wear your hats. Follow the decision given. b. Proceed to the area without wearing d. your hats. Explain to your supervisor afterwards the reason for your action. a. Wear your hat while in the area, consistent with the police policy. Do not attempt to arrest the suspects at this time as their identity is known. Wait for another time to arrest them.

ANSWER: B Option “b” is correct because you have analyzed the risk of not wearing the hat and exercised common sense that, in this case, following police policy will compromise your ability to arrest the suspects in a safe manner. Option “a” is wrong because immediate action is required to apprehend the suspects and should not be delayed due to a minor policy issue. Option “c” is wrong because you have not exercised any decision-making or judgment and have simply followed policy that is not appropriate in this situation; the suspects may flee and, consequently, re-offend. Option “d” is wrong because you are taking a risk of losing the suspects and evidence because of a minor policy issue. NOTES:

Police Practice Test 1

Page 15 of 22

REF: 34. Logic problems:

You are preparing a report concerning a car accident. The following five pieces of information are to be included in the report: 1. Officer MacInnis explained that the car was heading east bound on 84th Avenue in the first lane and had a struck a young girl who was running across 84th Avenue. 2. The statement was not obtained at this time, as the driver of the car was too emotionally upset. 3. Officer Smith attended the scene of a car accident and was briefed by Officer MacInnis of what had occurred.
4. The driver was then turned over to Officer Smith for a statement.

5. Officer MacInnis then stated that he had detained the driver of the car and that he had read him his rights.
In which of the following order should the information in the report be presented?

a. 1, 5, 2, 4, 3 b. 3, 5, 1, 4, 2

c. 1, 2, 4, 3, 5 d. 3, 1, 5, 4, 2

ANSWER: D There are several ways in which to correctly arrive at this answer. Here is one way: The four options indicate that there are two possible first statements: statement 1 (options “a” and “c”) and statement 3 (options “b” and “d”). In statement 1, Officer MacInnis is explaining the situation to someone, yet it is unclear as to whom. In statement 3, Officer Smith is just arriving at the scene and begins the briefing with Officer MacInnis. Therefore, Officer Smith is likely the person being spoken to in statement 1, such that statement 3 should occur before statement 1. This narrows the possible correct option to options “b” and “d”. In option “b”, statement 5 is the next piece of information to use. However, the statement seems to imply some other statement had been made (“Officer MacInnis then stated...”). Statement 1, the second piece of information to use in option “d”, is very likely this statement and logically should precede statement 5. Therefore, option “d” is the correct answer. NOTES: REF:

35. You are investigating a serious accident involving five cars. The car at the front of the accident is labeled 1 and the car at the back of the accident is labeled 5. The remaining cars are labeled according to their location in the chain of cars. The information related to the accident is sketchy and somewhat disorganized. You know the following: I. The drivers involved in the accident are Paul, Kim, Jennifer, Kevin, and Sarah. II. Kevin was in one of the cars ahead of Kim III. Sarah and Jennifer were in car 1 and 5, although not necessarily in that order. Which one of the following statements must be false?

Police Practice Test 1

Page 16 of 22

a. Kim is in a car ahead of Sarah b. Kim is in car 2

c. Paul is in car 2 d. Kim is in a car behind Paul

ANSWER: B There are several ways in which to correctly arrive at this answer. Here is one way: This is a problem where it is extremely helpful to draw a diagram to organize yourself. From the information in the problem text and in clue I, the following diagram can be used

Place the information found in the clues into the table. Clue III is the most useful, so we will start with that one. From this clue, we can state that Sarah and Jennifer are not in cars 2, 3, or 4. Similarly, we can state that Paul, Kim and Kevin are not in cars 1 or 5. This information can be placed in the table.

Next, Clue II states that Kevin was ahead of Kim. From the information currently in our table, The farthest back Kim can be is in car 4. Therefore, to ensure that Kevin remains ahead of Kim, the farthest back Kevin can be is in car 3. Similarly, from the information currently in our table, the farthest in front Kevin can be is in car 2. Therefore, to ensure that she remains behind Kevin, the farthest front Kim can be is in car 3. This information can be placed in the table.

Police Practice Test 1

Page 17 of 22

NOTES: REF:

Now we are ready to answer question 1. Option “a” is wrong, because there is a possibility that Sarah could be in car 5 such that the option could be true. Option “c” is wrong because there is the possibility that Paul is in car 2. Option “d” is wrong because there is the possibility that Paul is in car 2 and Kim is in car 3. Option “b” is the correct option, because we have concluded from the information provided that Kim cannot be in car 2.

36. If Kevin is in car 3, which one of the following statements must be true?

a. Paul is in a car behind Kim b. Kim is in a car ahead of Sarah

c. Sarah is in car 1 d. Paul is in a car ahead of Kevin

ANSWER: D Let us continue with using the tables generated in the last question. We have a new piece of information: Kevin is in car 3. With Kevin in car 3, this means that Kim must be in car 4 (Clue II). In that Jennifer and Sarah are either in cars 1 or 5 (clue III), this means that Paul must be in car 2. This information can be placed in the table.

NOTES: REF:

Option “a” is wrong because we have concluded that Paul is car 2, ahead of Kim in car 4. Option “b” is wrong because there remains the possibility that Sarah is in car 1, ahead of Kim in car 4. Option “c” is wrong because there remains the possibility that Sarah is in car 5. Option “d” is correct because we have concluded that Paul is in car 2, ahead of Kevin in car 3.

37. How much is 150% of 80?

Police Practice Test 1

Page 18 of 22

a. 1.8 b. 5.3

c. 70.0 d. 120.0

ANSWER: D To solve this question you must multiply 80 by 1.5 (which is 120 or choice D). NOTES: REF:

38. Of the 300 people working at a medical facility, 14% are physicians' assistants. How many workers at the medical facility are not physicians' assistants?

a. 42 b. 86

c. 258 d. 286

ANSWER: C SOLUTION: To solve this question you must determine that 86% (100% minus 14%) of the workers are not physicians assistants, and then find out how many workers that is (300x86% = 258). NOTES: REF:

39. A pump discharges water at a rate of 184 gallons per minute. Which one of the following is the number of gallons of water this pump will discharge in an hour?

a. 1,104 gallons b. 1,840 gallons

c. 2,208 gallons d. 11,040 gallons

ANSWER: D To solve this question you must know that there are 60 minutes in an hour. So you multiply 184 gallons (the amount discharged in one minute) by 60 (the number of minutes in an hour). 60 x 184 = 11,040 gallons (D). NOTES: REF:

40. An office received 2,400 license applications during the month of April, 1/5 of which were for tavern licenses. If 1/8 of the applications for tavern licenses were rejected, how many applications for tavern licenses were ACCEPTED?

a. 60 b. 180

c. 420 d. 780

ANSWER: C To solve this question, you must first determine the number of applications for tavern licenses. In this case, 1/5 of ALL applications were for tavern licenses, so there were 480 tavern license applications (1/5 X 2400 = 480). Of the 480 tavern license applications, 1/8 of these were rejected. To determine the number of rejected tavern license applications, you multiply 480 X 1/8. The result is 60. To find the number of tavern license applications that were ACCEPTED, you subtract the number of rejected tavern license applications, 60, from the total number of tavern license applications, 480. The result, 420, is the number of tavern license applications that were ACCEPTED.

Police Practice Test 1

Page 19 of 22

NOTES: REF: 41. Cincinnati had a population of three hundred sixty-four thousand, one hundred fourteen people in 1990. What would be the population of Cincinnati if the city grew by ten thousand?

a. 365,114 b. 374,114 ANSWER: B NOTES: REF:

c. 464,114

42. Round the numbers 42 and 31 to the nearest ten. Then subtract the rounded numbers to estimate the difference. Which statement is true? a. Your estimate is larger than the exact c. Your estimate is the same as the exact difference. difference.

b. Your estimate is smaller than the exact difference.

ANSWER: B NOTES: REF: 43. About 11 million people live in Ohio. About how many of them live in cities? Use the graph to answer the question.

Police Practice Test 1

Page 20 of 22

a. 8 million b. 3 million ANSWER: A NOTES: REF:

c. 1 million

44. Navin wants to make a small walkway between the house and the garage, using red and white bricks of the same size. He wants to use 32 bricks in all, 8 red and 24 white. What fraction of the bricks will be red? a. b. c. d.

ANSWER: D NOTES: REF: 45. Look at the graph line below. The greatest difference in daily high temperatures occurred between which two consecutive days?

Police Practice Test 1

Page 21 of 22

a. Between Tuesday and Wednesday b. Between Monday and Tuesday ANSWER: A NOTES: REF:

c. Between Friday and Saturday d. Between Sunday and Monday

46. Mr. Wright is planning to teach five self-defense classes, one class each day. On Monday, Wednesday, and Friday, he can have 20 students in each beginner’s class. On Tuesday and Thursday, he can have 10 students in each advanced class. What is the total number of students Mr. Wright can enroll in all five classes? Choose the expression that will help you solve the problem. a. b. c. d.

ANSWER: D NOTES: REF: 47. Which of the following sentences best describes a basic principle of a democracy?

A democracy is a governmental system c. in which the people have protection from aggressors. b. A democracy is a governmental system d. in which the people have equal economic opportunities.
a.

A democracy is a governmental system in which the people have the right to petition their government. A democracy is a governmental system in which the people have the power to govern.

ANSWER: D NOTES: REF: 48. While serving on a jury, a citizen is expected to

Police Practice Test 1

Page 22 of 22

a.

go along with the majority to ensure a c. exercise an independent judgment case is successful. before deciding how to vote. b. consult independently with the judge d. protect the individual defendant whenever a question of law emerges. against the majority opinion.

ANSWER: C NOTES: REF: 49. Many presidents have asked Congress to pass a law giving them the line item veto. Congress has always resisted primarily because this would

give the executive branch added power c. increase the federal deficit, thus to check the legislative branch. reducing Congress’s ability to spend more money. b. furnish lobbyists with a mechanism for d. force Congress to reduce the number increasing their influence on Congress. of staff working for its members.
a.

ANSWER: A NOTES: REF: 50. Jim is arrested in a criminal case. He refuses to answer any questions about the alleged crime, claiming that he is “taking the Fifth Amendment.” “Taking the Fifth Amendment” means that a. Jim is pleading innocent to the charges c. Jim does not have to answer questions against him. that might incriminate him. d. Jim is too young to be arrested for this crime.

b. Jim has not had the opportunity to discuss the case with a lawyer.

ANSWER: C NOTES: REF:

Police Math Test

Page 1 of 30

Police Math Test Answer Key
Multiple Choice

Identify the letter of the choice that best completes the statement or answers the question.

1. Graph Question A line graph is a way to summarize how two pieces of information are related and how they vary depending on one another. The numbers along a side of the line graph are called the scale. Example 1:

The graph above shows how John's weight varied from the beginning of 1991 to the beginning of 1995. The weight scale runs vertically, while the time scale is on the horizontal axis. Following the gridlines up from the beginning of the years, we see that John's weight was 68 kg in 1991, 70 kg in 1992, 74 kg in 1994, and 73 kg in 1995. Examining the graph also tells us that John's weight increased during 1991 and 1995, stayed the same during 1991, and fell during 1994. What was John’s weight in 1993?

a. 74 kg

c. 77 kg

Police Math Test

Page 2 of 30

b. 71 kg

d. 79 kg

ANSWER: A

NOTES: REF:

Correct answer is 74

2. Graph question: Example 2:

This line graph shows the average value of a pickup truck versus the mileage on the truck. When the truck is new, it costs $14000. The more the truck is driven, the more its value falls according to the curve above. Its value falls $2000 the first 20000 miles it is driven. When the mileage is 80000, what is the truck's value?

a. $2000 b. $4000

c. $8000 d. $1200

ANSWER: B

NOTES: REF:

When the mileage is 80000, the truck's value is about $4000.

Police Math Test

Page 3 of 30

3. The pie chart below shows the ingredients used to make a sausage and mushroom pizza. The fraction of each ingredient by weight is shown in the pie chart below. We see that half of the pizza's weight comes from the crust. The mushrooms make up the smallest amount of the pizza by weight, since the slice corresponding to the mushrooms is smallest. Note that the sum of the decimal sizes of each slice is equal to 1 (the "whole" pizza").

The cheese and sausage makes up what percentage of the pizza?

a. 24% b. 45%

c. 53% d. 33%

ANSWER: D The cheese and sausage makes up 33 %.

NOTES: REF:

0.075 + 0.25 = 0.325 which rounded off = %33

4. The pie chart below shows the fractions of dogs in a dog competition in seven different groups of dog breeds. We can see from the chart that 4 times as many dogs competed in the sporting group as in the herding group. We can also see that the two most popular groups of dogs accounted for almost half of the dogs in the competition.

Police Math Test

Page 4 of 30

Suppose 1000 dogs entered the competition in all. How many dogs would be in the toy group?

a. 72 b. 120

c. 144 d. 210

ANSWER: B We could figure the number of dogs in any group by multiplying the fraction of dogs in any group by 1000. In the toy group, for example, there were 0.12 × 1000 = 120 dogs in the competition.

NOTES: REF:

5. A double bar graph is similar to a regular bar graph, but gives 2 pieces of information for each item on the vertical axis, rather than just 1. The bar chart below shows the weight in kilograms of some fruit sold on two different days by a local market. This lets us compare the sales of each fruit over a 2 day period, not just the sales of one

Police Math Test

Page 5 of 30

fruit compared to another. We can see that the sales of star fruit and apples stayed most nearly the same. The same amount of apples and oranges was sold on the second day.

How much did the sale of oranges increase from day 1 to day 2?

a. 10 kilograms b. 30 kilograms

c. 20 kilograms d. 15 kilograms

ANSWER: A

NOTES: REF:

The sale of oranges increased from day 1 to day 2 by 10 kilograms.

6. Refer to the below graph to answer the following questions:

Police Math Test

Page 6 of 30

A bar graph is fairly easy to read. However, before we can learn anything useful, we have to know what the graph is about (in this case "Sales") and the parameters or limits of the graph (in this case "Sales by Quarters of the year" and "Thousands of Dollars"). Simply reading the numbers at the top of each bar isn't enough. We have to give them some "flesh and blood". Putting all of our data together, we can make a meaningful statement such as: "Quarter 3 was our best quarter with sales of $4,075,000.00." We can mine other facts from this graph: What was the second best quarter?

a. 1 b. 2

c. 3 d. 4

ANSWER: B The second best quarter was quarter 2. Note that the question did not ask you for the amount. Always reread a question after you have completed it to make sure you actually answered the question that was asked.

Police Math Test

Page 7 of 30

NOTES: REF: 7. What was the lowest quarterly sales?

a. $2,800 b. $44,000

c. $2,800,000 d. $3,800,000

ANSWER: C The lowest quarterly sales were $2,800,000. Not 2,800. Remember, to make charts easier to read(and fit reasonably on a page), the chart is often scaled. In this case, each unit plotted represents $1,000 and each label, grid or division represents multiples of $500,000. Always make sure you have taken any scaling into account and that you have expressed the correct units (in this case, dollars).

NOTES: REF:

8. In what quarter did we have the worst sales?

a. 1 b. 2

c. 3 d. 4

ANSWER: D Quarter 4. In this chart, the shortest bar represents the least sales.

NOTES: REF:

To answer this question we must first add up all of the quarterly sales. Sometimes, it may be enough to estimate the total. In this case, however, the amounts are so close that we will have to add them up to be sure. 3050 + 4060 + 4075 + 2800 = 13,985. Whoops, not quite! No, we did not reach our goal. Notice that this is a two-step problem. Most real-world problems require two or more steps to reach a solution.

Police Math Test

Page 8 of 30

Refer to below chart:

9. What two expenditures, added together, equal what is spent on rent?

a. clothes & transportation b. other & transportation

c. food & transportation d. clothes & food

ANSWER: D

Police Math Test

Page 7 of 30

NOTES: REF: 7. What was the lowest quarterly sales?

a. $2,800 b. $44,000

c. $2,800,000 d. $3,800,000

ANSWER: C The lowest quarterly sales were $2,800,000. Not 2,800. Remember, to make charts easier to read(and fit reasonably on a page), the chart is often scaled. In this case, each unit plotted represents $1,000 and each label, grid or division represents multiples of $500,000. Always make sure you have taken any scaling into account and that you have expressed the correct units (in this case, dollars).

NOTES: REF:

8. In what quarter did we have the worst sales?

a. 1 b. 2

c. 3 d. 4

ANSWER: D Quarter 4. In this chart, the shortest bar represents the least sales.

NOTES: REF:

To answer this question we must first add up all of the quarterly sales. Sometimes, it may be enough to estimate the total. In this case, however, the amounts are so close that we will have to add them up to be sure. 3050 + 4060 + 4075 + 2800 = 13,985. Whoops, not quite! No, we did not reach our goal. Notice that this is a two-step problem. Most real-world problems require two or more steps to reach a solution.

Police Math Test

Page 10 of 30

NOTES: REF:

$7,500 + $22,500 is equal to $30,000. To determine what the Canadian percentage is, you must divide the Canadian amount by the total: $7,500 divided by $30,000 is equal to .25. To put it into a percentage format, you must multiply this value by 100: .25 x 100 is equal to 25%.

11. What percentage of all the seized American funds were seized in 1992?

a. 29% b. 78%

c. 80% d. 99%

ANSWER: C The question is focusing on the American funds, such that you can ignore all the Canadian funds. To compute a percentage, you need to know what the total value of American funds that were seized. Therefore, you must add up all the American funds seized over the years: $72,500 + $850,000 + $63,427 + $22,500 + $54,073 is equal to $1,062,500. To determine what the 1992 percentage is, you must divide the 1992 amount by the total: $850,000 divided by $1,062,500 is equal to .80. To put it into a percentage format, you must multiply this value by 100: .80 x 100 is equal to 80%.

NOTES: REF:

12. Look at this series: 44, 44, 50, 50, 56, . . . What number should come next?

a. 44 b. 48

c. 56 d. 62

ANSWER: C This is an alternation with repetition series, in which each number repeats itself, then increases by 6.

NOTES: REF:

Police Math Test

Page 11 of 30

13. Which of the following multiplications is right?

a. 4 ´ 27 = 108 b. 4 ´ 34 = 146 c. 7 ´ 22 = 144

d. 8 ´ 17 = 136 e. 5 ´ 27 = 145

ANSWER: A

NOTES: REF:

27 times 4 is 108. 34 times 4 is 136. 22 times 7 is 154. 27 times 5 is 135.

14. How much is 150% of 80?

a. 1.8 b. 5.3

c. 70.0 d. 120.0

ANSWER: D SOLUTION: To solve this question you must multiply 80 by 1.5 (which is 120 or choice D).

NOTES: REF:

15. Of the 300 people working at a medical facility, 14% are physicians' assistants. How many workers at the medical facility are not physicians' assistants?

a. 42 b. 86

c. 258 d. 286

ANSWER: C SOLUTION: To solve this question you must determine that 86% (100%

Police Math Test

Page 12 of 30

NOTES: REF:

minus 14%) of the workers are not physicians assistants, and then find out how many workers that would be. (300x86% = 258).

16. A pump discharges water at a rate of 184 gallons per minute. Which one of the following is the number of gallons of water this pump will discharge in one hour?

a. 1,104 gallons b. 1,840 gallons

c. 2,208 gallons d. 11,040 gallons

ANSWER: D SOLUTION: To solve this question you must know that there are 60 minutes in an hour. So you multiply 184 gallons (the amount discharged in one minute) by 60 (the number of minutes in an hour). The answer is 11,040 gallons (D).

NOTES: REF:

17. Write .2% as a decimal

a. .2 b. .02

c. .002 d. .2

ANSWER: C Step 1: Move the decimal point two places to the right, and place % at the end of the number.

Example:

Police Math Test

Page 13 of 30

Step 2: Write the percentage accordingly.

Example:

NOTES: REF: 18. Write 3.4% as a fraction.

a. 34/1000 b. 34/10

c. 34/100 d. 340/100

ANSWER: A PERCENT EQUIVALENTS. "Percent" means "out of 100." If you understand this concept, it becomes very easy to change a percent to an equivalent decimal or fraction.

5% = 5/100 = .05

NOTES: REF:

2.6% = 2.6/100 = .026

19. Write 3/4% as a decimal.

a. .75 b. .075 c. .0075

d. .00075 e. 75

Police Math Test

Page 14 of 30

ANSWER: C Percentages can be converted into decimals in two steps:

Step 1: Move the decimal point two places to the left, and eliminate % from the end of the number.

Example:

Step 2: Write the decimal accordingly.

Example:

NOTES: REF:

Police Math Test

Page 15 of 30

20. Find 60% of 70.

a. 420 b. 4.2 c. 1162

d. 4200 e. 42

ANSWER: E

NOTES: REF:

60 % = 60/100 of 70 = 42

21. Reduce 20/60 to it’s lowest common denominator:

a. 2/3 b. 1/3

c. 2/6 d. 1/4

ANSWER: B To reduce a fraction to lowest terms, divide the numerator and denominator by their Greatest Common Factor (GCF). This is also called simplifying the fraction. Here's how to reduce 20/60 to the LCD:

NOTES: REF: 22. What does 1/5 +1/3 =

a. 3/4 b. 7/8

c. 5/7 d. 8/15

Police Math Test

Page 16 of 30

ANSWER: D

Before you can add or subtract fractions with different denominators, you must first find equivalent fractions with the same denominator, like this: Find the smallest multiple (LCM) of both numbers. Rewrite the fractions as equivalent fractions with the LCM as the denominator.
When working with fractions, the LCM is called the least common denominator (LCD).

NOTES: REF: 23. What does 1/5 x 2/3 =

a. 2/3 b. 5/8

c. 2/15 d. 2/11

ANSWER: C To multiply fractions:

1. 2. 3.

Simplify the fractions if not in lowest terms. Multiply the numerators of the fractions to get the new numerator. Multiply the denominators of the fractions to get the new denominator.
Simplify the resulting fraction if possible.

NOTES: REF:

Police Math Test

Page 17 of 30

24. What does 3 divided by 1/4 =

a. 3 b. 12

c. 21 d. 24

ANSWER: B To divide any number by a fraction:

First step: Find the reciprocal of the fraction.
Second step: Multiply the number by the reciprocal of the fraction. Third step: Simplify the resulting fraction if possible.

Fourth step: Check your answer: Multiply the result you got by the divisor and be sure it equals the original dividend. Note that you can only divide by non-zero fractions.

NOTES: REF: 25. If the average of five numbers is -10, and the sum of three of the numbers is 16, then what is the average of the other two numbers?

a. -33 b. -1 c. 5

d. 20 e. 25

ANSWER: A Let the five numbers be a, b, c, d, e. Then their average is (a + b + c + d + e)/5 = -10. Now three of the numbers have a sum of 16, say, a + b + c = 16. So substitute 16 for a + b + c in the average above: (16 + d + e)/5 = -10. Solving this equation for d + e gives d + e = -66. Finally, dividing by 2 (to form the

Police Math Test

Page 18 of 30

NOTES: REF:

average) gives (d + e)/2 = -33. Hence, the answer is (A).

26. In traveling from city A to city B, John drove for 1 hour at 50 mph and for 3 hours at 60 mph. What was his average speed for the whole trip?

a. 50 b. 53 1/2 c. 55

d. 56 e. 57 1/2

ANSWER: E The total distance is 1(50) + 3(60) = 230. The total time is 4 hours, hence, Average Speed = Total Distance/Total Time = 230/4 = 57 1/2. The answer is (E). Note: the answer is not the mere average of 50 and 60, rather the average is closer to 60 because he traveled longer at 60 mph (3 hrs) than at 50 mph (1 hr).

NOTES: REF: 27. 2/7 x 4/5 =

a. 3/8 b. 7/29

c. 9/31 d. 8/35

ANSWER: D

NOTES: REF: 28. 5/12 x 9/10 =

a. 13/41 b. 6/7

c. 3/8 d. 11/45

Police Math Test

Page 19 of 30

ANSWER: C

NOTES: REF: 29. 2 5/8 x 6 =

a. 9 2/3 b. 15 3/4

c. 12 1/2 d. 24

ANSWER: B

NOTES: REF: 30. 5 1/3 x 6 3/8 =

a. 30 1/3 b. 33

c. 29 1/5 d. 34

ANSWER: D

NOTES: REF: 31. Number-series problem: 9, 10, 12, 15, 19, ___

Police Math Test

Page 20 of 30

a. 18 b. 21

c. 13 d. 24

ANSWER: D

Answer: 24
The relationship: continue to add one plus the previously added number to the progression. Here's a closer look: Calculate the difference between the adjacent numbers to find the pattern: These numbers are increasing at a rate of 1, 2, 3, 4. Therefore, the next increasing number should be 5 making the correct answer 24. The completed series is: 9, 10, 12, 15, 19, 24 -- or

NOTES: REF:

9 + 1 = 10 + (1 + 1) = 12 + (2 + 1) = 15 + (3 + 1) = 19 + (4 + 1) = 24

32. 43, 39, 35, 31, _____

a. 37 b. 17

c. 27 d. 45

ANSWER: C The relationship in this number series is -4. The correct answer is 27. Most number-series questions are more difficult than this one, but this gives you a general idea of what to expect.

NOTES: REF:

33. 189, 93, 45, 21, 9, ___

a. 3 b. 18

c. 5 d. 23

Police Math Test

Page 21 of 30

ANSWER: A The answer is 3. The relationship actually begins with the missing number:

NOTES: REF:

The completed series is: 189, 93, 45, 21, 9, 3 -- or (93 X 2 + 3 = 189), (45 x 2 + 3 = 93), (21x 2+ 3 = 45), (9 x 2 + 3 = 21), (3 X 2 + 3 =9), (0 x 2 + 3 = 3)

34. 1, 4, 8, 11, 22, 25, 50, ___, ___

a. 51,103 b. 53,106

c. 55, 117 d. 63, 127

ANSWER: B The answer is: 53,106. The series is created starting with the number "1" and alternately adding "3" and then multiplying by "2" to each succeeding number.

NOTES: REF:

The completed series is: 1, 4, 8, 11, 22, 25, 50, 53, 106 -- or (1 + 3 = 4), (4 x 2 = 8), (8 + 3 = 11), 11 x 2 = 22), (22 + 3 = 25), (25 x 2 = 50), (50 + 3 = 53), (53 x 2 = 106)

35. How many faces does this box have?

a. 2 b. 4 c. 6

Police Math Test

Page 22 of 30

ANSWER: C NOTES: Knowledge and Skills REF: 36. Where should the number 347 be placed in this row so that the numbers will be in order from the smallest to the largest?

a.

b.

c.

ANSWER: A NOTES: Knowledge and Skills REF: 37. Cincinnati had a population of three hundred sixty-four thousand, one hundred fourteen people in 1990. What would be the population of Cincinnati if the city grew by ten thousand?

a. 365,114 b. 374,114 c. 464,114

ANSWER: B NOTES:

Police Math Test

Page 23 of 30

REF:

Conceptual Understanding

38. The Ohio State Fair is held every year in Columbus. At one of the state fairs, there were 48 Girl Scouts marching in the parade. There were 6 girls in each row. Which equation would you use to find how many rows of Girl Scouts were marching in the parade?

a. 48 + 6 = n b. n × 6 = 48 c. 48 − n = 6

ANSWER: B NOTES: Applications & Problem Solving REF: 39. Round the numbers 42 and 31 to the nearest ten. Then subtract the rounded numbers to estimate the difference. Which statement is true?

a. Your estimate is larger than the exact difference. b. Your estimate is smaller than the exact difference. c. Your estimate is the same as the exact difference.

ANSWER: B NOTES: Conceptual Understanding REF: 40. Jenny, Brian, and Omar were in three sports events. The chart shows how many minutes it took the children to complete each event.

The winner is the person with the lowest total time in all the events. Which child won?

Swimming Bicycling Running

Jenny 4.3 minutes 6.8 minutes 5.1 minutes

Brian 5.1 minutes 7.2 minutes 4.9 minutes

Omar 3.9 minutes 7.5 minutes 5.0 minutes

Police Math Test

Page 24 of 30

a. Jenny b. Brian c. Omar

ANSWER: A NOTES: Applications & Problem Solving REF: 41. Approximately 11 million people live in Ohio. About how many people live in cities? Use the graph to answer the question.

a. 8 million b. 3 million c. 1 million

ANSWER: A NOTES: Applications & Problem Solving REF:

Police Math Test

Page 25 of 30

42. Carmen made a wood sculpture from small pieces of wood. The sculpture is 2 feet 10 inches tall. Carmen places her sculpture on a base that is 6 inches tall. How tall are the sculpture and the base together?

a. 2 feet 4 inches b. 3 feet 4 inches

c. 3 feet 6 inches d. 8 feet 10 inches

ANSWER: B NOTES: Applications & Problem Solving REF: 43. What is 30% of $48.00?

a. $14.40 b. $16.00

c. $18.00 d. $33.60

ANSWER: A NOTES: Knowledge and Skills REF: 44. Which of the following pairs of lines appears to be perpendicular? a. c.

b.

d.

ANSWER: A NOTES: Knowledge and Skills REF:

Police Math Test

Page 26 of 30

45. There are 4 boys to every 6 girls in Mr. Simpson’s class. If there are 30 students in the class, how many of those students are girls?

a. 10 b. 12

c. 18 d. 20

ANSWER: C NOTES: Applications & Problem Solving REF: 46. In the picture below of part of a ruler, what does each small division represent?

a. b.

c.

inch inch

d.

inch inch

ANSWER: B NOTES: Conceptual Understanding REF: 47. Neil’s club held a fund-raising drive. During the first week, the club received the donations shown below from five members.

$8.00 $10.00 $10.00 $22.00

Police Math Test

Page 27 of 30

$25.00

What was the average amount donated by the members during the first week?

a. $15.00 b. $17.00

c. $25.00 d. $75.00

ANSWER: A NOTES: Knowledge and Skills REF: 48. During their last vacation, a family took a train trip. They traveled 594 miles in 9 hours. What was the train’s average speed?

a. 56 mph b. 60 mph

c. 66 mph d. 585 mph

ANSWER: C NOTES: Applications & Problem Solving REF: 49. Ernest paid for a book with a $20 bill and received one dollar, one quarter, two dimes, and four pennies in change. How much did he pay for the book?

a. $18.49 b. $18.51

c. $19.49 d. $19.51

ANSWER: B NOTES: Applications & Problem Solving REF: 50. Coach Newman has been keeping a record of the temperature in his classroom in the chart below.

Police Math Test

Page 28 of 30

How many hours did the temperature rise only one degree?

a. none b. 1

c. 2 d. 3

ANSWER: C NOTES: Conceptual Understanding REF: 51. If one bench can seat 8 people, what is the fewest number of benches needed to seat 42 people?

a. 4 b. 5

c. 6 d. 7

ANSWER: C NOTES: Applications & Problem Solving REF:

Police Math Test

Page 29 of 30

52. A container holds a. 3 quarts. gallons when full. This amount can also be written as c.

b. 12 pints.

quarts. d. 15 pints.

ANSWER: B NOTES: Knowledge and Skills REF: 53. Mike and Treecie are planning a party. If they plan to make 30 hamburgers for 25 people, what is the least number of hamburgers they will need if 40 people show up?

a. 30 b. 48

c. 55 d. 65

ANSWER: B NOTES: Applications & Problem Solving REF: 54. Which is the correct order to show capacity from smallest to largest?

a. pint, cup, quart, gallon b. cup, pint, gallon, quart

c. pint, cup, gallon, quart d. cup, pint, quart, gallon

ANSWER: D NOTES: Conceptual Understanding REF: 55. Which of the following is a correct statement?

a. 0.02 > 20% b. 0.02 = 20%

c. 20% < 0.02 d. 20% > 0.02

Police Math Test

Page 30 of 30

ANSWER: D NOTES: Conceptual Understanding REF:

Police Grammar

Page 1 of 13

Police Grammar/ Spelling Test Answer Key
Multiple Choice Identify the letter of the choice that best completes the statement or answers the question.

1. Choose the word or phrase that has the same meaning as the underlined word. The witness corroborated the suspect's story. a. verified c. added to b. contradicted d. questioned

ANSWER: A verify means to corroborate, confirm, or attest to contradict means to express the opposite added to means to augment question means to cast doubt on NOTES: REF:

2. Choose the word or phrase that has the same meaning as the underlined word. It was not a very pragmatic plan.

a. plausible b. serious

c. practical d. sensible

ANSWER: C plausible means apparently valid or likely serious means earnest or sincere practical means pragmatic sensible means reasonable or wise NOTES: REF:

3. Choose the word or phrase that has the same meaning as the underlined word. The police sequestered the suspect.

a. caught b. isolated

c. arrested d. released

ANSWER: B caught means to seize or trap isolate means to sequester

Police Grammar

Page 2 of 13

NOTES: REF:

arrest means to seize or bring into custody release means to let go

4. Choose the word or phrase that has the same meaning as the underlined word. In order to conceal her guilt, Linda told a blatant lie. a. harmless c. careless b. subtle d. obvious

ANSWER: D harmless means not harmful, inoffensive subtle means not obvious careless means done without care or concern obvious means blatant NOTES: REF:

5. Choose the word or phrase that has the same meaning as the underlined word. He was ignorant of the proper procedures.

a. unaware b. conscious

c. uncertain d. cognizant

ANSWER: A unaware means ignorant conscious means aware uncertain means having questions regarding cognizant means aware NOTES: REF:

6. Choose the sentence that represents the best English Usage. a. We seldom ever receive this type c. Each of these regulations apply to of request anymore. your case. b. Neither of the employees are doing d. I have enclosed a copy of the file what is expected of him. you requested. ANSWER: D The sentence with the best English Usage is Number D. The correct way to write Number A is "We seldom receive this type of request." The correct way to write Number B is "Neither of the employees is doing what is expected of him." The correct way to write Number C is "Each of these regulations applies to your case." NOTES: REF:

7. Choose the sentence that represents the best English Usage. a. Of the two runners, John is the c. John is the worst of the two

Police Grammar

Page 3 of 13

worst. b. Of the two runners, John is the better.

runners. d. John is the best of the two runners.

ANSWER: B The sentence with the best English Usage is number B. The correct way to write Number A is "Of the two runners, John is worse." The correct way to write Number C is "John is the worse of the two runners." The correct way to write Number D is "John is the better of the two runners." NOTES: REF:

8. Which of the following words can be defined as: The process of deriving general principles from particular instances?

a. concoction b. deduction

c. induction d. reduction

ANSWER: C The bolded text in the question is one of the definitions of induction provided by ITP Nelson Canadian Dictionary of the English Language: An Encyclopedic reference (1997). Webster’s Ninth New Collegiate Dictionary (1984) defines induction as, “inference of a generalized conclusion from particular instances,” paralleling the definition provided in ITP Nelson Canadian Dictionary of the English Language: An Encyclopedic reference (1997). NOTES: REF:

9. Which one, if any, of the following sentences contains a misspelled word? a. The discussion of budgetary issues c. They are unable to give us a took up most of the time. definate answer. b. This is not a frequently occurring d. None contains a misspelled word. situation. ANSWER: C SOLUTION: (A) Budgetary is sometimes misspelled as budgetery (this type of ending is sometimes ary and sometimes ery). Here it is spelled correctly. There is no misspelled word in this choice. (B) The final consonant in words such as occur must be doubled before ing and other suffixes are added. Not doubling the final consonant in such cases is a mistake. The final consonant is doubled in occurring. There is no misspelled word in this choice. (C) Definate is misspelled. The correct spelling is definite. This choice answers the question, Which one of the following sentences contains a misspelled word? This choice contains a misspelled word and therefore is the correct answer.

(D) One of the three previous sentences contains a misspelled word (Choice C).

Police Grammar

Page 4 of 13

NOTES: REF:

Choice D is therefore not the correct answer.

10. Which one of the following is grammatically INCORRECT? a. Only one of the software packages c. The number of applications has suits our needs. increased this year. b. Are their information correct? d. Enclosed are copies for every committee member. ANSWER: B SOLUTION: (A) This sentence is grammatically CORRECT as presented. The subject of this sentence is "one" and the verb used is "suits." It is grammatically correct to say, "Only one suits our needs." (A common error is to think that "packages" is the subject of this sentence, so "suit" should be the verb.) Since choice A is grammatically CORRECT as presented, it is not the answer to the sample question. (B) This sentence is grammatically INCORRECT as presented. The subject of this sentence is "information" and the verb used is "are." It is grammatically incorrect to say "information are." It is correct to say "information is." (A common error is to think that the use of "their" in this sentence means that "they" is the subject of this sentence, so "are" should be the verb.) Since choice B is grammatically INCORRECT as presented, it IS the answer to the sample question.

(C) This sentence is grammatically CORRECT as presented. The subject of this sentence is "number" and the verb used is "has increased" It is grammatically correct to say "The number has increased." (A common error is to think that "applications" is the subject of this sentence, so "have increased" should b e the verb.) Since choice C is grammatically CORRECT as presented, it is not the answer to the sample question.
(D) This sentence is grammatically CORRECT as presented. The subject of the sentence is "copies" and the verb used is "are." It is grammatically correct to say, "copies are." (A common error is to think that the verb should be "is" simply because "Enclosed is" is the more usual phrase since only one enclosure is the more usual circumstance.) Since choice D is grammatically CORRECT as presented, it is not the answer to the sample question.

NOTES: REF:

11. Choose the correctly spelled word::

a. confidance b. confidence ANSWER: B NOTES:

c. confedense d. none of these

Police Grammar

Page 5 of 13

REF: 12. Choose the correctly spelled word::

a. disappoint b. dissapoint ANSWER: A NOTES: REF: 13. Choose the correctly spelled word::

c. diseppoint d. none of these

a. defendent b. deffendent
ANSWER: D defendant NOTES: REF:

c. diffendant d. none of these

14. Choose the correctly spelled word::

a. amung b. among ANSWER: B NOTES: REF: 15. Choose the correctly spelled word::

c. ammung d. none of these

a. occassionnally b. occassionally ANSWER: C NOTES: REF: 16. Choose the correctly spelled word::

c. occasionally d. none of these

a. calendar b. calender ANSWER: A NOTES: REF: 17. Choose the correctly spelled word::

c. calander d. none of these

a. merchendice

c. merchendise

Police Grammar

Page 6 of 13

b. merchandice
ANSWER: D merchandise NOTES: REF:

d. none of these

18. Choose the correctly spelled word::

a. competant b. compitent
ANSWER: D competent NOTES: REF:

c. compettent d. none of these

19. Choose the correctly spelled word::

a. apparant b. apperant
ANSWER: D apparent NOTES: REF:

c. apparrant d. none of these

20. Choose the correctly spelled word::

a. insistant b. incistant ANSWER: C NOTES: REF: 21. Choose the correctly spelled word::

c. insistent d. none of these

a. reference b. refferance ANSWER: A NOTES: REF: 22. Choose the correctly spelled word::

c. referance d. none of these

a. separete b. separate ANSWER: B

c. seperate d. none of these

Police Grammar

Page 7 of 13

NOTES: REF: 23. Choose the correctly spelled word::

a. truely b. truley ANSWER: C NOTES: REF: 24. Choose the correctly spelled word::

c. truly d. none of these

a. begining b. beginning ANSWER: B NOTES: REF: 25. Choose the correctly spelled word::

c. beggining d. none of these

a. preffered b. preferred ANSWER: B NOTES: REF: 26. Choose the correctly spelled word::

c. prefferd d. none of these

a. memorandem b. memorandom ANSWER: C NOTES: REF: 27. Choose the correctly spelled word::

c. memorandum d. none of these

a. posibility b. possability
ANSWER: D possibility NOTES: REF:

c. possebility d. none of these

28. Choose the correctly spelled word::

Police Grammar

Page 8 of 13

a. probibility b. probebility ANSWER: C NOTES: REF: 29. Choose the correctly spelled word::

c. probability d. none of these

a. prominent b. prominant ANSWER: A NOTES: REF: 30. Choose the correctly spelled word::

c. promenant d. none of these

a. judgemant b. judgment ANSWER: B NOTES: REF: 31. Choose the correct word or phrase. I didn't expect John to be out." "He _______________ shopping.

c. judgemint d. none of these

a. must be gone b. should go ANSWER: D NOTES: REF: 32. Why isn't she at home?" "She _______________ the bus.

c. ought to have gone d. must have gone

a. could miss b. could have missed ANSWER: B NOTES: REF:

c. might miss d. can have missed

33. To travel in the European Community you _______________ a passport.

a. haven't got b. don't need

c. have need d. mustn't have

Police Grammar

Page 9 of 13

ANSWER: B NOTES: REF: 34. When we arrived, the play _______________ .

a. has been started b. had been started ANSWER: C NOTES: REF:

c. had started d. would have started

35. If you _______________ wet in the rain, you wouldn't have caught a cold.

a. hadn't got b. wouldn't have got ANSWER: A NOTES: REF: 36. I asked her if _______________.

c. haven't got d. wouldn't get

a. liked she the candy b. did she like the candy ANSWER: D NOTES: REF:

c. likes she the candy d. she liked the candy

37. I'll wash the dishes, if you _______________ the living room.

a. will clean b. clean ANSWER: B NOTES: REF:

c. have cleaned d. would clean

38. What does your suitcase _______________?" "It's red and has a black handle.

a. like? b. look like? ANSWER: B NOTES: REF:
39. Who _______________ the room?

c. seem to be? d. similar to?

Police Grammar

Page 10 of 13

They only have done two walls and they left the paint open.

a. paints b. will paint ANSWER: D NOTES: REF:

c. painted d. has been painting

40. Please _______________ my mother when we get back home.

a. tell to me to call b. remind to me to call ANSWER: D NOTES: REF:

c. remember me to call d. remind me to call

41. Were you able to get the discount? " "No, but I wish I _______________."

a. might b. will ANSWER: C NOTES: REF:

c. could have d. may have

42. I'm always too busy. If I _______________ the time, I would attend the conference.

a. had had b. have ANSWER: D NOTES: REF:

c. had been d. had

43. What happened to your computer?" "It's _______________ for at least a week now.

a. broke b. broken ANSWER: C NOTES: REF:

c. been broken d. broked

44. I think it is time for you to _______________ your own living.

a. win b. maintain

c. gain d. earn

Police Grammar

Page 11 of 13

ANSWER: D NOTES: REF: 45. Which one of the following words is misspelled?

a. manageable b. circumstants

c. legality d. None of the above is misspelled

ANSWER: B SOLUTION: To answer this question, you must examine each of the words. The words “manageable” (choice A) and “legality” (choice C) are spelled correctly. The word “circumstants” (choice B) is misspelled. The correct spelling is “circumstance”. NOTES: REF:

46. Of the following, which one should be THIRD in an alphabetic file?

a. Docker, John b. Decker, Jane

c. Dinckman, June d. Dockman, James

ANSWER: A SOLUTION: To answer this question, you must first put the names into alphabetical order. “Decker, Jane” would be first. “Dinckman, June” would be second. “Docker, John” would be third followed by “Dockman, James”. The question asks for the third name on the list. The answer is “Docker, John,” (choice A). NOTES: REF:

47. Paragraph Organization The question below is based upon the following group of scrambled sentences. You are to read the sentences and then indicate which order of presentation would represent their best arrangement. 1. Eventually, they piece all of this information together, and make a choice. 2. Before actually deciding upon a corrections job, people usually think about several possibilities.

3. They imagine themselves in different situations, and in so doing, they probably think about their interests, goals, and abilities.
4. Choosing among occupations in the correctional services field is an important decision to make. QUESTION: Which one of the following is the best arrangement of these sentences?

Police Grammar

Page 12 of 13

a. 2-3-1-4 b. 2-3-4-1

c. 4-2-1-3 d. 4-2-3-1

ANSWER: D SOLUTION: The best arrangement of the sentences is 4-2-3-1. Sentence 4 introduces the main idea of the paragraph: “choosing a correctional services occupation.” Sentences 2-3-1 then follow up on this idea by describing, in order, the steps involved in making such a choice. Choice D is the BEST ANSWER to the question. NOTES: REF:

48. Correction Officer Gilson failed to take proper precautions. His failure to take proper precautions caused a personal injury accident. QUESTION: Which one of the following best presents the information above? a. Correction Officer Gilson failed to c. Correction Officer Gilson's failure take proper precautions that caused to take proper precautions caused a personal injury accident. a personal injury accident. b. Proper precautions, which d. Correction Officer Gilson, who Correction Officer Gilson failed to failed to take proper precautions, take, caused a personal injury was in a personal injury accident. accident. ANSWER: C SOLUTION: Choices A and B both convey the idea that the proper precautions caused the accident. Choice D states that Gilson was in a personal injury accident; the information presented states that Gilson caused a personal injury accident, but we don’t know if Gilson was in the accident. Choice C is the only choice that accurately restates the information presented. NOTES: REF:

49. Information Presentation: Martin Wilson failed to take proper precautions. His failure to take proper precautions caused a personal injury accident. QUESTION: Which one of the following best presents the information above? a. Martin Wilson failed to take c. Martin Wilson’s failure to take proper precautions that caused a proper precautions caused a personal injury accident. personal injury accident. b. Proper precautions, which Martin d. Martin Wilson, who failed to take Wilson failed to take, caused a proper precautions, was in a personal injury personal injury accident. accident.

Police Grammar

Page 13 of 13

ANSWER: C SOLUTION: Choices A and B both convey the idea that the proper precautions caused the accident. Choice D states that Wilson was in a personal injury accident; the information presented states that Wilson caused a personal injury accident, but we don’t know if Wilson was in the accident. Choice C is the ONLY choice that accurately restates the information presented NOTES: REF:

50. Select the space where the underlined name or number should be placed to be in correct order.

a. This location b. This location c. This location ANSWER: E NOTES: REF:

d. This location e. This location

Reading Comprehension Test

Page 1 of 18

Reading Comprehension Test Answer Key

Multiple Choice

Identify the letter of the choice that best completes the statement or answers the question.
Reading Comprehension

Based on the information provided for you in the passage below, you are to answer the question that follows. The answer is taken directly from the passage. Thus, if you have trouble determining the correct answer, look back into the paragraph for clues.

Passage:

Although the detectives do not have a suspect, evidence from the crime scene suggests several possibilities. The direct route of the burglar indicates that the perpetrator most likely knew the inside of the home fairly well. He passed up the camera and VCR in the living room and the Gucci tennis bracelet and silver jewelry lying on the dresser in the bedroom. No steps were wasted. The fact that the burglar was able to deactivate the elaborate alarm system suggests that the burglar may be a career criminal. The fact that the burglar took only the real emeralds and left the “paste” seems to indicate that he or she is a gemstone expert. The Franklin Insurance Company (FIC) insured the emeralds for $100,000. They were taken from the safe that is hidden behind a picture in the bathroom.

1.

Which of the following factors indicate(s) that the burglar knew the inside of the home fairly well?

a. The fact that the burglar was able to d. The fact that the detectives do not

Reading Comprehension Test

Page 2 of 18

deactivate the elaborate alarm yet have a suspect. system. b. The fact that the burglar took a e. Both a and c. direct route to the emeralds. c. The fact that the burglar took only the real emeralds.

ANSWER: B

The passage suggests the burglar knew exactly where to go to obtain the emeralds and therefor knew the inside of the home.

“The direct route of the burglar indicates that the perpetrator most likely knew the inside of the home fairly well. He passed up the camera and VCR in the living room and the Gucci tennis bracelet and silver jewelry lying on the dresser in the bedroom. No steps were wasted.”

NOTES: REF:

It can be deduced the direct route indicated in the passage was to the emeralds.

Passage:

Too many consumers believe in the quick fix, the easy solution, the magic ingredient, the miracle cure. And, of course, there are corporations ready to provide them with the product which will solve their health care problems. An independent review board is needed to interpret the nutritional data available. If such a review procedure is not adopted, then our society is left to the whims of the private sector whose vested interests and public pronouncements on very specific products already seriously compromises public understanding of these health issues. The nutrition information Americans are getting may be fragmentary, contradictory and confusing, when it’s not outright incorrect, according to nutritional biochemist T. Cohn Campbell. Campbell, who has been doing nutrition research for the past 25 years, is one of the co-authors of the National Academy of Sciences’ report on “Diet, Nutrition and Cancer,” which recommended increased consumption of fruits, vegetables and whole-grain cereals. He says that confusion about nutritional guidelines and chronic disease risk is directly attributable to the use of scientific data which are much too narrowly focused for policy and marketplace purposes. An extensive nutritional study conducted in China recently provides some examples of the inconsistencies that Campbell is concerned about. Some U.S. studies associate low cholesterol levels in the blood with increased risk of colon cancer. In China, however, incidences of colon cancer were low among those who had

Reading Comprehension Test

Page 3 of 18

low cholesterol levels, a finding much more consistent with the recognized beneficial effects of low cholesterol levels. Such an observation provides further evidence that the U.S. data showing an increase in colon cancer at lower cholesterol levels is an experimental fluke.

Another example, dieting is the way to lose weight in the States. We count calories, but obviously have limited success with this method since so many of us are constantly dieting and continually overweight. Chinese people consume about 20 percent more calories than Americans, yet there is very little obesity amongst the Chinese, and they are healthier in general than Americans. This finding tells us that excess caloric intake does not necessarily cause excess body weight nor is it likely to be the determinant of chronic disease risk. One difference between the diets is the proportion of fat in calories consumed. Chinese fat consumption ranges from 6 to 25 percent, with an average of about 15 percent of daily calories from fat. This is well below the 30 percent usually recommended in American diets. So, the current wisdom which says that people cannot subsist on diets with fat intakes much lower than 30 percent of calories is seriously called into question by the Chinese study.

2. Which of the statements given below is best supported by the passage above? a. The Chinese suffer from less obesity because they have a more active life-style. c. The existence of an independent review board keeps corporations from peddling

products whose effectiveness is questionable. b. The average person in China d. Confusion about nutritional consumes half the fat of the average guidelines in the U.S. is due to the improper use of American.

scientific research.

ANSWER: D The answer is Choice d. This is supported by a sentence in the middle of the passage which says, “Confusion about nutritional guidelines and chronic disease is directly attributable to the use of scientific data which are much too narrowly focused for policy and marketplace purposes.” The passage then backs this up with a few examples. It calls the U.S. data howing an increase in colon cancer at lower cholesterol levels an experimental fluke.

It also mentions two commonly held but incorrect beliefs: that excess intake of

Reading Comprehension Test

Page 4 of 18

calories necessarily leads to excess body weight and that people need a diet consisting of 30 percent fat. Even though the entire article talks about scientific research and incorrect nutritional beliefs held by Americans, you may not have liked the answer. You may have felt that the confusion could have been caused by other factors. You might have thought that saying the confusion was “due to the improper use of scientific research” was assuming too much. If so, this is what we meant earlier when we talked about being overly analytical and picky. The question stem asks what statement is “best supported by the passage,” and the entire article is about narrow scientific information improperly used for nutritional guidelines. In addition, none of the other choices is possible. Choice a is incorrect because it isn’t stated anywhere in the passage. You may feel that Choice a is correct because you can t figure out why the Chinese aren’t overweight if they eat so many more calories than they need, but the author says nothing about how active they are.

NOTES: REF:

Choice b is incorrect, but tricky. You may have been misled by the sentences near the end of the passage. One sentence says that the average Chinese diet is about 15 percent fat. The next sentence says that a diet consisting of 30 percent fat is recommended for Americans. Since 15 percent is half of 30 percent, it is tempting to choose Choice b. Note, however, that it says recommended. Nowhere does it say that the average American diet consists of 30 percent fat. As we said, it’s tricky. (In fact, the average American is lucky if his/her diet is only 30 percent fat, but that information wasn’t in the passage either.) Choice c is also tricky because it sounds as if the review board is already in existence. It says, “The existence of an independent review board keeps...” The passage on the other hand asks for a review board to save consumers from the claims of companies in the private sector. For Choice c to be in line with the passage, it would have to say something like would keep. We didn’t think up tricky little things like this to be mean. We did it to show you how important it is to read every word, to look at every clue, to think about the meaning of the passages and the choices, and to recheck your answers whenever you have the time.

Passage

"Increasingly, behavior termed ' rage' being viewed as a public health issue, because of road is the number of deaths and injuries related to it. Such behavior is often a reaction to feeling one has been treated unfairly by another driver, and is much less likely to occur if a driver is treated fairly. ' play' the road includes the observance not only of traffic regulations but Fair on also of the rules of courtesy. Courteous driving is based on common sense consideration for other drivers and a strong desire to make the roads safe for everyone. Good highway manners should become just as much a matter of habit as other kinds of manners."

Reading Comprehension Test

Page 5 of 18

3. Which one of the following statements is best supported by the above selection?

a. Being courteous when driving is more important than observing traffic regulations. b. Courteous driving contributes to road safety.

c. Those who are generally polite are also courteous drivers. d. Unlike driving courtesy, the observance of traffic regulations is a matter of habit.

ANSWER: B SOLUTION: Part of what this selection discussed is what it means to be a courteous driver and why it is important to be a courteous driver. The passage says that observing traffic regulations alone is insufficient to promote safety, but being courteous is important as well. This point is made by the use of "not only … but also …" Choice A is incorrect because it says that being courteous is more important. The correct answer, Choice B, is based on the chain of ideas that courteous driving is "fair play," fair play reduces road rage, and reduced road rage increases safety. Choice C may be true, but there is nothing in the passage about courteous driving being connected to general politeness. Therefore, Choice C is incorrect. Choice D contrasts the observance of traffic regulations with driving courtesy, saying that the observance of traffic regulations is a matter of habit, something which is not discussed in the paragraph. Choice D is incorrect.

NOTES: REF:

Passage:

Transporting Inmates:

Correction officers who are transporting inmates are responsible for the safety and security of such inmates from the time they leave the jail until the inmates are delivered to their estination or until the inmates are returned to the jail. The officers should not make any stops during the trip unless it is absolutely necessary. If stopping is absolutely necessary, the officers must take all possible precautions to maintain their own safety and the safety and security of the inmates. If an unavoidable delay is experienced (for example heavy traffic, car trouble, or bad weather) the officers must take the first opportunity to notify the jail of the delay. If a situation occurs which is not covered by specific instructions, the officers must contact their jail' watch s commander if it is possible. If it is not possible to contact the watch commander, the officers must exercise their best judgement to fulfill their responsibility to maintain the safety and

Reading Comprehension Test

Page 6 of 18

security of the inmates."

4. Which one of the following statements concerning the transportation of inmates is best supported by the passage above?

a. If it appears that the trip might be delayed, the officers transporting the inmates should notify the jail immediately.

c. The officers transporting the inmates are responsible for maintaining the safety and security of the inmates at all times during the trip. b. If it is necessary to make a stop, the d. If the vehicle transporting the inmates should be secured with leg- officers and the inmates becomes irons and handcuffs. involved in a traffic accident, the first thing the officers should do is notify their watch commander.

ANSWER: C SOLUTION: To answer this question, evaluate all the choices.

Choice A: The passage states that the jail should be notified if a delay is experienced, but it does not state that the jail should be notified if a delay might occur. In a situation where it appears that a delay might occur but it does not ever take place, it would not be necessary to notify the jail since a delay was not experienced. This choice is incorrect.

Choice B: The passage states that if stopping is absolutely necessary, the officers must take all possible precautions to maintain their own safety and the safety and security of the inmates. It does not state that inmates must be secured with legirons and handcuffs. The appropriate method will vary depending on the nature of the inmates, the purpose of the trip, and the circumstances of the stop. This choice is incorrect.

Choice C: This choice is another way of stating the information provided in the first sentence: "Correction officers who are transporting inmates are responsible for the safety and security of such inmates from the time they leave the jail until the inmates are delivered to their destination or until the inmates are returned to the jail." The first sentence describes the responsibility of the officers as "the

Reading Comprehension Test

Page 7 of 18

safety and security of the inmates" and the duration of the responsibility as the complete trip – "from the time they leave the jail until the inmates are delivered to their destination or until the inmates are returned to the jail." This choice is the only correct choice.

NOTES: REF:
Passage

Choice D: There is nothing in the passage that states that the first thing the officers should do is notify the watch commander if the vehicle they are using to transport the inmates becomes involved in a traffic accident. According to the passage, the officers must report delays to the jail at "first opportunity." It does not specify the watch commander. Moreover, the appropriate first response might vary: someone might need immediate first aid, an inmate might need to be secured, or the roadway might need to be cleared to avoid more accidents. This choice is incorrect.

"Oxydizing materials shall include a substance that yields oxygen readily to stimulate the combustion of organic matter. Oxidizing materials shall be stored in cool, dry, ventilated locations and separated from stored organic materials. Oxidizing materials shall be stored separately from flammable liquids, flammable solids, combustible materials, hazardous

chemicals, corrosive liquids and such other noncompatible materials. Bulk oxidizing materials
shall not be stored on or against wooden surfaces."

5. Which one of the following statements is correct according to the above passage?

a. Oxidizing materials should be c. stored away from organic materials. b. Organic matter should be stored on d. wooden piles.

Oxidizing materials should be stored with flammable solids. Corrosive liquids should be stored with hazardous

ANSWER: A

Reading Comprehension Test

Page 8 of 18

SOLUTION: To answer this question, evaluate all the choices. Choice A states that Oxidizing material should be stored away from organic materials. The paragraph says that oxidizing materials shall be separated from stored organic materials. This choice is correct.

Choice B states that organic material should be stored on wooden piles. The paragraph states that bulk oxidizing materials shall not be stored on… wooden surfaces. The paragraph does not discuss where organic material should be stored. Therefore, this choice is incorrect.

Choice C states that oxidizing materials should be stored with flammable solids. The paragraph states that oxidizing material shall be stored separately from flammable solids. So, this choice is incorrect.

Choice D states that corrosive liquids should be stored with hazardous chemicals. The paragraph states that oxidizing materials shall be stored separately from corrosive liquids but it does not state where corrosive liquids should be stored. Therefore, this choice is incorrect. NOTES: REF:

Reading Comprehension

Passage:

Mass markets are dead. It’s time to let go of the old mass marketing strategies – or so it would seem. Markets are subdividing like crazy, creating a kaleidoscope of goods and services. Every day a new product, or a new variation on an old product, appears. There’s Shredded Wheat, and Mini-Wheats, and Frosted Mini-Wheats. There’s diet Coke and Classic Coke, Cherry Coke and decaffeinated Coke. It’s a global phenomenon. In the urban centers (at the very least) of each nation, countless brands of soaps and soups, jeans and jerseys, soft drinks and softballs can be found, each promising to be better or different. But lest we be too hasty, perhaps we should look at this more closely. Have the markets really been transformed into a myriad of unrelated niches, creating the need for a multitude of ad campaigns? Is it all change and no consistency? Are there no theories or traditions to make sense out of this? In reality, large markets have always subdivided into smaller specialty ones.

Reading Comprehension Test

Page 9 of 18

The main difference is that today they subdivide at a phenomenal rate. Market division is happening faster and faster, and the new segments show up everywhere, almost simultaneously. But who are the customers? In all product categories and places, people increasingly occupy many, and often disparate, segments and opt for many different brands. It’s no longer brand loyalty that holds sway, and you can’t use the old stereotypical guidelines. It doesn’t hold that customers who buy herbal tea prefer cotton t-shirts and brown rice. It’s coffee and a bagel in the morning, sushi for dinner, and herbal tea before bedtime. In this world of rapid transit and instantaneous communication, people everywhere are becoming more alike in their tastes and preferences. And one thing they certainly share is the desire for ever-present, everchanging variety. Companies who continue to have a line of items specifically for clearly defined preference groups have certainly failed to accurately assess the marketplace.

6. Which of the following statements is best supported by the passage?

a. The existence of so many products c. Ad campaigns need to be more prevents successful marketing varied and creative to keep the strategies. attention of the

new consumers. b. It’s not that there are no more mass d. People’s tastes are very ethnic. markets; it’s that the nature of the mass has changed.

ANSWER: B

There is no one statement that tells us Choice b is correct, so we have to build a case to support it.

One clue can be found in the first two sentences:

Mass markets are dead...or so it would seem. Right away the author indicates that she doesn’t really believe that they are dead. Then, in the middle of the passage, she asks, have the markets really been transformed into a myriad of unrelated niches. . In other words, have they been split into a multitude of segments that have nothing in common? Because this is in the form of a question and because she uses the word really, we can infer that she does not believe this idea is true either. Finally, in the last third of the passage she tells us that: People everywhere are becoming more alike in their tastes and preferences. And one thing they have

Reading Comprehension Test

Page 10 of 18

in common is the desire for ever-present, ever-changing variety. Thus, the author is saying that there is still a mass market a mass of consumers world-wide. And, rather than strictly following the old guidelines (health food types, MiddleAmericans, Asians, Central-Europeans, and so on), these consumers have formed themselves into a new kind of mass market which is broad-based but varied and changeable.

NOTES: REF:

We could also use the process of elimination to answer this question because none of the other answers works very well. Choice a is not supported by the passage. The author does say, Its time to let go of the old mass marketing strategies, but she lets us know that she doesn’t really believe this by using the phrase or so it would seem. And she goes on to question whether there are, in fact, no consistencies, no theories, nothing to make sense out of this multiplicity. At one point, she says: Have the markets really been transformed into a myriad of unrelated niches, creating a need for a multitude of ad campaigns? As we stated earlier, the use of the word really and the fact that this statement is put into question form leads us to believe that she doesn’t think so. The author also plays up the similarities within the new marketplace and definitely seems to be leading the reader toward finding ways to work with the new trends. When the entire passage is taken into consideration, we must conclude that the author would feel that successful marketing strategies are possible. Choice c is incorrect because it is neither stated directly nor hinted at in the passage. The author never says how the ad campaigns should be different; she just says that the old ways wont work any more. The products are varied, but she never says that the advertising campaigns have to be varied, also. While ad campaigns probably should be creative and varied to attract consumers, the passage says nothing about this. Therefore, Choice C is incorrect because there is really nothing in the passage to support it. Choice d is incorrect; in fact, we could almost say that it is contradicted by the passage. The passage mentions bagels and sushi, but if anything it indicates that people are becoming less ethnic. It speaks of the multitude of products in the urban centers of each nation and the fact that people everywhere are becoming more alike in their tastes and preferences. In effect, it is saying that many people are not staying in their ethnic groups any more.

Passage:

According to Hume, all of life and its experiences is merely a passing fancy with nothing tying it together. There is no order, no organization, merely a dizzying array of fantastic and bewildering images. Kant revolutionized Western philosophy by proving that true experience consists of the judgments we impose upon the data of the senses. The senses yield information about the world, but its understanding which gives the world its true and proper form. Understanding applies its own laws on the sense experiences and transforms them into a coherent and consistent body of knowledge.

Reading Comprehension Test

Page 11 of 18

Kant called these laws categories, and said that they are intrinsic to the mind. The mind intuits them; they are basic to the mind. For Kant, space and time are forms of intuition and man can only experience his world within a spacio-temporal frame. Nothing can be known outside of this framework. Some of the following statements are true according to the passage, some are false.

7. Some of the following statements are true according to the passage, some are false.

I. According to Kant, categories are basic to the mind. II. Kant revolutionized Eastern philosophy. III. True experience comes directly from the data of the senses, according to Kant. IV. According to Kant, man can only experience his world within a framework of time and space.

a. Statements I and III are true. b. Statements I and IV are true.

c. Statements II and IV are true. d. Statements I, II, and IV are true.

ANSWER: B Solution: This may have been an intimidating question for a number of reasons: 1) it concerns philosophy, 2) it uses a number of words that you may not be familiar with, and 3) it has a strange form. First of all, dont be intimidated by the subject matter. You dont have to have a background in a certain area to answer basic reading comprehension questions. All the information you need should be right there in the passage. Second, you may often come across words youre not that familiar with when dealing with exam questions. Some makers of questions like to create long, complicated sentences and use unfamiliar words because its an easy way to create difficult questions.

Usually you can get around this by translating the sentences into your own words, or by looking for clues in other sentences or in the other choices given. Finally, theres the strange form of the question. This style of question is called a permutation item question. The easiest way to deal with a question of this type is to look at each statement separately to determine if it is true or false. Then, after

Reading Comprehension Test

Page 12 of 18

youve figured out that Statement I, for example, is true, make a note of it somewhere. If youre allowed to write in your exam booklet (or if you have a good eraser), make a T or write true next to the statement. Otherwise, make a note on your scrap paper. Then do the same for all the other statements. When youve taken care of all of them in this way, it will be very easy to answer the question.

Statement I is true because it is stated almost word for word in the sixth and seventh sentences. The passage says Kant called these laws categories and said they were intrinsic to the mind. The next sentence expands on this, saying that the mind intuits them; they are basic to the mind. Therefore, its safe to say that categories are basic to the mind. You may not have known what intrinsic meant, but reading along you can see that the author seems to be going on about the same thing in Sentence 7 that he started in Sentence 6. Therefore, you can guess that Statement I is true. Another way that you can figure out that Statement I is true is by putting a question mark next to it, examining each of the other statements, and then seeing how it fits into the scheme of things. As it turns out, Statement I is included as part of the answer in all the choices except Choice c. Choice C says that Statement II is true. As you will see in the next paragraph, Statement II is false. Therefore, Statement I has to be true.

Statement II is false because Kant revolutionized Western not Eastern philosophy. (As we said earlier, you need to read carefully and pay attention to every word.) Statement III is false because Kant thought that true experience consists of the judgments we impose upon the data of the senses. The senses yield information about the world, but its understanding which gives the world its true and proper form. Now, this is one of those passages that you might have wanted to translate into your own words to make sense of it. You could have said something like: True experience is the judgments we put on the info that comes from our senses. The senses give information, but understanding gives the true form. Then, you could have simplified it even more: True experience = judgments. Senses give information, but not the true form. By reorganizing the sentences in this way, it becomes clear that true experience only comes from the judgments we make; it doesnt come directly from the senses.

Statement IV is true according to the last two sentences which talk about space and time: For Kant, space and time are forms of intuition and man can only experience his world within a spacio-temporal frame. Nothing can be known outside of this framework. From the first of these two sentences, you probably figured out that spacio-temporal meant time and space even if you didnt know this to begin with. (They are in the same sentence, and spacio closely resembles space.) So, now that you know Statements I and IV are true, you can pick Choice b and feel secure. But lets say you are doing this on your own, and youre not too sure about I or IV, but you know that Statements II and III are false. By knowing that II and III are false, you can eliminate any choice that contains one of them. Since Choice a contains Statement III and Choices c and d contain Statement II, you are left with the only possible right answer Choice b. As you can see there are often many ways to arrive at the correct answer.

Reading Comprehension Test

Page 13 of 18

NOTES: REF:
Passage:

With the coming of the VCR and the camcorder, personal and affordable television production became a reality. Yet the same technological revolution that allowed the amateur to produce near studio-quality productions has given the television industry an amazing array of special effects. Through advances in computer technology, we are treated to such wonders as digital representation of athletic events, photos of athletes spinning into view, screens split any which way, and logos exploding into fireworks. And, of course, versions of this same technology are starting to be available in the home. Called desktop video, because all the components can fit on a standard desk or table, this system uses the computer to create and the VCR to print. They offer titling, computer graphics, animation, digital video effects, and the ability to mix live video with computer pictures. Whats needed is a computer to generate and manipulate images, a camera to originate pictures, a VCR or camcorder to use as a source for editing and overdubbing, another VCR to edit and re-record and finally the right software to make it all happen. Systems also take advantage of a variety of components like: character generators to create electronic text in a variety of styles and colors; video titling programs which mix words with pictures, animate images, and move from screen to screen; painting systems that let you draw, paint, or manipulate pictures; animation Systems that create moving images in two or three dimensions using realistic, shaded objects; and digital video effects hardware which let images and words roll, tumble, and dance around the screen. 8. According to the above passage,

a. the technological revolution has allowed the amateur to produce studio-quality

c. digital video effects hardware lets images and words roll around the screen.

productions. b. video titling systems allow you to d. a camera may be used for editing create electronic text and mix words and overdubbing. with pictures.

ANSWER: C The answer is Choice c. Unless you are a person with an electronics phobia, or someone who panics when faced with any technical writing, the only trick to this

Reading Comprehension Test

Page 14 of 18

NOTES: REF:

question is reading ~ choice a is incorrect because the second sentence in the passage says that the amateur is allowed to produce near studio-quality productions. The productions are close to being as good as the real thing, but theyre not quite good enough. Choice b is incorrect because, according to the passage, video titling programs mix words with pictures, animate images, and move from screen to screen. Its character generators which create electronic text. Choice d is incorrect because a camcorder (a device which contains the capability of both a camera and a VCR), not a camera, is used for editing and overdubbing. Choice c is correct and is found in the last sentence of the passage.

Passage:

Nearly twenty years ago, the United Farm Workers boycott of grapes was successful
in eliminating deadly poisons like DDT, DDE, and Dieldrin from fields worked by union workers. Yet today more than fifty other chemical products used on table grapes have been identified. The most lethal poisons are listed below. Each has injured or killed farm workers, and each might be present on the grapes you buy. There is methyl bromide, responsible for more occupationally-related deaths than any other pesticide and for permanent brain damage or blindness. Parathion and Phosdrin also can kill, sometimes very quickly. Since they are sprayed from the air, they also negatively affect people in the surrounding areas. (Ninety percent of all aerially sprayed pesticides miss their targets.) Dinoseb has a cumulative effect, causing illness and blindness. Finally, there is Captan which can cause cancer, birth defects, and changes in cell structure. This compound is the one most frequently found on storebought grapes.

Federal and state agencies have recommended the ban of each of these

substances yet they continue to be used today (1988). Pesticide poisoning has doubled

Reading Comprehension Test

Page 15 of 18

in the past ten years: more than 300,000 farm workers are made ill every year through pesticide exposure, and female farm workers are seven times more likely to suffer miscarriages than the average American woman. Of course, farm workers are not the only ones affected by the massive amounts of pesticides dumped on the nations food supply. A survey done in San Francisco showed that 44 percent of fruits and vegetables contained measurable amounts of pesticide residues and 42 percent of those contained more than one chemical. Pesticides are thought to be responsible for groundwater contamination in 23 states groundwater provides 50 percent of our drinking water supply.

9. All of the following are true according to the above passage, except;

a. methyl bromide is responsible for more occupationally-related fatalities than any

c. more than forty chemical products used on table grapes have been identified.

other pesticide. b. United Farm Workers have called d. Captan can cause cancer, birth for another nation-wide boycott of defects, and changes in cells. table grapes.

ANSWER: B Solution: This question illustrates the danger of using common sense and/or outside knowledge when neither are called for. You may know that the United Farm Workers called for another grape boycott, which was still going on when this booklet was written (1988), but the passage never says this. You may have inferred that this is the direction in which the passage is going, but, again, the passage never says it. Be very careful never to read things into passages on exam questions. Just stick to the facts as they are given unless you are forced to infer something, which occasionally you will have to do. Choice a is found in the fifth sentence of the passage. Choice c is covered by the second sentence. The sentence

Reading Comprehension Test

Page 16 of 18

NOTES: REF:
Passage:

says more than 50, but since more than 50 is also more than 40, there is no contradiction between Choice c and the passage. Choice d is found in the tenth sentence. Since changes in cell structure would be considered the same as changes in cells, there is no contradiction between the passage and the answer choice.

In spite of the fact that very few organizations keep accurate records of employee exit interviews, we can make some generalizations about why women leave and how long they stay. A small proportion of women leave their jobs after working less than a year. Usually, they leave because they dislike the work, their co-workers or supervisors, or because there has been a change in their personal lives. Another small percentage leave after ten to fifteen years, and they tend to leave for personal reasons or because it would significantly advance their careers. Women who leave after working between three and five years are likely to do so because they feel blocked in their careers. Many trainee jobs are merely interesting, moderate paying jobs without much hope for advancement.

Young women find the jobs attractive when they are first starting out, but become disappointed over time because they are looking for more and are qualified for more. Often women leave jobs that they like and are good at because they watch others advance more rapidly simply because they are men.

10. Which of the following statements is best supported by the passage?

a. Most young women find their first c. Organizations should keep jobs attractive if they lead to more accurate exit interviews. advanced

positions. b. Women become frustrated when they see men advancing rapidly.

d. Many trainee positions do not lead very far on the career ladder.

ANSWER: D Solution: Choice a is incorrect because it jumps to a conclusion. We dont know if most young women are looking to get ahead. It seems that this is the direction that the passage is taking, but the passage gives us nothing solid upon which to base

Reading Comprehension Test

Page 17 of 18

NOTES: REF:
Passage:

this conclusion. Choice b also jumps to a conclusion. The passage says that often women leave jobs... because they watch others advance more rapidly simply because they are men. This means that it is not the rapid advancement of men that bothers the women, but the fact that the men are advancing more rapidly simply because they are men, and not because they are more qualified. The wording of Choice b, on the other hand, is such that it seems the women dont want the men to advance rapidly period. Its subtle, but there is a difference. Choice c is incorrect because it is too strong. While it may seem that the author would like more accurate exit interviews so that he could determine why women leave their jobs, he never says this. The word should is what makes this choice unacceptable. Choice d is supported by the passage which states that many trainee jobs are merely interesting, moderate-paying jobs without much hope for advancement.

Most of us assume that we and other human beings are basically rational. Being rational means that once we figure out what we want to happen, we develop a theory or plan to bring it about, act in such a way as to foster the plan, seek to correct any action which interferes with our desired result, and feel good or badly depending on our degree of success. This concept of rationality assumes that people plan their actions and are therefore personally responsible for them. It also assumes that people would not act against their intentions. In addition, it assumes that people do their utmost to control their own ability to put their actions and desires in motion, rather than being pawns in someone else’s game or victims of fate.

Given these assumptions, it’s understandable that, when confronted with their own irrationality, most people become rather upset. They become frightened when they realize that their actions are countering their intentions and that they have been unaware of this fact. If they can’t trust themselves, whom can they trust? In this rational society, where concrete facts and the scientific method are worshipped, the only logical conclusion anyone can come to is that there is something wrong with herself. (Such thinking might run something like this: “If I want one thing, but do something that works against that outcome, there must be something wrong with my thinking, or my ability to plan and assess my actions.”) Therefore, people become confused, bewildered, ashamed, frustrated, guilty, and angry when they realize that they were unable to plan and put into effect what they intended to, that they were predisposed to sabotage their plans, and that others were aware of what they were doing all along. It would be far better for most of us if we realized that we very often do things for reasons that make no sense to our intellect. If we did, we would spend far less time defending our actions, and we would be able to spend more time figuring out how to get where we want to go and what’s really stopping us.

11. Which of the following is best supported by the passage?

Reading Comprehension Test

Page 18 of 18

a. Most of the time people act in ways c. that are consistent with their intentions. b. It is sometimes easier for someone d. else to see that we are working against ourselves than it is for us to see it.

If a person wants one thing but does another, there is something wrong with him or her. People do their utmost to control their own ability to put their desires in motion.

ANSWER: B Solution: The answer can be found in the third to the last sentence. This sentence says that people became upset when they realized that they were predisposed to sabotaging their own plans, and that others were aware of what they were doing all along. Choice a is incorrect because the passage doesnt indicate how often people act in ways that are consistent with what they want to achieve. Choice c is incorrect because the passage doesnt say this. While it does say that in this rational society. the only logical conclusion anyone can come to is that there is something wrong with him/herself, this doesnt mean that there actually Ls anything

NOTES: REF:

wrong with him or her. The author is merely giving the conclusion that would be reached by a person who believes that we should do things for rational reasons. It is the authors stated opinion, however, that we dont always do things for rational reasons, and we should just accept this. Choice d is incorrect because it isnt very well supported by the passage. The author says that it assumes that people do their utmost to control their own ability to put their actions and desires in motion. . . . The idea of rationality assumes that people will do their utmost, but the passage does not tell us if people do their utmost or not. Perhaps people sabotage themselves more than they work toward their goals. Perhaps people spend more time blaming others and outside circumstances. Perhaps people would rather have others take responsibility for them than be independent. Choice d is incorrect because it makes too great an assumption.

PATI TEST

PATI TEST ANSWER KEY
Multiple Choice Identify the letter of the choice that best completes the statement or answers the question. Situation: Nine colleges (O, P, Q, R, S, T, U, V, W) provide staff for three law firms: B, C,and D. Each college provides exactly one law student who is employed by only one of the three law firms. Law firm B cannot employ a law student from O, P, or Q. Law firm C cannot employ a law student from R, S, or T. Law firm D cannot employ a law student from U, V, or W. Law firm B must have exactly one more law student than law firm C. 1. If U and O are the only law students employed by law firm C, how many of the nine law students must be employed by Law firm D? a. 3 b. 4 c. 5 d. 6 e. 7 ANSWER: B First, list the nine colleges which provide law students: O P Q R S T U V W, and the law firms: B, C, and D. Next, summarize the rules: [1] One law student from each college [2] B, not O, P, or Q. C, not R, S, or T. D, not U, V, or W [3] B must employ one more law student than C We are told that U and O are the only law students employed by law firm C. Because of rule [4], which states that law firm B must employ one more law student than law firm C, we can conclude that B must employ three law students. If 2 are in C, and 3 are in B, for a total of 5, then the remaining 4 must be employed by law firm D, choice (B) is correct. NOTES: REF:

PATI TEST 1

2. If no law students are employed by Law firm D, which college must be represented in Law firm B? a. O b. P c. R d. U e. V ANSWER: C First, list the nine colleges which provide law students: O P Q R S T U V W, and the three law firms which employ the nine law students: B, C, and D. Next, summarize the rules: [1] One law student from each college [2] B, not O, P, or Q. C, not R, S, or T. D, not U, V, or W [3] B must employ one more law student than C If no law students are employed by law firm D, all nine must be employed by either law firm B or C. We are asked which country must be represented in law firm B. If all the law students must be employed by B or C, any law student that cannot be employed by C, must therefore be employed by B. Rule 2 tells us that R, S, and T are not allowed in C. Of the three that must be employed by law firm B, only R is one of the answer choices. NOTES: REF: 3. What is the greatest number of law students that could be employed by law firm D? a. 9 b. 8 c. 7 d. 6 e. 5

PATI TEST 1

ANSWER: D First, list the nine colleges which provide law students: O P Q R S T U V W, and the law firms: B, C, and D. Next, summarize the rules: [1] One law students from each college [2] B, not O, P, or Q. C, not R, S, or T. D, not U, V, or W [3] B must employ one more law students than C We know that there are nine law students, one from each college. Can all of them be employed by law firm D? We know from rule [2] that law students from three of the countries, U, V, and W, cannot be employed by law firm D. This eliminates three, and leaves six law students. Can all the other criteria be met if these six law students are placed in D? The only other rule we need to consider is rule [4], which states that law firm B must employ one more law student than law firm C. If six are in D, there are three law students remaining to be divided between B and C. If we place two into B and one into C, this satisfies rule [4]. Therefore, six is the maximum number of law students that could be employed by law firm D, choice D is correct. NOTES: REF: 4. Which colleges must provide staff for Law firm B, if W is the only one of the nine schools providing a law student for Law firm C? a. R and V b. S and T c. S and U d. T and V e. U and V ANSWER: E First, list the nine colleges which provide law students: O P Q R S T U V W, and the law firms: B, C, and D. Next, summarize the rules: [1] One law student from each college [2] B, not O, P, or Q. C, not R, S, or T. D, not U, V, or W [3] B must employ one more law student than C If only the law student from W is placed in C, rule [4] specifies that law firm B must have one more law student than C, so we know that B has two law students. If one is in C, and two are in B, the remaining law students, six, must be in law firm D. Rule [2] states that law students from U, V, or W cannot be employed by D. Therefore, these three must be employed by either C or B. Since we already know that W is in C, law students from U and V must be the two law students employed by law firm B. Choice E is correct.

PATI TEST 1

NOTES: REF: 5. If Law firm C has law students from only U, V, and Q, the law students employed by Law firm D must be: a. O and P b. O and R c. P and S d. P and T e. Q and S ANSWER: D First, list the nine colleges which provide law students: O P Q R S T U V W, and the law firms: B, C, and D. Next, summarize the rules: [1] One law student from each college [2] B, not O, P, or Q. C, not R, S, or T. D, not U, V, or W [3] B must employ one more law student than C If three law students are employed by law firm C, rule [4] tells us that four law students, or one more than the number employed by C, must be employed by law firm B. Since there are nine law students total, if we subtract these seven (three from C and four from B), this leaves two law students that must be employed by law firm D. Rule [2] tells us that these two law students cannot be U, V, or W. We already know that U and V are in law firm C, so W must be in law firm B. Of the remaining law students, we know, again from rule [2], that O, P, and Q cannot be in law firm B. We know from the problem that the law student from Q is already placed in C, so that leaves O and P who must be employed by law firm D. NOTES: REF: Situation: Five executives of a European corporation are in Madrid for a conference. Mr A can speak Spanish and Italian Mrs B can speak Spanish and English Miss C can speak English and Italian Ms D can speak French and Spanish Rev. E from Italy, can also speak French

PATI TEST 1

6. Which of the following can act as interpreter when Miss C and Ms D wish to confer? a. Only Mr A b. Only Mrs B c. Only Rev E d. Mr A or Mrs B e. Any of the other three executives. ANSWER: E Explanation: When C and D speak they can use English, Italian, French and Spanish between them. Mr A speaks Spanish and Italian. Mrs B speaks English and Spanish. Rev E speaks French and Italian. NOTES: REF: 7. If a sixth executive is brought in, to be understood by the maximum number of the original five, he should be fluent in a. English and French b. Italian and English c. French and Italian d. Italian and Spanish e. English and Spanish ANSWER: D Explanation: Three executives speak Spanish (A, B, and D). The other two (C and E) speak Italian. NOTES: REF: Situation: There are 7 flags arranged in a row. The flags are all single-color, solid color flags. The flags are red, white, green, and blue. There are exactly 2 red flags. At least one color has exactly 3 flags. At least one color has exactly 1 flag. A blue flag must not be next to another blue flag. A green flag must always be next to another green flag. A white flag must be in the middle of the row.

PATI TEST 1

8. If the blue flag can only be one flag away from a green flag, what position could a red flag be? a. b. c. d. e. Position #3 Position #4 Position #5 Position #7 Position #4 or #5

ANSWER: D How to Answer: Step 1. Draw a row with 7 spaces: _ _ _ _ _ _ _ Step 2. Find an anchor. A white flag must be in the middle of the row. Write a W in position 4: __ __ __ W __ __ __ Step 3. You have at least 1 white flag and exactly 2 red flags. There must be, at most, 4 flags that are either blue or green. We know that green must have at least two flags, since green flags must always be next to each other, so at most, we can have two blue flags, and at least, one blue flag. Step 4. Determine which color has the three flags. We know that red has only two flags--no more, no less. If white has three flags, this would only leave two flags (3 whites + 2 reds = 5). Since green alone must have two flags, this option would exclude blue, and thus, white cannot have three flags. If blue were to have three3 flags, then green would only have one, and this too violates the original condition regarding green. Thus, we have two red flags (explicit), exactly one white flag (implicit), exactly three green flags (implicit), and as a result of this, only one blue flag (implicit). This is an example of drawing warranted conclusions. Step 5. Graph your conditions. We have three green flags, and green flags MUST stand next to each other, so the green flags must occupy the three spaces on the left of the W or the three spaces on the right of the W. Conversely, the two reds and the one blue must be on the opposite side from the green flags. We do not know where the reds and blues actually stand: G G G W __ __ __ or __ __ __ W G G G (A) Position #3 . No, this must be occupied by a green or blue.

PATI TEST 1

(B) Position #4 . No, this is the middle position occupied by W. (C) Position #5 . No, see answer A. (D) Position #7 . * Correct - A red flag can be at #1 & #2 or #6 & #7. (E) Position #4 or #5 . No, see above. NOTES: REF: 9. What arrangement follows the conditions above and puts the most flags next to a differently colored flag? a. b. c. d. e. G,G,G,W,B,R,R B,R,R,W,G,G,G R,G,B,W,B,G,R R,B,R,W,G,G,G R,W,B,G,G,G,R

ANSWER: D (A) G,G,G,W,B,R,R . No, this satisfies the conditions, but does not mix the colors as the question asks, since the two reds are next to each other. (B) B,R,R,W,G,G,G . No, see above. (C) R,G,B,W,B,G,R . No, this separates the greens, violating a condition. (D) R,B,R,W,G,G,G . * Yes, this satisfies the conditions and mixes the colors as asked. (E) R,W,B,G,G,G,R . No, this moves the white incorrectly. NOTES: REF: Situation: The campers in a summer camp are broken up into five groups: the Nightingales, the Owls, the Peacocks, the Swallows, and the Turtle Doves. Each summer, three of the five groups participates in the mid-summer talent show. The rules specifying the order of participation of the groups in the show are as follows: Each group must participate at least once in any two consecutive years. No group participates for three consecutive years. Participate takes place in cycles, with each cycle ending when each of the five groups have participated three times. Only then does a new cycle begin. No group participates more than three times within any cycle.

PATI TEST 1

10. If the groups participating in the first year of a given cycle are the Nightingales, the Owls and the Peacocks, which of the following could be the groups participating the second year of the cycle? a. b. c. d. e. Nightingales, Owls, Swallows Nightingales, Owls, Turtle Doves Nightingales, Peacocks, Turtle Doves Owls, Peacocks, Turtle Doves Owls, Swallows, Turtle Doves

ANSWER: E NOTES: REF: 11. Any cycle for the groups' participation in the talent shows must be completed at the end of exact how many years? a. b. c. d. e. five six seven eight nine

ANSWER: A NOTES: REF: 12. Which of the following must be true about the three groups that participate in the talent show in the first year? a. b. c. d. e. At most two of them participate together in the third year At least two of them participate together in the second year All three of them participate together in the fourth year All three of them participate together in the fifth year None of them participate in the third year

ANSWER: A NOTES: REF:

PATI TEST 1

13. If in a particular cycle, the Nightingales, Owls and Swallows participate in the talent show in the first year, which of the following must be true? a. b. c. d. e. Nightingales participate in the second and third years Owls participate in the third and fourth years Nightingales and Owls participate in the third year Peacocks and Turtle Doves both participate in the fifth year Swallows and Turtle Doves both participate in the fifth year

ANSWER: D NOTES: REF: 14. If in a particular cycle, the Nightingales, Owls and Turtle Doves participate in the first year and if Owls and Peacocks participate in the fourth year, any of the following could be a clan that participates in the third year except: a. b. c. d. e. Nightingales, Owls, Peacocks Nightingales, Owls, Swallows Nightingales, Peacocks, Swallows Owls, Peacocks, Swallows Peacocks, Swallows, Turtle Doves

ANSWER: E NOTES: REF: 15. One form of reasoning holds that by eliminating all possible explanations until only one remains, that one should be accepted. Critics argue that the flaw in this form of reasoning is that one cannot know about all possible explanations. Which of the following examples best supports this criticism? a. b. c. d. e. the possible causes of heart disease the possible results of rolling dice the possible family members who left the house unlocked the possible candidates running for mayor of Atlanta, Georgia the possible countries with nuclear weapons

PATI TEST 1

ANSWER: A The possible causes of heart disease Point-Accepting the only explanation does not take all possible explanations into account. Issue-Description question/example NOTES: REF: 16. Doctor: The law of genetics holds that if both parents have brown eyes, then they can have only brown-eyed children. Patient: That is not true; my mother has blue eyes, and I have brown eyes. The patient has misinterpreted the doctor's statement to mean that a. b. c. d. only brown-eyed people can have blue-eyed children brown-eyed people cannot have blue-eyed children people with blue eyes invariably have blue-eyed children parents with the same eye color have children with a different eye color e. parents with different eye color have children with the same eye color ANSWER: C People with blue eyes invariably have blue-eyed children Point-Children have the same color eyes as their parent(s) Issue-Extension question/conclusion NOTES: REF: 17. Certain similarities between prehistoric art and the art of children has led some people to the mistaken conclusion that either early humans had the mentality of children or that they were as unskilled as children. These conclusions assume which of the following? a. Art that is considered sophisticated today must always have been considered sophisticated. b. What is easy for humans today must always have been easy. c. The significance of art is consistent over time. d. Prehistoric humans painted in the same way that children now paint. e. Modem humans have learned from prehistoric man.

PATI TEST 1

ANSWER: C The significance of art is consistent over time. Point-Similar results connote similar causes, regardless of context. Issue-Extension question/assumption NOTES: REF: 18. During the cultural revolution in China under Chairman Mao, thousands of "enemies of the republic" were killed. When Mao's critics accused him of confusing his personal enemies with enemies of the republic, he responded, "I deny the accusation, and the proof is that you are still alive." Which of the following assumptions was Mao making? a. b. c. d. e. All the enemies of the republic are dead. His critics are his personal enemies. Some personal enemies are also enemies of the republic. Enemies of the republic are not personal critics. Those killed were personal enemies.

ANSWER: A All the enemies of the republic are dead. Point-Any living critic proves that Mao did not have personal enemies killed. Issue-Extension question/assumption NOTES: REF: 19. Harry typically vacations in Tahoe. Two years ago, Harry spent his vacation in Madrid. In relation to the first sentence, what does the second sentence do? a. b. c. d. e. It clarifies an assumption. It notes an exception. It adds emphasis. It draws a conclusion. It makes a comparison.

ANSWER: B It notes an exception. NOTES:

PATI TEST 1

REF: 20. No one who has a sore throat need consult a doctor, because sore throats will recover without medical intervention. In recent years several cases of epiglottitis have occurred. Epiglottitis is a condition that begins with a sore throat and deteriorates rapidly in such a way that the throat becomes quite swollen, thus restricting breathing. Sometimes the only way to save a patient’s life in these circumstances is to insert a plastic tube into the throat below the blockage so that the patient can breathe. It is highly advisable in such cases that sufferers seek medical attention when the first symptoms occur, that is, before the condition deteriorates. Which one of the following is the best statement of the flaw in the argument? a. The author draws a general conclusion on the basis of evidence to a particular instance. b. The author assumes that similar effects must have similar causes. c. The author uses a medical term, “epiglottitis,” and does not clarify its meaning. d. The author makes two claims that contradict each other. e. The author bases her conclusion at the end of the passage on inadequate evidence. ANSWER: D The author makes two claims that contradict each other. (is the best statement of the flaw in the argument) NOTES: REF: 21. The end of overcrowding at colleges and universities provides them with the opportunity to improve the quality of the educational services they offer. As enrollment declines, services and campus facilities should better serve student needs. If true, which of the following statements most weakens the above conclusion? a. The quality of educational services does not depend on the variety of services offered. b. Fees paid by students are the major source of funding for educational services.

PATI TEST 1

c. Educational services are a critical factor in a student's choice of school. d. As campus facilities grow older, their maintenance becomes more expensive. e. Student needs are different than they were when colleges and universities were overcrowded. ANSWER: B Fees paid by students are the major source of funding for educational services. Point-As demands on resources decrease, services and facilities should improve. Issue-Extension question/weakening evidence NOTES: REF: 22. When pregnant laboratory rats are given caffeine equivalent to the amount a human would consume by drinking six cups of coffee per day, an increase in the incidence of birth defects results. When asked if the government would require warming labels on products containing caffeine, a spokesperson stated that it would not, because if the finding of these studies were to be refuted in the future, the government t would lose credibility. Which of the following is most strongly suggested by the government's statement above? a. b. c. d. e. A warning that applies to a small population is inappropriate. Very few people drink as many as six cups of coffee a day. There are doubts about the conclusive nature of studies on animals. Studies on rats provide little data about human birth defects. The seriousness of birth defects involving caffeine is not clear.

ANSWER: C There are doubts about the conclusive nature of studies on animals. Point-If the government acts prematurely, it loses credibility. Issue-Extension question/conclusion NOTES: REF:

PATI TEST 1

23. Sam: Olive oil can help prevent heart attacks, according to physicians. Betty: It cannot. My mother cooked with olive oil her entire life, and she died of a heart attack last year. Betty's statement can best be countered by pointing out that a. b. c. d. Betty's mother was an exception other factors could have nullified the influence of the olive oil Betty does not know that her mother always cooked with olive oil It has never been scientifically proven that olive oil causes heart attacks e. Betty's mother might have used olive oil irregularly ANSWER: B Other factors could have nullified the influence of the olive oil Point-Olive oil did not prevent heart attack in Betty's mother. Issue-Extension question/weakening evidence NOTES: REF: 24. Teresa has missed the last three practices of the dance step. She cannot perform the maneuver. In relation to the first sentence, what does the second sentence do? a. b. c. d. e. It states a consequence. It suggests a cause. It offers proof. It limits a preceding idea. It adds emphasis.

ANSWER: A It states a consequence. NOTES: REF:

PATI TEST 1

25. A study has shown that there are still millions of people who are unaware that they endanger their health by smoking cigarettes. This is so despite government campaigns to warn people of the dangers of smoking. Reluctantly, one has to draw the conclusion that the mandatory warnings that tobacco companies are required to print have had no effect. Which one of the following, if true, would refute the argument in the passage? a. Many people who continue to smoke are aware of the dangers of smoking. b. Some people smoke cigarettes for legitimate reasons. c. Government has had to force companies to warn potential customers of the dangers of their products. d. Some people who are aware of the dangers of smoking were made aware of them by the mandatory warnings. e. Smoking is clearly responsible for a substantial proportion of preventable illness in the country. ANSWER: D Some people who are aware of the dangers of smoking were made aware of them by the mandatory warnings, (refutes the argument). NOTES: REF:

Sponsor Documents

Or use your account on DocShare.tips

Hide

Forgot your password?

Or register your new account on DocShare.tips

Hide

Lost your password? Please enter your email address. You will receive a link to create a new password.

Back to log-in

Close